Sei sulla pagina 1di 95

1998 Harvard/MIT Math Tournament

CALCULUS Answer Sheet

Name:

School: Grade:

1 6

2 7

3 8

4 9

5 10

TOTAL:
CALCULUS

Question One. [3 points]


Farmer Tim is lost in the densely-forested Cartesian plane. Starting
from the origin he walks a sinusoidal path in search of home; that
is, after t minutes he is at position (t,sint) .
Five minutes after he sets out, Alex enters the forest at the origin
and sets out in search of Tim. He walks in such a way that after he
has been in the forest for m minutes, his position is (m,cos t).
What is the greatest distance between Alex and Farmer Tim while
they are walking in these paths?

Question Two. [3 points]


A cube with sides 1m in length is filled with water, and has a tiny
hole through which the water drains into a cylinder of radius 1m.
If the water level in the cube is falling at a rate of 1 cm s , at what
rate is the water level in the cylinder rising?

Question Three. [4 points]


2
Find the area of the region bounded by the graphs of y = x , y = x ,
and x = 2 .

Question Four. [4 points]


1

x x x 2 3 ∫ f(x)dx
Let f(x) = 1 + + + +…, for −1 ≤ x ≤ 1. Find e0 .
2 4 8

Question Five. [5 points]


x
sin(1−x)
Evaluate lim x .
x→1
Question Six. [5 points]
Edward, the author of this test, had to escape from prison to work
in the grading room today. He stopped to rest at a place 1,875 feet
from the prison and was spotted by a guard with a crossbow.
The guard fired an arrow with an initial velocity of 100 ft s . At the
same time, Edward started running away with an acceleration of
1 ft s 2 . Assuming that air resistance causes the arrow to decelerate at
1 ft s 2 and that it does hit Edward, how fast was the arrow moving at
ft
the moment of impact (in s )?

Question Seven. [5 points]


A parabola is inscribed in equilateral triangle ABC of side length 1 in
the sense that AC and BC are tangent to the parabola at A and B,
respectively:
Find the area between AB and the parabola.

Question Eight. [6 points]


Find the slopes of all lines passing through the origin and tangent to
2 3
the curve y = x + 39 x − 35.

Question Nine. [7 points]



1
Evaluate ∑ n⋅ 2
n= 1
n −1

Question Ten. [8 points]


Let S be the locus of all points (x, y) in the first quadrant such that
x y
+ = 1 for some t with 0<t<1. Find the area of S.
t 1− t
HMMT 1998: Calculus Solutions

1. Problem: Farmer Tim is lost in the densely-forested Cartesian plane. Starting from the origin he walks
a sinusoidal path in search of home; that is, after t minutes he is at position (t, sin t).
Five minutes after he sets out, Alex enters the forest at the origin and sets out in search of Tim. He
walks in such a way that after he has been in the forest for m minutes, his position is (m, cos t).
What is the greatest distance between Alex and Farmer Tim while they are walking in these paths?

Solution: At arbitrary time t, Farmer Tim is at position (t, psin t) and Alex is at position (t − 5, cos t).
Hence at time t, the distance, d, between Tim and Alex is d = (sin t − cos t)2 + 25. To find the maximum
value of d, we solve for t such that dd dt = 0.
dd (sin t−cos t)(cos t+sin t) 2 2
dt =
√ 2
. Then dd 2 2
dt = 0 ⇒ sin t − cos t = 0 ⇒ sin t = cos t. Equality happens if t is
(sin t−cos t) +25
any constant multiple of π4 .
Notice that to maximize d, we need to maximize (sin t − cos t)2 . This is achieved when cos t = − sin t.
Because we determined earlier that t is a constant multiple of π4 , then with this new condition, we see that
t must be a constant multiple of 3π
4 .
2

Then (sin t − cos t) = 2 ⇒ d = 29 .

2. Problem: A cube with sides 1m in length is filled with water, and has a tiny hole through which the
water drains into a cylinder of radius 1m. If the water level in the cube is falling at a rate of 1 cm/s, at what
rate is the water level in the cylinder rising?

Solution: The magnitude of the change in volume per unit time of the two solids is the same. The change
in volume per unit time of the cube is 1 cm·m2 /s. The change in volume per unit time of the cylinder is
π · dh 2 dh
dt · m , where dt is the rate at which the water level in the cylinder is rising.
dh 1
Solving the equation π · dt · m2 = 1 cm · m2 /s yields π cm/s .

3. Problem: Find the area of the region bounded by the graphs of y = x2 , y = x, and x = 2.

Solution: There are two regions to consider. First, there is the region bounded by y = x2 and y = x, in
the interval [0, 1]. In this interval, the values of y = x are greater than the values of y = x2 , thus the area is
Z 1
x − x2 dx.

calculated by
0
Second, there is the region bounded by y = x2 and y = x and x = 2, in the interval [1, 2]. In this interval,
Z 2
x2 − x dx.

the values of y = x2 are greater than the values of y = x, thus the area is calculated by
Z 1 Z 2 1
2 2
 
Then the total area of the region bounded by the three graphs is x − x dx + x − x dx = 1 .
0 1
s Z
1

2 3
f (x)dx
x x x
4. Problem: Let f (x) = 1 + 2 + 4 + 8 + . . ., for −1 ≤ x ≤ 1. Find e 0 .

1 2
Solution: Observe that f (x) is merely an infinite geometric series. Thus f (x) = 1− x = 2−x . Then
2
Z 1
2 √ √
= 2 ln 2. Then e2 ln 2 = 22 = 2 .
0 2 − x
x
5. Problem: Evaluate lim x sin(1−x) .
x→1
x x
sin(1−x) sin(1−x)
Solution: Rewrite the expression to evaluate as eln x . Then we must evaluate lim eln x .
  x→1
x x
lim ln x sin(1−x) = lim ln x . Because direct calculation of the limit results in indeterminate
x→1 x→1 sin(1 − x)
 
x
form ( 10 · 0), we can use L’Hopital’s rule to evaluate the limit. By L’Hopital’s rule, lim ln x =
x→1 sin(1 − x)
ln x + 1
lim . This limit is simply -1.
x→1 − cos(1 − x)

1
x
sin(1−x)
Hence lim eln x = e−1 = 1
e .
x→1

6. Problem: Edward, the author of this test, had to escape from prison to work in the grading room
today. He stopped to rest at a place 1,875 feet from the prison and was spotted by a guard with a crossbow.
The guard fired an arrow with an initial velocity of 100 ft/s. At the same time, Edward started running
away with an acceleration of 1 ft/s2 . Assuming that air resistance causes the arrow to decelerate at 1 ft/s2
and that it does hit Edward, how fast was the arrow moving at the moment of impact (in ft/s)?

Solution: We use the formula for distance, d = 21 at2 + vt + d0 . Then after t seconds, Edward is at
location 1875 + 12 (1)(t2 ) from the prison. After t seconds, the arrow is at location 21 (−1)(t2 ) + 100t from the
prison. When the arrow hits Edward, both objects are at the same distance away from the tower. Hence
1875 + 21 (1)(t2 ) = 12 (−1)(t2 ) + 100t. Solving for t yields t2 − 100t + 1875 = 0 ⇒ t = 25 or t = 75. Then it
must be t = 25, because after the arrow hits Edward, he will stop running.
After 25 seconds, the arrow is moving at a velocity of 100-25(1) = 75 ft/s .

7. Problem: A parabola is inscribed in equilateral triangle ABC of side length 1 in the sense that AC
and BC are tangent to the parabola at A and B, respectively. Find the area between AB and the parabola.

Solution: Suppose A = (0, 0), B = (1, 0), and C = ( 12 , 23 ). Then the parabola in question goes through
√ √
(0, 0) and (1, 0) and has tangents with slopes of 3 and − 3, respectively, at these points. Suppose the
dy
parabola has equation y = ax2 + bx + c. Then dx = 2ax + b.
dy
√ √
At point (0, 0), dx = b. Also the slope at (0, 0), as we determined earlier, is 3. Hence b = 3. Similarly,
dy
√ √
at point (1, 0), dx = 2a + b. The slope at (1, 0), as we determined earlier, is − 3. Then a = − 3.
√ √
Since the parabola goes through (0, 0), c = 0. Hence the equation of the parabola is y = − 3x2 + 3x.
The desired area is simply the area under the parabolic curve in the interval [0, 1].
Z 1 √ √  √
Hence − 3x2 + 3x dx = 63 .
0

8. Problem: Find the slopes of all lines passing through the origin and tangent to the curve
y 2 = x3 + 39x − 35.

Solution: Any line passing throug the origin has equation y = mx, where m is the slope of the line. If a
dy
line is tangent to the given curve, then at the point of tangency, (x, y), dx = m.
dy dy 2
2 3x +39
First, we calculate dx of the curve: 2ydy = 3x dx + 39dx ⇒ dx = 2y . Substituting mx for y, we get
the following system of equations:

m2 x2 = x3 + 39x − 35
3x2 + 39
m=
2mx

Solving for x yields the equation x3 − 39x + 70 = 0 ⇒ (x − 2)(x + 7)(x − 5) = 0 ⇒ x = 2 or x = −7 or


x = 5. These solutions indicate the x-coordinate of the points

at which the desired lines are tangent to the
51
curve. Solving for the slopes of these lines, we get m = ± 2 for x = 2, no real solutions for x = −7, and
√ √ √
285 51 285
m=± 5 for x = 5. Thus m = ± 2 ,± 5 .

X 1
9. Problem: Evaluate n−1
.
n=1
n · 2
2
x
Solution: Note that if we take the integral of f (x) in problem 4, we get the function F (x) = x + 2·2 +
3
x 1 1
3·22 + . . .. ZEvaluating this integral in the interval [0, 1], we get 1 + 2·2 + 3·22 + . . ., which is the desired sum.
1
2
Hence dx = 2 ln 2.
0 2 − x

2
x y
10. Problem: Let S be the locus of all points (x, y) in the first quadrant such that t + 1−t = 1 for some
t with 0 < t < 1. Find the area of S.

Solution: Solving for t in √ the given equation, we get t2 + (y − x − 1)t + x = 0. Using the quadratic
(x+1−y)± (y−x−1)2 −4x
equation, we get t = 2 . For all valid combinations of (x, y), t is positive and less than 1
(this is easy to see by inspection). All valid combinations of (x, y) are those that make (y − x − 1)2 − 4x ≥ √ 0.
Solving for y in the equation (y −x−1)2 −4x = 0 yields y 2 −(2x+2)y +(x−1)2 ≥ 0 ⇒ y = (x+1)±2 x.
y
In the original equation, it is given that xt + 1−t = 1, and 0 < t < 1. This implies that x, y < 1. Then the
2

only possible y < 1 that satisfies (y − x − 1) − 4x = 0 is y = x + 1 − 2 x. √
Then to satisfy the inequality (y − x − 1)2 − 4x ≥ 0, we must have y ≤ x + 1 − 2 x. Recall that this is

when 0 < y < 1. Hence we integrate in the interval [0, 1]: ∈10 x + 1 − 2 x = 16 .

3
Calculus
Harvard-MIT Math Tournament
February 27, 1999

1. Find all twice differentiable functions f (x) such that f 00 (x) = 0, f (0) = 19, and f (1) = 99.

2. A rectangle has sides of length sin x and cos x for some x. What is the largest possible area of
such a rectangle?

3. Find √
Z 4π 2 sin x
√ ( + 1)dx.
−4π 2 1 + x4

4. f is a continuous real-valued function such that f (x + y) = f (x)f (y) for all real x, y. If f (2) = 5,
find f (5).

5. Let f (x) = x + 2x+


1
1 for x > 0. Find f (99)f 0 (99).
2x+ 1
..
2x+ .

d
6. Evaluate dx (sin x − 43 sin3 x) when x = 15.

7. If a right triangle is drawn in a semicircle of radius 1/2 with one leg (not the hypotenuse) along
the diameter, what is the triangle’s maximum possible area?

8. A circle is randomly chosen in a circle of radius 1 in the sense that a point is randomly chosen
for its center, then a radius is chosen at random so that the new circle is contained in the original
circle. What is the probability that the new circle contains the center of the original circle?

9. What fraction of the Earth’s volume lies above the 45 degrees north parallel? You may assume
the Earth is a perfect sphere. The volume in question is the smaller piece that we would get if the
sphere were sliced into two pieces by a plane.

10. Let An be the area outside a regular n-gon of side length 1 but inside its circumscribed circle,
let Bn be the area inside the n-gon but outside its inscribed circle. Find the limit as n tends to
An
infinity of B n
.
Calculus Solutions
Harvard-MIT Math Tournament
February 27, 1999

Problem C1 [3 points]

Find all twice differentiable functions f (x) such that f ′′ (x) = 0, f (0) = 19, and f (1) = 99.

Solution: Since f ′′ (x) = 0 we must have f (x) = ax + b for some real numbers a, b. Thus f (0) =
b = 19 and f (1) = a + 19 = 99, so a = 80. Therefore f (x) = 80x + 19.

Problem C2 [3 points]

A rectangle has sides of length sin x and cos x for some x. What is the largest possible area of such
a rectangle?

1
Solution: We wish to maximize sin x · cos x = 2 sin 2x. But sin 2x ≤ 1, with equality holding for
x = π/4, so the maximum is 21 .

Problem C3 [4 points]

Find √
Z 4π 2 sin x
√ ( + 1)dx.
−4π 2 1 + x4

sin x
Solution: The function 1+x 4 is odd, so its integral over this interval is 0. Thus we get the same

answer if we just integrate dx, namely, 8π 2.

Problem C4 [4 points]

f is a continuous real-valued function such that f (x + y) = f (x)f (y) for all real x, y. If f (2) = 5,
find f (5).

Solution 1: Since
√ f (nx) = f (x)n for all integers n, f (5) = f (1)5 and f (2) = f (1)2 , so f (5) =
5/2
f (2) = 25 5.

Solution 2: More generally, since f (nx) = f (x)n for all integers n, f (1) = c = f (1/n)n for some
constant c and all integers n. Thus f (k/n) = f (1/n)k = f (1)k/n = ck/n for all rational numbers

k/n. By continuity,
√ it follows that f (x) = cx for all real numbers x. Since f (2) = 5, c = 5, so
f (5) = 25 5.

1
Problem C5 [5 points]

1
Let f (x) = x + 2x+ 1 for x > 0. Find f (99)f ′ (99).
2x+ 1
..
2x+ .

Solution: Assume that the continued fraction converges (it √ does) so that f (x) is well defined. Notice
1 2 2
that f (x) − x = x+f (x) , so f (x) − x = 1, or f (x) = 1 + x2 (we need the positive square root
x
since x > 0). Thus f ′ (x) = √1+x 2
, so f (x)f ′ (x) = x. In particular, f (99)f ′ (99) = 99.

Problem C6 [5 points]

d
Evaluate dx (sin x − 43 sin3 x) when x = 15.

Solution: Of course this problem can be done by brute force, differentiating and then using the half
angle formula to find sin and cos of 15, but there is a quicker way. eix = cos x + i sin x, so sin(3x)
is the imaginary part of (cos x + i sin x)3 , which is 3 cos2 x sin x − sin3 x = 3 sin x − 4 sin3 x, so √
the
1
expression we are differentiating is just 3 sin(3x). Hence the derivative is cos(3x), and cos 45 = 22 .

Problem C7 [5 points]

If a right triangle is drawn in a semicircle of radius 1/2 with one leg (not the hypotenuse) along
the diameter, what is the triangle’s maximum possible area?

Solution: It is easy to see that we will want one vertex of the triangle to be where the diameter
meets the semicircle, so the diameter is divided into segments of length x and 1 − x, where x is the
length of the leg on the diameter. The other leg of the triangle will
√ be the geometric mean of these
p x x(1−x)
two numbers, x(1 − x). Therefore the area of the triangle is 2 , so it will be maximized

d 3
when dx (x − x4 ) = 3x2 − 4x3 = 0, or x = 3/4. Therefore the maximum area is 3 3
32 .

Problem C8 [6 points]

A circle is randomly chosen in a circle of radius 1 in the sense that a point is randomly chosen
for its center, then a radius is chosen at random so that the new circle is contained in the original
circle. What is the probability that the new circle contains the center of the original circle?

Solution: If the center of the new circle is more than 1/2 away from the center of the original
circle then the new circle cannot possibly contain the center of the original one. Let x be the
distance between the centers (by symmetry this is all we need to consider), then for 0 ≤ x ≤ 1/2
x
the probability of the new circle containing the center of the original one is 1 − 1−x . Hence we need
R 1/2 x 1 R 1/2 x
to compute 0 (1 − 1−x )dx = 2 − 0 1−x dx. To evaluate the integral, we can integrate by parts
to get
Z 1/2 1 1 1
1/2 1/2
−x ln(1 − x)|0 − − ln(1 − x)dx = − ln( ) − [(1 − x) ln(1 − x) − (1 − x)]0 = ln 2 − .
0 2 2 2

2
x 1 R 1/2 x
Alternatively, we can use polynomial division to find that 1−x = −1 + 1−x , so 0 1−x dx =
R 1/2 1
0 (−1 + 1−x )dx = ln 2 − 12 . Therefore the probability is 1
2 − (ln 2 − 21 ) = 1 − ln 2.

Problem C9 [7 points]

What fraction of the Earth’s volume lies above the 45 degrees north parallel? You may assume
the Earth is a perfect sphere. The volume in question is the smaller piece that we would get if the
sphere were sliced into two pieces by a plane.

Solution 1: Without loss of generality, look at a sphere of radius 1 centered at the origin. If you like
cartesian coordinates, then you can slice the sphere into discs with the same z coordinate, which
√ √
have radius 1 − z 2 , so the region we are considering has volume √1 2/2 π(1 − z 2 )dz = π( 23 − 5122 ),
R

8−5 2
and dividing by 4π/3 we get 16 .

Solution 2: For those who prefer spherical coordinates, we can find the volume of the spherical cap
plus a cone whose vertex is the center of the sphere. This region is where r ranges from 0 to 1, θ
ranges from 0 to 2π, and φ ranges from 0 to π/4. Remembering we need to subtract off the volume of
the cone, which has height √12 and a circular base of radius √12 , then divide by 34 π to get the fraction
R 1 R 2π R π/4
r 2 sin φdφdθdr− 31 π( √1 )2 √1
0 0 0
of the volume of the sphere, we find that we need to evaluate 4π/3
2 2
. The

4π−2π 2
integral is just 2π 13 (− cos π/4 + cos 0) = 6 . Putting this back in the answer and simplifying

8−5 2
yields 16 .

Solution 3: Cavalieri’s Principle states that if two solids have the same cross-sectional areas at every
height, then they have the same volume. This principle is very familiar in the plane, where we know
that the area of a triangle depends only on the base and height, not the precise position of the apex.
To apply it to a sphere, consider a cylinder with radius 1 and height 1. Now cut out a cone whose
base is the upper circle of the cylinder and whose apex is the center of the lower circle. Then at a
height z the area is π(12 − z 2 ), exactly the same as for the upper hemisphere! The portion lying
above the the 45 degrees north parallel is that which ranges from height √12 to 1. The volume of
the cylinder in this range is π · 12 (1 − √12 ). The volume of the cone in this range is the volume of the
entire cone minus the portion from height 0 to √1 , i.e., 31 π(12 · 1 − ( √12 )2 √12 ). Therefore the fraction
2
π(1− √1 )− 13 π(1− 1
√ ) √
8−5 2
of the Earth’s volume that lies above the 45 degrees north parallel is 2
4π/3
2 2
= 16 .

p
Solution 4: Another way to approach this problem is to integrate the function 1 − x2 − y 2 over
the region x2 + y 2 ≤ √12 , subtract off a cylinder of radius and height √12 , then divide by the volume
of the sphere. One could also use the nontrivial fact that the surface area of a portion of a sphere of
radius r between two parallel planes separated by a distance z is 2πr2 z, so in particular the surface
area of this cap is 2π(1 − √12 ). Now, the ratio of the surface area of the cap to the surface area of
the sphere is the same as the ratio of the volume of the cap plus the cone considered in Solution 2
to the volume of the whole sphere, so this allows us to avoid integration entirely.

3
Problem C10 [8 points]

Let An be the area outside a regular n-gon of side length 1 but inside its circumscribed circle, let
Bn be the area inside the n-gon but outside its inscribed circle. Find the limit as n tends to infinity
An
of B n
.

Solution: The radius of the inscribed circle is 21 cot πn , the radius of the circumscribed circle is
1 π n π
2 csc n , and the area of the n-gon is 4 cot n . The diagram below should help you verify that these
are correct.

r 1/2

π/n

π 2 π
An π(csc n ) −n cot n
Then An = π( 12 csc πn )2 − n
4 cot πn , and Bn = n
4 cot πn − π( 12 cot πn )2 , so Bn = π π 2 . Let s
n cot n −π(cot n )
An
denote sin πn and c denote cos πn . Multiply numerator and denominator by s2 to get B n
π−ncs
= ncs−πc 2.
( π )3 ( π )2
Now use Taylor series to replace s by πn − n6 + . . . and c by 1 − n2 + . . .. By l’Hôpital’s rule
it will suffice to take just enough terms so that the highest power of n in the numerator and
denominator is determined, and that turns out to be n−2 in each case. In particular, we get the
π π 3 2 π3
An π−n n +n 23 ( n ) +... 3 n2
+...
limit Bn = π 2 π 3
n n −n 3 ( n ) −π+π( n π 2
) +...
= 1 π3
→ 2.
+...
3 n2

4
Cal ulus
Ri e Mathemati s Tournament 2000

1. Find the slope of the tangent at the point of in e tion of y = x3 3x2 + 6x + 2000.
2. Karen is attempting to limb a rope that is not se urely fastened. If she pulls herself
up x feet at on e, then the rope slips x3 feet down. How many feet at a time must she
pull herself up to limb as eÆ iently as possible?
3. A re tangle of length 41  and height 4 is bise ted by the x-axis and is in the rst and
fourth quadrants. The graph of y = sin(x) + C divides the area of the square in half.
What is C?
4. For what value of x (0 < x < 2 ) does tan x + ot x a hieve its minimum?
P i
5. For 1 < x < 1, let f (x) = i=1 xi . Find a losed form expression (a losed form
1

expression is one not involving summation) for f .


p
6. A hallway of width 6 feet meets a hallway of width 6 5 feet at right angles. Find the
length of the longest pipe that an be arried horizontally around this orner.
7. An envelope of a set of lines is a urve tangent to all of them. What is the envelope of
the family of lines y = x20 + x(1 x12 ), with x0 ranging over the positive real numbers?
0
Hint: slope at a point P on a urve is dxdy
jP .
8. Find
R  ln sin d.
2
0
v
u s r q
u
t p
9. Let f (x) = x + 0 + x + 0 + x + : : :. If f (a) = 4, then nd f (a). 0

p
10. A mirror is onstru ted in the shape of y equals  x for 0  x  1, and 1 for
1 < x < 9. A ray of light enters at (10,1) with slope 1. How many times does it boun e
before leaving?
Cal ulus Solutions
Ri e Mathemati s Tournament 2000

1. y = x3 3x2 + 6x + 2000, so y 0 = 3x2 6x + 6 and y 00 = 6x 6, so the point of in e tion


is the solution to 6x 6 = 0, or x = 1. At x = 1, the slope is f 0 jx=1 = 3(1)2 6(1) + 6 =
3.

2. The hange in Karen's position is x x3 . The optimal length to limb is at a p riti al


point. The only realisti riti al point is at the solution to 1 3x = 0 or x = 33 .
2

3.
R 1

sin x + C = 0, from the
4
statement of the problem. So, [ os x + Cx℄j04 = 0. p

Thus,
0

p 2 2 4
 
os 4 + 4 C + 1 = 0. So, 2 + 4 C + 1 = 0, and solving for C , we nd that C =  .
2

4. Let y = tan x. So, we want to nd the minimum of y + y1 , where 0  y  1. Taking the


derivative and minimizing, we nd that the minimum o urs at y = 1, so the minimum
of the given fun tion o urs at ar tan 1 = 4 .
P i P
5. f (x) = 1 x . So, f 0 (x) = 1 xi = 1 . Thus, f (x) = ln(1 x).
i=1 i i=0 1 x
6. Assume the pipe barely ts around the orner (i.e. it is in p onta t with the orner).
The lower orner is at (0; 0) and the upper orner is at (6; 6 5). Call x0 the point on
p r longestpa pipep ould
the lower wall it hits at the tightest spot. Given an x0 , the
2
be with
one end at x0 and leaning against the (6; 6 5) orner is x20 + (6 5 + 36 x0 6 ) . We want
5

the minimum of all of these "longest pipes", be ause the pipe needs to t at all angles
around the orner. Taking the derivative (without the square root for simpli ity) and
setting it equal to 0, we need to solve x30 6x20 + 36x0 1296 = 0. Wep an qui kly nd
that x0 = 12 is the only good solution, so the maximum length is 12 6.
dy
7. Using the hint, take dx . Set this equal to 0 and solve for x relative to x0 . Plug this in
for x0 in the given family of lines to obtain the envelope y = x + x1 ; x > 0.
8. Let I denote the given integral. Under the transformation  !  , I transforms to
R 2 ln( os())d. So, 2

0
R
2I R= 02 ln(sin  os )d
= 0 (ln(sin 2R) ln 2)d(2)=2
= 2 ln 2 + 21 0 sin( )d giving I  ln 2.
pi R 
= 2  ln 2 + 0 sin( )d
2
= 2

= pi2  ln 2 + I
9. Note rst that ([f (x)℄2 x)2 = f (x), so if f (a) = 4, then (16 a)2 = 4, so a = 14.
f 0 (x)
0
Now, f (x) =
1+
2([f (x)℄2 x)
, so f 0 (14) = 4 .
2f (x) 31
10. Solution: Throughout this solution we will use the fa t that when light boun es o a
mirror, the angle of in iden e is equal to the angle of re e tion. First the beam hits
the point (8,-1), then (6,1), (4,-1), (2,1), and then is travelling
p5 1 p5 along the line p y = x 1.
Thus the beam hits the parabola at the ppoint (1 + 2 ; 2 ).p To estimate 5, noti e
1
p
that 222 = 484 and 232 = 529, so 5 = 10 500
= 2:2 : : :. Thus 1 2 5 = :6 : : :, so the light
hits the parabola at approximately (.4,-.6). The slope of the tangent to the parabola
at this point is 21 (:4) 1=2 , whi h is about -.8, so we need to nd the slope of the beam
after it re e ts o of this tangent. For purposes of nding this slope, hange oordinates
so that the point of interse tion is the origin. The beam is oming in along y = x, and
y = 1:2x is perpendi ular to the tangent. The diagram below should larify the setup.

perpendicular to
tangent
reflected
beam

y=x

tangent

We will nd the new path of the light by nding the re e tion about the line y = 1:2x
of a point on its in oming path. We know the point (1,1.2) is on the line y = 1:2x, so a
perpendi ular through this point is y 1:2 = :8(x 1), whi h interse ts y = x at the
point (1.1,1.1). Thus the new path goes through the point (.9,1.3), so it has slope 1.4
(all values rounded to one de imal pla e). Going ba k to our original oordinate system,
the light is now travelling along the line y + :6 = 1:4(x :4), so it next hits the mirror
at (1.5,1). After that the x oordinate in reases by 2=1:4 = 1:4 between boun es, so it
hits (2.9,-1), (4.3,1), (5.7,-1), (7.1,1), (8.5,-1), and nally (9.9,1). A loser examination
of the approximations made (e.g. by re ning them to two de imal pla es) reveals that
the last boun e is a tually further to the left (at (9.21,1), to be more pre ise), so indeed
the light does boun e 12 times.
Calculus Test
Harvard-MIT Math Tournament
March 3, 2001

1. A sequence of ants walk from (0, 0) to (1, 0) in the plane. The nth ant walks along
n semicircles of radius n1 with diameters lying along the line from (0, 0) to (1, 0). Let Ln be
the length of the path walked by the nth ant. Compute lim Ln .
n→∞

2. The polynomial 3x5 −250x3 +735x is interesting because it has the maximum possible
number of relative extrema and points of inflection at integer lattice points for a quintic
polynomial. What is the sum of the x-coordinates of these points?

3. A balloon that blows up in the shape of a perfect cube is being blown up at a rate such
that at time t fortnights, it has surface area 6t square furlongs. At how many cubic furlongs
per fortnight is the air being pumped in when the surface area is 144 square furlongs?

4. What is the size of the largest rectangle that can be drawn inside of a 3-4-5 right
triangle with one of the rectangle’s sides along one of the legs of the triangle?

5. Same as question 4, but now we want one of the rectangle’s sides to be along the
hypotenuse.

6. The graph of x2 − (y − 1)2 = 1 has one tangent line with positive slope that passes
through (x, y) = (0, 0). If the point of tangency is (a, b), find sin−1 ( ab ) in radians.

7. Find the coefficient of x12 in the Maclaurin series (i.e. Taylor series around x = 0) for
1
1−3x+2x2
.


cot−1 (n2 + n + 1).
P
8. Evaluate
n=0

9. On the planet Lemniscate, the people use the elliptic table of elements, a far more
advanced version of our periodic table. They’re not very good at calculus, though, so they’ve
asked for your help. They know that Kr is somewhat radioactive and deteriorates into Pl, a
very unstable element that deteriorates to form the stable element As. They started with a
block of Kr of size 10 and nothing else. (Their units don’t translate into English, sorry.) and
nothing else. At time t, they let x(t) be the amount of Kr, y(t) the amount of Pl, and z(t)
the amount of As. They know that x0 (t) = −x, and that, in the absence of Kr, y 0 (t) = −2y.
Your job is to find at what time t the quantity of Pl will be largest. You should assume that
the entire amount of Kr that deteriorates has turned into Pl.

+1
2u332 +u998 +4u1664 sin u691
R
10. Evaluate the definite integral 1+u666
du.
−1
Calculus Test Solutions
Harvard-MIT Math Tournament
March 3, 2001

1. A sequence of ants walk from (0, 0) to (1, 0) in the plane. The nth ant walks along
n semicircles of radius n1 with diameters lying along the line from (0, 0) to (1, 0). Let Ln be
the length of the path walked by the nth ant. Compute lim Ln .
n→∞
Solution: A semicircle of radius n1 has length 12 π n2 = πn , so n such semicircles have


total length π .

2. The polynomial 3x5 −250x3 +735x is interesting because it has the maximum possible
number of relative extrema and points of inflection at integer lattice points for a quintic
polynomial. What is the sum of the x-coordinates of these points?
Solution: The first derivative is 15x4 − 750x2 + 735, whose roots (which give the relative
extrema) sum to 750/15 = 50. The second derivative is 60x3 − 1500x, whose roots (which
give the points of inflection) sum to 1500/60 = 25, for a grand total of 75 .

3. A balloon that blows up in the shape of a perfect cube is being blown up at a rate such
that at time t fortnights, it has surface area 6t square furlongs. At how many cubic furlongs
per fortnight is the air being pumped in when the surface area is 144 square furlongs?
Solution: The surface area at time t is 6t, so the volume is t3/2 . Hence the air is being
√ √
pumped in at a rate of 32 t. When the surface area is 144, t = 24, so the answer is 3 6 .

4. What is the size of the largest rectangle that can be drawn inside of a 3-4-5 right
triangle with one of the rectangle’s sides along one of the legs of the triangle?
Solution: Clearly one vertex of the rectangle will be at the right angle. Position the
triangle with the leg of length 4 along the x-axis and the leg of length 3 along the y-axis.
Then the hypotenuse is along the line y = 3 − (3/4)x.
Suppose the rectangle has a side of length y along the leg of length 3. Then the area is
y(4/3)(3 − y) = 4y − (4/3)y 2 . The derivative of this is 0 when 4 − (8/3)y = 0, or y = 3/2,
giving an area of 3.
Or, if you prefer, suppose the rectangle has a side of length x along the leg of length 4. Then
the area is x(3 − (3/4)x) = 3x − (3/4)x2 . The derivative of this is 0 when 3 − (3/2)x = 0,
or x = 2, again giving an area of 3 .

5. Same as question 4, but now we want one of the rectangle’s sides to be along the
hypotenuse.
Solution: Put the hypotenuse along the x-axis, with the short leg starting at the origin
so that the right angle is at the point (9/5, 12/5). For notational convenience, let’s just scale
everything by a factor of 5 and then remember to divide the final area by 25, so now the top
point is at (9, 12).
Let (a, 0) be the point where the edge of the rectangle along the hypotenuse starts. Then
the height is h = (4/3)a since the leg of length 3 is along the line y = (4/3)x. The leg of
length 4 is along the line x = 25 − (4/3)y, so the horizontal edge of the rectangle ends at
b = 25 − (4/3)h = 25 − (16/9)a. The area of the rectangle is (b − a)h = (25 − (16/9)a −
a)(4/3)a = 1003
a − 10027
a2 . The derivative of this is 0 when 100
3
= 200
27
a, or a = 9/2. Thus the
(100/3)(9/2)−(100/27)(81/4)
maximum area is 25
=3.

6. The graph of x2 − (y − 1)2 = 1 has one tangent line with positive slope that passes
through (x, y) = (0, 0). If the point of tangency is (a, b), find sin−1 ( ab ) in radians.
dy
Solution: Differentiating both sides of the equation, we find that 2x − 2(y − 1) dx = 0,
dy x a b b a
and so dx = y−1 = b−1 . The line passing through (0, 0) and (a, b) has slope a , so a = b−1 .
2 2 2 2
Solving simultaneously with a − (b − 1) = 1, we get b − b − [(b − 1) + 1] = 0, and so
b = 2, a = (2). Finally, sin−1 ( ab ) = π4 .
p

7. Find the coefficient of x12 in the Maclaurin series (i.e. Taylor series around x = 0) for
1
1−3x+2x2
.
Solution: If you know formal power series, then this is not such a hard question, but
1 1/2 1
since this is a calculus test... Use partial fractions to get 1−3x+2x 2 = 1−2x − 1−x . Now each

of these can be expanded as a geometric series (or take derivatives and get the same result)
to get 12 (1 + 2x + 4x2 + 8x3 + · · · ) − (1 + x + x2 + x3 + · · · ), so the coefficient of xn is 2n−1 − 1.
When n = 12, that’s 2047 .


cot−1 (n2 + n + 1).
P
8. Evaluate
n=0
∞ ∞ ∞
1
cot(n2 + n + 1) =
P P P
Solution: arctan( n2 +n+1 ) = arctan(n + 1) − arctan(n)
n=0 n=0 n=0
by the sum/difference formula for tangent. This sum, taken out to n = N , telescopes to
− arctan(0) + arctan(N + 1). So as N goes to infinity, the sum goes to π/2 .

9. On the planet Lemniscate, the people use the elliptic table of elements, a far more
advanced version of our periodic table. They’re not very good at calculus, though, so they’ve
asked for your help. They know that Kr is somewhat radioactive and deteriorates into Pl, a
very unstable element that deteriorates to form the stable element As. They started with a
block of Kr of size 10 and nothing else. (Their units don’t translate into English, sorry.) and
nothing else. At time t, they let x(t) be the amount of Kr, y(t) the amount of Pl, and z(t)
the amount of As. They know that x0 (t) = −x, and that, in the absence of Kr, y 0 (t) = −2y.
Your job is to find at what time t the quantity of Pl will be largest. You should assume that
the entire amount of Kr that deteriorates has turned into Pl.
Solution: This problem is long-winded since it’s giving an autonomous linear system of
differential equations without using any such language (and it includes a number of subtle
references). The system we have is x0 = −x, y 0 = x − 2y. It’s not hard to see that x = 10e−t
satisfies the first equation and the initial condition. Plugging this into the second equation
and using the integrating factor e2t (or using eigenvalues and eigenvectors to solve the system
directly, though I don’t want to begin to explain what that means) lets us solve for y. More
precisely, we want to solve y 0 + 2y = 10e−t . Multiply by e2t and simplify the left hand side
to get (ye2t )0 = 10et . Integrating both sides with respect to t then yields ye2t = 10et + C, or
y = 10e−t + Ce−2t . Since y(0) = 0, we find C = −10. Now to maximize y, we solve y 0 (t) = 0,
or −10e−t + 20e−2t = 0, or t = ln 2 .

+1
2u332 +u998 +4u1664 sin u691
R
10. Evaluate the definite integral 1+u666
du.
−1
4u1664 sin u691
Solution: The term 1+u666
is odd in u, so its integral is 0. Now make the sub-
+1
R 2u332 +u998 +1
R 2+v2
1 −332/333 1
stitution u = v 1/333 ⇒ du = 333 v dv to find that 1+u666
du = 333 1+v 2
dv =
−1 −1
+1 R1 R1 1
 
1 1 2 1 2 2
(1 + tan−1 1) = 333 2
1 + π4 .
R   
333
1 + 1+v2 dv = 333 1 + 1+v2 dv = 333 1 + 1+v2 dv = 333
−1 0 0
Harvard-MIT Math Tournament
March 17, 2002
Individual Subject Test: Calculus


1. Two circles have centers that are d units apart, and each has diameter d. For any d,
let A(d) be the area of the smallest circle that contains both of these circles. Find lim A(d)
d2
.
d→∞

x2 −(x+h)2
2. Find lim h
.
h→0

3. We are given the values of the differentiable real functions f, g, h, as well as the derivatives
of their pairwise products, at x = 0:

f (0) = 1; g(0) = 2; h(0) = 3; (gh)0 (0) = 4; (hf )0 (0) = 5; (f g)0 (0) = 6.

Find the value of (f gh)0 (0).

4. Find the area of the region in the first quadrant x > 0, y > 0 bounded above the graph
of y = arcsin(x) and below the graph of the y = arccos(x).

5. What is the minimum vertical distance between the graphs of 2 + sin(x) and cos(x)?

6. Determine the positive value of a such that the parabola y = x2 + 1 bisects the area of
the rectangle with vertices (0, 0), (a, 0), (0, a2 + 1), and (a, a2 + 1).

7. Denote by hxi the fractional part of the real number x (for instance, h3.2i = 0.2). A
{1, 2, 3, . . . , M }, with each integer having
positive integer N is selected randomly from
the set
87
the same probability of being picked, and 303 N is calculated. This procedure is repeated
M times and the average value A(M ) is obtained. What is lim A(M )?
M →∞


( 2−1)/2
dx
R
8. Evaluate √
(2x+1) x2 +x
.
0

9. Suppose f is a differentiable real function such that f (x) + f 0 (x) ≤ 1 for all x, and
f (0) = 0. What is the largest possible value of f (1)? (Hint: consider the function ex f (x).)
R1
10. A continuous real function f satisfies the identity f (2x) = 3f (x) for all x. If 0 f (x) dx =
R2
1, what is 1 f (x) dx?

1
Harvard-MIT Math Tournament
March 17, 2002
Individual Subject Test: Calculus


1. Two circles have centers that are d units apart, and each has diameter d. For any d,
let A(d) be the area of the smallest circle that contains both of these circles. Find lim A(d)
d2
.
d→∞
 √ 2
d+ d
π 2 π
Solution: This equals lim d2
= 4
.
d→∞

x2 −(x+h)2
2. Find lim h
.
h→0
x2 −x2 −h2 −2hx
Solution: This equals lim h
= lim −h − 2x = −2x . Alternate Solution: This
h→0 h→0
2
is the definition of the derivative of −x with respect to x, which is −2x.

3. We are given the values of the differentiable real functions f, g, h, as well as the
derivatives of their pairwise products, at x = 0:

f (0) = 1; g(0) = 2; h(0) = 3; (gh)0 (0) = 4; (hf )0 (0) = 5; (f g)0 (0) = 6.

Find the value of (f gh)0 (0).


Solution: 16 By the product rule, (f gh)0 = f 0 gh + f g 0 h + f gh0 = ((f g)0 h + (gh)0 f +
(hf )0 g)/2. Evaluated at 0, this gives 16.

4. Find the area of the region in the first quadrant x > 0, y > 0 bounded above the
graph of y = arcsin(x) and below the graph of the y = arccos(x).
π/4
R
Solution: We can integrate over y rather than x. In particular, the solution is sin y dy+
0
π/2   √
√1
R
cos y dy = 1 − 2
2=2− 2.
π/4

5. What is the minimum vertical distance between the graphs of 2 + sin(x) and cos(x)?
Solution: The derivative of 2 + sin(x) − cos(x) is cosx + sinx, which in the interval
0 ≤ x < 2π is zero at x = 3π
4
, 7π
4
. At 7π
4
, when sin(x) is negative and cos(x) is positive, the

distance reaches its minimal value of 2 − 2 .

6. Determine the positive value of a such that the parabola y = x2 + 1 bisects the area
of the rectangle with vertices (0, 0), (a, 0), (0, a2 + 1), and (a, a2 + 1).

Solution: 3 The area of the rectangle is a3 + a. The portion under the parabola has
Ra 2
area 0 x + 1dx = a3 /3 + a. Thus we wish to solve the equation a3 + a = 2(a3 /3 + a);

dividing by a and rearranging gives a2 /3 = 1, so a = 3.

1
7. Denote by hxi the fractional part of the real number x (for instance, h3.2i = 0.2).

87set {1,
A positive integer N is selected randomly from the 2, 3, . . . , M }, with each integer
having the same probability of being picked, and 303 N is calculated. This procedure is
repeated M times and the average value A(M ) is obtained. What is lim A(M )?
M →∞
Solution: This method of picking N is equivalent

87 to uniformly randomly selecting a
positive integer. Call this the average value of 303 N for N a positive integer. In lowest
87 29 0 1
terms, 303 = 101 , so the answer is the same as the average value of 101 , 101 , . . . , 100
101
, which is
1+2+···+100 100·101/2 50
101·101
= 101·101
= 101
.

( 2−1)/2
dx
R
8. Evaluate √
(2x+1) x2 +x
.
0√
Solution: Let u = x2 + x. Then du = 2√2x+1 du
R
2 dx. So the integral becomes 2 (4x2 +4x+1) ,
−1
x +x √
or 2 4udu (2u), yielding a final answer of tan−1 (2 x2 + x) + C for the in-
R
2 +1 . This is tan

definite integral. The definite integral becomes tan−1 (1) − tan−1 (0) = π4 .

9. Suppose f is a differentiable real function such that f (x) + f 0 (x) ≤ 1 for all x, and
f (0) = 0. What is the largest possible value of f (1)? (Hint: consider the function ex f (x).)
Solution: 1 − 1/e Let g(x) = ex f (x); then g 0 (x) = ex (f (x) + f 0 (x)) ≤ ex . Integrating
R1 R1
from 0 to 1, we have g(1) − g(0) = 0 g 0 (x)dx ≤ 0 ex dx = e − 1. But g(1) − g(0) = e · f (1),
so we get f (1) ≤ (e − 1)/e. This maximum is attained if we actually have g 0 (x) = ex
everywhere; this entails the requirement f (x) + f 0 (x) = 1, which is met by f (x) = 1 − e−x .

R 1 10. A continuous real R 2 function f satisfies the identity f (2x) = 3f (x) for all x. If
0
f (x) dx = 1, what is 1 f (x) dx?
R2 R1
Solution: 5 Let S = 1 f (x) dx. By setting u = 2x, we see that 1/2 f (x) dx =
R1 R2 R 1/2
1/2
f (2x)/3 dx = 1 f (u)/6 du = S/6. Similarly, 1/4 f (x) dx = S/36, and in general
R 1/2n−1 R1
1/2n
f (x) dx = S/6n . Adding finitely many of these, we have 1/2n f (x) dx = S/6 + S/36 +
R1
· · · + S/6n = S · (1 − 1/6n )/5. Taking the limit as n → ∞, we have 0 f (x) dx = S/5. Thus
S = 5, the answer.

2
Harvard-MIT Mathematics Tournament
March 15, 2003

Individual Round: Calculus Subject Test

1. A point is chosen randomly with uniform distribution in the interior of a circle of radius
1. What is its expected distance from the center of the circle?

2. A particle moves along the x-axis in such a way that its velocity at position x is given
by the formula v(x) = 2 + sin x. What is its acceleration at x = π6 ?

3. What is the area of the region bounded by the curves y = x2003 and y = x1/2003 and
lying above the x-axis?

4. The sequence of real numbers x1 , x2 , x3 , . . . satisfies limn→∞ (x2n + x2n+1 ) = 315 and
limn→∞ (x2n + x2n−1 ) = 2003. Evaluate limn→∞ (x2n /x2n+1 ).

5. Find the minimum distance from the point (0, 5/2) to the graph of y = x4 /8.

6. For n an integer, evaluate


µ ¶
1 1 1
lim √ + √ + · · · + q .
n→∞ n2 − 02 n2 − 12 n2 − (n − 1)2

7. For what value of a > 1 is Z a2


1 x−1
log dx
a x 32
minimum?

8. A right circular cone with a height of 12 inches and a base radius of 3 inches is filled
with water and held with its vertex pointing downward. Water flows out through a
hole at the vertex at a rate in cubic inches per second numerically equal to the height
of the water in the cone. (For example, when the height of the water in the cone is 4
inches, water flows out at a rate of 4 cubic inches per second.) Determine how many
seconds it will take for all of the water to flow out of the cone.

9. Two differentiable real functions f (x) and g(x) satisfy

f 0 (x)
= ef (x)−g(x)
g 0 (x)

for all x, and f (0) = g(2003) = 1. Find the largest constant c such that f (2003) > c
for all such functions f, g.

10. Evaluate Z ∞
1 − x2
dx.
−∞ 1 + x4

1
Harvard-MIT Mathematics Tournament
March 15, 2003

Individual Round: Calculus Subject Test — Solutions


1. A point is chosen randomly with uniform distribution in the interior of a circle of radius
1. What is its expected distance from the center of the circle?
Solution: 2/3
The probability of the point falling between a distance r and r + dr from the center
is the ratio of the area of the corresponding annulus to the area of the whole circle:
π[(r+dr)2 −r2 ]
π
→ 2πrπ dr = 2r dr for small values of dr. Then the expected distance is
R1 2
0 r cot 2r dr = 3 .

2. A particle moves along the x-axis in such a way that its velocity at position x is given
by the formula v(x) = 2 + sin x. What is its acceleration at x = π6 ?

Solution: 5 3/4

Acceleration is given by a = dv
dt
= dv dx
·
dx dt
= dv
dx
· v = cos x · (2 + sin x) = 5 3/4.
3. What is the area of the region bounded by the curves y = x2003 and y = x1/2003 and
lying above the x-axis?
Solution: 1001/1002
The two curves intersect at (0, 0) and (1, 1), so the desired area is
Z 1³ ´ · ¸1
1/2003 2003 x2004/2003 x2004 1001
x −x dx = − = .
0 2004/2003 2004 0 1002
4. The sequence of real numbers x1 , x2 , x3 , . . . satisfies limn→∞ (x2n + x2n+1 ) = 315 and
limn→∞ (x2n + x2n−1 ) = 2003. Evaluate limn→∞ (x2n /x2n+1 ).
Solution: −1
We have limn→∞ (x2n+1 −x2n−1 ) = limn→∞ [(x2n +x2n+1 )−(x2n +x2n−1 )] = 315−2003 =
−1688; it follows that x2n+1 → −∞ as n → ∞. Then
x2n x2n + x2n+1
lim = lim − 1 = −1,
n→∞ x n→∞ x2n+1
2n+1

since x2n + x2n+1 → 315 while x2n+1 → −∞.


5. Find the minimum distance from the point (0, 5/2) to the graph of y = x4 /8.

Solution: 17/2
We want to minimize x2 +(x4 /8−5/2)2 = x8 /64−5x4 /8+x2 +25/4, which is equivalent
to minimizing z 4 /4 − 10z 2 + 16z, where we have set z = x2 . The derivative of this
expression is z 3 − 20z + 16, which √
is seen on inspection
√ to have 4 as a root, leading to
the factorization (z − 4)(z + 2 − 2 2)(z + 2 −√2 2). Since z = x2 ranges over [0, ∞),
the possible minima are√at z = 0, z = −2 + 2 2, and z = 4. However, the derivative
is positive on (0, −2 + 2 2), so this leaves only 0 and 4 to be tried. We find that the
minimum is in fact achieved
q at z = 4, so the closest point on the graph is given by

x = ±2, with distance 2 + (24 /8 − 5/2)2 = 17/2.
2

1
6. For n an integer, evaluate
µ ¶
1 1 1
lim √ + √ + · · · + q .
n→∞ n2 − 02 n2 − 12 n2 − (n − 1)2

Solution: π/2
Note that √ 1
n2 −i2
= 1
n
·√ 1
, so that the sum we wish to evaluate is just a Riemann
1−( ni )2
sum. Then,
µ ¶ Z 1
1 n−1
X 1 1 −1 1 π
lim q = √ dx = [sin x]0 = .
n→∞ n i=0 1 − ( i )2 0 1 − x2 2
n

7. For what value of a > 1 is Z a2


1 x−1
log dx
a x 32
minimum?
Solution: 3
R 2
Let f (a) = aa x1 log x−1
32
df
dx. Then we want da = 0; by the Fundamental Theorem of
Calculus and the chain rule, this implies that
à ! à !
1 a2 − 1 1 a−1 d Z a2 1 x−1 Z a
1 x−1
2a 2 log − log = log dx − log dx = 0,
a 32 a 32 da c x 32 c x 32
2 2
where c is any constant with 1 < c < a. Then 2 log a 32−1 = log a−1 32
, so that ( a 32−1 )2 =
a−1
32
. After canceling factors of (a−1)/32 (since a > 1), this simplifies to (a2 −1)(a+1) =
32 ⇒ a3 + a2 − a − 33 = 0, which in turn factors as (a − 3)(a2 + 4a + 11) = 0. The
quadratic factor has no real solutions, so this leaves only a = 3. However, we have that
a > 1, and we can check that f (1) = 0, lima→∞ f (a) > 0, and f (3) < 0, so the global
minimum does occur at a = 3.

8. A right circular cone with a height of 12 inches and a base radius of 3 inches is filled
with water and held with its vertex pointing downward. Water flows out through a
hole at the vertex at a rate in cubic inches per second numerically equal to the height
of the water in the cone. (For example, when the height of the water in the cone is 4
inches, water flows out at a rate of 4 cubic inches per second.) Determine how many
seconds it will take for all of the water to flow out of the cone.
Solution: 9π/2
When the water in the cone is h inches high, it forms a cone similar to the original, so
that its base has radius h/4 and its volume is hence πh3 /48. The given condition then
states that à !
d πh3 πh2 dh dh 32
= −h ⇒ · = −h ⇒ 2h · =− .
dt 48 16 dt dt π
Integrating with respect to t, we get that h2 = −32t/π + C; setting t = 0, h = 12, we
get C = 144. The cone empties when h = 0, so 0 = −32t/π + 144 ⇒ t = 9π/2.

2
9. Two differentiable real functions f (x) and g(x) satisfy

f 0 (x)
= ef (x)−g(x)
g 0 (x)

for all x, and f (0) = g(2003) = 1. Find the largest constant c such that f (2003) > c
for all such functions f, g.
Solution: 1 − ln 2
d
Rearranging the given equation gives f 0 (x)e−f (x) = g 0 (x)e−g(x) for all x, so dx (e−f (x) −
−g(x) 0 −f (x) 0 −g(x) −f (x) −g(x)
e ) = −f (x)e + g (x)e = 0. Thus, e −e is a constant, and it must
−f (0) −1 −f (2003) −g(2003)
be less than e = e . Thus, e <e +e = 2e−1 = eln 2−1 ⇒ f (2003) >
−1

1 − ln 2. On the other hand, we can find positive-valued functions e−f (x) , e−g(x) that
take on the required values at 0 and 2003 and have constant difference arbitrarily close
to e−1 . For example, for arbitrarily large t, we can set e−f (x) = e−(t(2003−x)+1) + e−1 −
e−(2003t+1) and e−g(x) = e−(t(2003−x)+1) , and we can check that the resulting functions
f, g satisfy the required conditions. Thus, we can make f (2003) arbitrarily close to
1 − ln 2, so this is the answer.

10. Evaluate Z ∞
1 − x2
dx.
−∞ 1 + x4

Solution: 0
R∞
Let S = 0 1/(x4 + 1) dx; note that the integral converges absolutely. Substituting
x = 1/u, so that dx = −1/u2 du, we have
Z ∞ Z 0 Z 0
1 1 du −u2
S= dx = = du
0 1 + x4 ∞ 1 + u−4 −u2 ∞ u4 + 1

Z ∞ Z ∞
u2 x2
= du = dx
0 1 + u4 0 1 + x4
(the
R∞
manipulations are justified by absolute convergence), from which we see that
2 4
0 (1 − x )/(1 + x ) dx = 0. Since the integrand is an even function, it follows that the
integral from −∞ to ∞ is zero as well.

3
Harvard-MIT Mathematics Tournament
February 28, 2004

Individual Round: Calculus Subject Test


f (x+h)−f (h)
1. Let f (x) = sin(sin x). Evaluate limh→0 x
at x = π.
2. Suppose the function f (x) − f (2x) has derivative 5 at x = 1 and derivative 7 at x = 2.
Find the derivative of f (x) − f (4x) at x = 1.
√ √
3. Find limx→∞ ( 3 x3 + x2 − 3 x3 − x2 ).
4. Let f (x) = cos(cos(cos(cos(cos(cos(cos(cos x))))))), and suppose that the number a
satisfies the equation a = cos a. Express f 0 (a) as a polynomial in a.
5. A mouse is sitting in a toy car on a negligibly small turntable. The car cannot turn on
its own, but the mouse can control when the car is launched and when the car stops
(the car has brakes). When the mouse chooses to launch, the car will immediately
leave the turntable on a straight trajectory at 1 meter per second.
Suddenly someone turns on the turntable; it spins at 30 rpm. Consider the set S of
points the mouse can reach in his car within 1 second after the turntable is set in
motion. (For example, the arrows in the figure below represent two possible paths the
mouse can take.) What is the area of S, in square meters?
.5 m

30 rpm

1m

6. For x > 0, let f (x) = xx . Find all values of x for which f (x) = f 0 (x).
7. Find the area of the region in the xy-plane satisfying x6 − x2 + y 2 ≤ 0.
8. If x and y are real numbers with (x + y)4 = x − y, what is the maximum possible value
of y?
9. Find the positive constant c0 such that the series

X n!
n
n=0 (cn)

converges for c > c0 and diverges for 0 < c < c0 .


10. Let P (x) = x3 − 23 x2 + x + 14 . Let P [1] (x) = P (x), and for n ≥ 1, let P [n+1] (x) =
R
P [n] (P (x)). Evaluate 01 P [2004] (x) dx.

1
Harvard-MIT Mathematics Tournament
February 28, 2004

Individual Round: Calculus Subject Test — Solutions


f (x+h)−f (h)
1. Let f (x) = sin(sin x). Evaluate limh→0 x
at x = π.
Solution: 0
The expression f (x+h)−fx
(h)
is continuous at h = 0, so the limit is just f (x)−f (0)
x
. Letting
sin(sin π)−sin(sin 0)
x = π yields π
= 0.

2. Suppose the function f (x) − f (2x) has derivative 5 at x = 1 and derivative 7 at x = 2.


Find the derivative of f (x) − f (4x) at x = 1.
Solution: 19
Let g(x) = f (x) − f (2x). Then we want the derivative of

f (x) − f (4x) = (f (x) − f (2x)) + (f (2x) − f (4x)) = g(x) + g(2x)

at x = 1. This is g 0 (x) + 2g 0 (2x) at x = 1, or 5 + 2 · 7 = 19.


√ √
3. Find limx→∞ ( 3 x3 + x2 − 3 x3 − x2 ).
Solution: 2/3
Observe that
h √
3
i x/3 + 1/27
lim (x + 1/3) − x3 + x2 = lim √ √ ,
x→∞ x→∞ ( 3 x3 + x2 )2 + ( 3 x3 + x2 )(x + 1/3) + (x + 1/3)2

by factoring the numerator as a difference of cubes. The numerator is linear in x, while


the denominator is at√ least 3x2 , so the limit as x → ∞ is 0. By similar arguments,
3
limx→∞ [(x − 1/3) − x3 − x2 ] = 0. So, the desired limit equals
√3
√3
2/3 + lim [(x − 1/3) − x3 − x2 ] − lim [(x + 1/3) − x3 + x2 ] = 2/3.
x→∞ x→∞

4. Let f (x) = cos(cos(cos(cos(cos(cos(cos(cos x))))))), and suppose that the number a


satisfies the equation a = cos a. Express f 0 (a) as a polynomial in a.
Solution: a8 − 4a6 + 6a4 − 4a2 + 1
This is an exercise using the chain rule. Define f0 (x) = x and fn (x) = cos fn−1 (x) for
n ≥ 0. We will show by induction that fn (a) = a and fn0 (a) = (− sin a)n for all n. The
case n = 0 is clear. Then fn (a) = cos fn−1 (a) = cos a = a, and
0
fn0 (a) = fn−1 (a) · (− sin fn−1 (a)) = (− sin a)n−1 · (− sin a) = (− sin a)n

by induction. Now, f (x) = f8 (x), so f 0 (a) = (− sin a)8 = sin8 a. But sin2 a = 1 −
cos2 a = 1 − a2 , so f 0 (a) = (1 − a2 )4 = a8 − 4a6 + 6a4 − 4a2 + 1.

1
5. A mouse is sitting in a toy car on a negligibly small turntable. The car cannot turn on
its own, but the mouse can control when the car is launched and when the car stops
(the car has brakes). When the mouse chooses to launch, the car will immediately
leave the turntable on a straight trajectory at 1 meter per second.
Suddenly someone turns on the turntable; it spins at 30 rpm. Consider the set S of
points the mouse can reach in his car within 1 second after the turntable is set in
motion. (For example, the arrows in the figure below represent two possible paths the
mouse can take.) What is the area of S, in square meters?
.5 m

30 rpm

1m

Solution: π/6
The mouse can wait while the table rotates through some angle θ and then spend the
remainder of the time moving along that ray at 1 m/s. He can reach any point between
the starting point and the furthest reachable point along the ray, (1 − θ/π) meters out.
So the area is given by the polar integral
Z π
(1 − θ/π)2 1 1 Zπ 2
dθ = · 2 φ dφ = π/6
0 2 2 π 0
(where we have used the change of variables φ = π − θ).

6. For x > 0, let f (x) = xx . Find all values of x for which f (x) = f 0 (x).
Solution: 1
Let g(x) = log f (x) = x log x. Then f 0 (x)/f (x) = g 0 (x) = 1 + log x. Therefore
f (x) = f 0 (x) when 1 + log x = 1, that is, when x = 1.

7. Find the area of the region in the xy-plane satisfying x6 − x2 + y 2 ≤ 0.


Solution: π/2

Rewrite the condition as |y| ≤ x2 − x6 . The right side is zero when x is −1, 0, or 1,
and it bounds an area symmetric about the x- and y-axes. Therefore, we can calculate
the area by the integral
Z 1 √ Z 1 √ Z 1√
2 x2 − x6 dx = 4 x 1− x4 dx = 2 1 − u2 du = π/2.
−1 0 0

8. If x and y are real numbers with (x + y)4 = x − y, what is the maximum possible value
of y?

Solution: 3 3 2/16

2
By drawing the graph of the curve (as shown), which is just a 135◦ clockwise rotation
and scaling of y = x4 , we see that the maximum is achieved at the unique point where
dy/dx = 0. Implicit differentiation gives 4(dy/dx
√ +√1)(x + y)3 = 1 − dy/dx, so setting

3 4
dy/dx = 0 gives 4(x + y) = 1. So x + y = √ 1/ 4 = √3 2/2, and x −
3
√ y = (x + y) = 3
2/8.
Subtracting and dividing by 2 gives y = ( 3 2/2 − 3 2/8)/2 = 3 3 2/16.

0.5 1 1.5

-0.5

-1

-1.5

9. Find the positive constant c0 such that the series



X n!
n
n=0 (cn)

converges for c > c0 and diverges for 0 < c < c0 .


Solution: 1/e
The ratio test tells us that the series converges if
µ ¶n
(n + 1)!/(c(n + 1))n+1 1 n
lim n
= · lim
n→∞ n!/(cn) c n→∞ n + 1
is less than one and diverges if it is greater than one. But
µ ¶n µ ¶−n
n 1 1
lim = n→∞
lim 1+ = .
n→∞ n+1 n e
Then the limit above is just 1/ce, so the series converges for c > 1/e and diverges for
0 < c < 1/e.
10. Let P (x) = x3 − 23 x2 + x + 14 . Let P [1] (x) = P (x), and for n ≥ 1, let P [n+1] (x) =
R
P [n] (P (x)). Evaluate 01 P [2004] (x) dx.
Solution: 1/2
By Note that P (1 − x) = 1 − P (x). It follows easily by induction that P [k] (1 − x) =
1 − P [k] (x) for all positive integers k. Hence
Z 1 Z 1
P [2004] (x) dx = 1 − P [2004] (1 − x) dx
0 0
Z 1
= 1− P [2004] (1 − x) dx
0
Z 1
= 1− P [2004] (u) du (u = 1 − x).
0

R1
Hence 0 P [2004] (x) dx = 1/2.

3
Harvard-MIT Mathematics Tournament
February 19, 2005

Individual Round: Calculus Subject Test

1. Let f (x) = x3 + ax + b, with a 6= b, and suppose the tangent lines to the graph of f at
x = a and x = b are parallel. Find f (1).
R ∞ cos u R
2. A plane curve is parameterized by x(t) = t u
du and y(t) = t∞ sinu u du for 1 ≤ t ≤
2. What is the length of the curve?
R1 R1
3. Let f : R → RR be a continuous function with 0 f (x)f 0 (x)dx = 0 and 0 f (x)2 f 0 (x)dx =
18. What is 01 f (x)4 f 0 (x)dx?

4. Let f : R → R be a smooth function such that f 0 (x)2 = f (x)f 00 (x) for all x. Suppose
f (0) = 1 and f (4) (0) = 9. Find all possible values of f 0 (0).

5. Calculate ³ ´
x
lim+ xx − xx .
x→0

6. The graph of r = 2 + cos 2θ and its reflection over the line y = x bound five regions in
the plane. Find the area of the region containing the origin.

7. Two ants, one starting at (−1, 1), the other at (1, 1), walk to the right along the
parabola y = x2 such that their midpoint moves along the line y = 1 with constant
speed 1. When the left ant first hits the line y = 12 , what is its speed?

8. If f is a continuous real function such that


R 2005
f (x − 1) + f (x + 1) ≥ x + f (x) for all x,
what is the minimum possible value of 1 f (x)dx?

9. Compute

X 4
.
k=0 (4k)!

10. Let f : R → R be a smooth function such that f 0 (x) = f (1 − x) for all x and f (0) = 1.
Find f (1).

1
Harvard-MIT Mathematics Tournament
February 19, 2005

Individual Round: Calculus Subject Test — Solutions

1. Let f (x) = x3 + ax + b, with a 6= b, and suppose the tangent lines to the graph of f at
x = a and x = b are parallel. Find f (1).
Solution: 1
Since f 0 (x) = 3x2 + a, we must have 3a2 + a = 3b2 + a. Then a2 = b2 , and since a 6= b,
a = −b. Thus f (1) = 1 + a + b = 1.
R ∞ cos u R
2. A plane curve is parameterized by x(t) = t u
du and y(t) = t∞ sinu u du for 1 ≤ t ≤
2. What is the length of the curve?
Solution: ln 2
dx dy
By the Second Fundamental Theorem of Calculus, dt
= − cost t and dt
= − sint t . There-
fore, the length of the curve is
v s
Z 2uà ! à !2 Z 2 Z 2
u dx 2 dy cos2 t sin2 t 1
t
+ dt = 2
+ 2 dt = dt = [ln t]21 = ln 2.
1 dt dt 1 t t 1 t

R1 R1
3. Let f : R → RR be a continuous function with 0 f (x)f 0 (x)dx = 0 and 0 f (x)2 f 0 (x)dx =
18. What is 01 f (x)4 f 0 (x)dx?
Solution: 486/5
Z 1 Z f (1)
0 1
0= f (x)f (x)dx = u du = (f (1)2 − f (0)2 ), and
0 f (0) 2
Z 1 Z f (1)
1
18 = f (x)2 f 0 (x)dx = u2 du = (f (1)3 − f (0)3 ).
0 f (0) 3
The first equation implies f (0) = ±f (1). The second equation shows that f (0) 6= f (1),
and in fact 54 = f (1)3 − f (0)3 = 2f (1)3 , so f (1) = 3 and f (0) = −3. Then
Z 1 Z f (1)
4 0 1 1 486
f (x) f (x)dx = u4 du = (f (1)5 − f (0)5 ) = (243 + 243) = .
0 f (0) 5 5 5

4. Let f : R → R be a smooth function such that f 0 (x)2 = f (x)f 00 (x) for all x. Suppose
f (0) = 1 and f (4) (0) = 9. Find all possible values of f 0 (0).

Solution: ± 3
Let f 0 (0) = a. Then the equation gives f 00 (0) = a2 . Differentiating the given equation
gives
2f 0 (x)f 00 (x) = f (x)f 000 (x) + f 0 (x)f 00 (x),
or f 0 (x)f 00 (x) = f (x)f 000 (x). Differentiating once more gives

f 0 (x)f 000 (x) + f 00 (x)2 = f (x)f (4) (x) + f 0 (x)f 000 (x)

1

or f 00 (x)2 = f (x)f (4) (x), giving

9 = f (4)
(0) = a 4
. Thus a = ± 3. These are indeed
±x 3
both attainable by f (x) = e .
00 0
Alternative Solution: Rewrite the given equation as ff 0 (x) (x)
= ff (x)
(x)
. Integrating both
0 0
sides gives ln f (x) = ln f (x) + C1 , and exponentiating gives f (x) = Cf (x). This has
solution f (x) = AeCx for constants A and C. Since f (0)√= 1, A = 1, and differentiating
we find that C 4 = f (4) (0) = 9, yielding f 0 (0) = C = ± 3.
5. Calculate ³ ´
x
lim+ xx − xx .
x→0

Solution: −1
We first calculate limx→0+ xx : it is just exp(limx→0+ x ln x). But
ln x 1/x
lim+ x ln x = lim+ = lim+ = lim −x = 0
x→0 x→0 1/x x→0 −1/x2 x→0+
x
by L’Hôpital’s
³ x Rule.
´ Therefore limx→0+ xx = 1. Then limx→0+ xx = 01 = 0, so
limx→0+ xx − xx = −1.
6. The graph of r = 2 + cos 2θ and its reflection over the line y = x bound five regions in
the plane. Find the area of the region containing the origin.
Solution: 9π/2 − 8

The original graph is closer to the origin than its reflection for θ ∈ ( π4 , 3π
4
) ∪ ( 5π
4
, 7π
4
),
and the region is symmetric about the origin. Therefore the area we wish to find is
the polar integral
Z 3π Z 3π
4 1 4
4 (2 + cos 2θ)2 dθ = 2 π (4 + 4 cos 2θ + cos2 2θ)dθ
π
4
2 4
Z 3π µ ¶
4 1
= 2 π 4 + 4 cos 2θ + (1 + cos 4θ) dθ
4
2
· ¸ 3π
1 4
= 9θ + 4 sin 2θ + sin 4θ
4 π
4
µ ¶ µ ¶
27π 9π 9π
= −4 − +4 = − 8.
4 4 2

2
7. Two ants, one starting at (−1, 1), the other at (1, 1), walk to the right along the
parabola y = x2 such that their midpoint moves along the line y = 1 with constant
speed 1. When the left ant first hits the line y = 12 , what is its speed?

Solution: 3 3 − 3
When the left ant first√ hits the line y = 12 , the right ant

hits the line y = 32 . The
left ant is then at (− 22 , 12 ), and the right ant is at ( 26 , 32 ). Let the left ant have
velocity with components vx√and vy , the right√ ant velocity with components wx and
dy vy wy
wy . Since dx = 2x, vx = − 2 and wx = 6. Since the midpoint of the ants moves
at speed 1 along the line y = 1, 12 (vx + wx ) = 1 and 12 (vy + w√y ) = 0. Therefore,
√ √ √ √
2vx = −vy = wy = 6wx = 6(2 − vx ). Solving for vx gives √26+√ 6
2
= 3 − 3. Then
the speed of the left ant is
q q √ √ √
vx2 + vy2 = vx2 + (− 2vx )2 = 3|vx | = 3 3 − 3.

8. If f is a continuous real function such that


R 2005
f (x − 1) + f (x + 1) ≥ x + f (x) for all x,
what is the minimum possible value of 1 f (x)dx?
Solution: 2010012
Let g(x) = f (x) − x. Then

g(x − 1) + x − 1 + g(x + 1) + x + 1 ≥ x + g(x) + x,

or g(x − 1) + g(x + 1) ≥ g(x). But now,

g(x + 3) ≥ g(x + 2) − g(x + 1) ≥ −g(x).

Therefore
Z a+6 Z a+3 Z a+6
g(x)dx = g(x)dx + g(x)dx
a a a+3
Z a+3
= (g(x) + g(x + 3))dx ≥ 0.
a

It follows that
Z 2005 333 Z 6n+7
X
g(x) = g(x)dx ≥ 0,
1 n=0 6n+1

so that
Z 2005 Z 2005 Z 2005 " #2005
x2 20052 − 1
f (x)dx = (g(x) + x)dx ≥ x dx = = = 2010012.
1 1 1 2 1
2

Equality holds for f (x) = x.

9. Compute

X 4
.
k=0 (4k)!

Solution: e + 1/e + 2 cos 1

3
This is the power series
4x4 4x8
4+ + + ···
4! 8!
evaluated at x = 1. But this power series can be written as the sum
à !
x x2 x3 x4 x5 x6 x7
1+ + + + + + + + ···
1! 2! 3! 4! 5! 6! 7!
à !
x x2 x3 x4 x5 x6 x7
+ 1− + − + − + − + ···
1! 2! 3! 4! 5! 6! 7!
à !
x2 x4 x6
+ 2 1− + − + ···
2! 4! 6!
= ex + e−x + 2 cos x.

It follows that the quantity is e + 1/e + 2 cos 1.

10. Let f : R → R be a smooth function such that f 0 (x) = f (1 − x) for all x and f (0) = 1.
Find f (1).
Solution: sec 1 + tan 1
Differentiating the given equation gives f 00 (x) = −f (x). This has solutions of the form
A cos(x) + B sin(x). Since f (0) = 1, A = 1. Then f 0 (x) = B cos(x) − sin(x) and

f (1 − x) = cos(1 − x) + B sin(1 − x)
= cos 1 cos x + sin 1 sin x + B sin 1 cos x − B cos 1 sin x
= (cos 1 + B sin 1) cos x + (sin 1 − B cos 1) sin x.

Therefore, B = cos 1+B sin 1 and −1 = sin 1−B cos 1, both of which yield as solutions
cos 1 1 + sin 1
B= = = sec 1 + tan 1.
1 − sin 1 cos 1

4
IXth Annual Harvard-MIT Mathematics Tournament
Saturday 25 February 2006

Individual Round: Calculus Test

1. A nonzero polynomial f (x) with real coefficients has the property that f (x) = f 0 (x)f 00 (x).
What is the leading coefficient of f (x)?
ex cos x − 1 − x
2. Compute lim .
x→0 sin(x2 )
3. At time 0, an ant is at (1, 0) and a spider is at (−1, 0). The ant starts walking
counterclockwise along the unit circle, and the spider starts creeping to the right along
the x-axis. It so happens that the ant’s horizontal speed is always half the spider’s.
What will the shortest distance ever between the ant and the spider be?

X k4
4. Compute .
k=1
k!
Z 1
dx
5. Compute √ √ .
0 x+ 3x
6. A triangle with vertices at (1003, 0), (1004, 3), and (1005, 1) in the xy-plane is revolved
all the way around the y-axis. Find the volume of the solid thus obtained.

7. Find all positive real numbers c such that the graph of f : R → R given by f (x) =
x3 − cx has the property that the circle of curvature at any local extremum is centered
at a point on the x-axis.
Z π/3
8. Compute x tan2 (x)dx.
0

9. Compute the sum of all real numbers x such that

2x6 − 3x5 + 3x4 + x3 − 3x2 + 3x − 1 = 0.

10. Suppose f and g are differentiable functions such that

xg(f (x))f 0 (g(x))g 0 (x) = f (g(x))g 0 (f (x))f 0 (x)

for all real x. Moreover, f is nonnegative and g is positive. Furthermore,


Z a
e−2a
f (g(x))dx = 1 −
0 2

for all reals a. Given that g(f (0)) = 1, compute the value of g(f (4)).
IXth Annual Harvard-MIT Mathematics Tournament
Saturday 25 February 2006

Calculus Test: Solutions

1. A nonzero polynomial f (x) with real coefficients has the property that f (x) = f 0 (x)f 00 (x).
What is the leading coefficient of f (x)?
1
Answer: 18
Solution: Suppose that the leading term of f (x) is cxn , where c 6= 0. Then the
leading terms of f 0 (x) and of f 00 (x) are cnxn−1 and cn(n − 1)xn−2 , respectively, so
cxn = cnxn−1 · cn(n − 1)xn−2 , which implies that n = (n − 1) + (n − 2), or n = 3, and
1
c = cn · cn(n − 1) = 18c2 , or c = 18 .
ex cos x − 1 − x
2. Compute lim .
x→0 sin(x2 )
Answer: 21
Solution: Let’s compute all the relevant Maclaurin series expansions, up to the
quadratic terms:
1
x cos x = x + . . . , ex cos x = 1 + x + x2 + . . . , sin(x2 ) = x2 + . . . ,
2
so
1 2
ex cos x − 1 − x 2
x + ... 1
lim 2
= lim 2
= .
x→0 sin(x ) x→0 x + . . . 2
3. At time 0, an ant is at (1, 0) and a spider is at (−1, 0). The ant starts walking
counterclockwise along the unit circle, and the spider starts creeping to the right along
the x-axis. It so happens that the ant’s horizontal speed is always half the spider’s.
What will the shortest distance ever between the ant and the spider be?

14
Answer: 4
Solution: Picture an instant in time where the ant and spider have x-coordinates a
and s, respectively. If 1 ≤ s ≤ 3, then a ≤ 0, and the distance between the bugs is at
least 1. If s > 3, then, needless to say the distance between the bugs is at least 2. If
−1 ≤ s ≤ 1, then s = 1 − 2a, and the distance between the bugs is
r
p √ (8a − 3)2 + 7
(a − (1 − 2a))2 + (1 − a2 ) = 8a2 − 6a + 2 = ,
8
p
which attains the minimum value of 7/8 when a = 3/8.

X k4
4. Compute .
k=1
k!
Answer: 15e
Solution: Define, for non-negative integers n,

X kn
Sn := ,
k=0
k!

1
where 00 = 1 when it occurs. Then S0 = e, and, for n ≥ 1,
∞ ∞ ∞ ∞ n−1 
kn kn (k + 1)n (k + 1)n−1

X X X X X n−1
Sn = = = = = Si ,
k=0
k! k=1
k! k=0
(k + 1)! k=0
k! i=0
i

so we can compute inductively that S1 = e, S2 = 2e, S3 = 5e, and S4 = 15e.

5. Compute Z 1
dx
√ √
0 x+ 3x

Answer: 5 − 6 ln 2
Solution: Writing x = u6 so that dx = 6u5 du, we have
Z 1 Z 1
dx 6u5 du
√ √ = 3 2
0 x+ 3x 0 u +u
Z 1 3
u du
= 6
u+1
Z0 1  
2 1
= 6 u −u+1− du
0 u+1
1 !
u 3 u2
= 6 − + u − ln |u + 1| = 5 − 6 ln(2)
3 2 0

6. A triangle with vertices at (1003, 0), (1004, 3), and (1005, 1) in the xy-plane is revolved
all the way around the y-axis. Find the volume of the solid thus obtained.
Answer: 5020π
Solution: Let T ⊂ R2 denote the triangle, including its interior. Then T ’s area is
5/2, and its centroid is (1004, 4/3), so
Z
5
x dx dy = · 1004 = 2510.
(x,y)∈T 2

We are interested in the volume


Z
2πx dx dy,
(x,y)∈T

but this is just 2π · 2510 = 5020π.

7. Find all positive real numbers c such that the graph of f : R → R given by f (x) =
x3 − cx has the property that the circle of curvature at any local extremum is centered
at a point on the x-axis.

Answer: 23
0 2
p
Solution:
p The equation 0 =
p f (x) = 3x − c has two real roots: ± c/3. Let
00
a := c/3. As f (−a) = −6 c/3 < 0, f has a unique local maximum at x = −a.

2
Because f has half-turn symmetry about the origin, it suffices to consider this local
extremum. The radius of curvature at any local extremum is
1 1
r(x) = = ,
|f 00 (x)| 6|x|
so the condition in the problem is equivalent to

r(−a) = f (−a)
1
= −a(a2 − c)
6a
1 = 6a2 (c − a2 ) = 2c(2c/3)

c = 3/2.

8. Compute
Z π/3
x tan2 (x)dx
0
√ 2
Answer: π 3 3 − π18 − ln 2
Solution: We have
Z π/3 Z π/3  
2 1
x tan (x)dx = x −1 + dx
0 0 cos2 (x)
π/3 Z π/3
x2 xdx
=− +
2 0 0 cos2 (x)
π/3 π/3 Z π/3 !
x2 dx
= − + x tan(x) − tan(x)dx (u = x; dv = )
2 0 0 0 cos2 (x)
π/3 √
x2 π2 π 3
= − + x tan(x) + ln |cos(x)| = − +
− ln(2)
2 0 18 3

9. Compute the sum of all real numbers x such that

2x6 − 3x5 + 3x4 + x3 − 3x2 + 3x − 1 = 0

Answer: − 12
Solution: The carefully worded problem statement suggests that repeated roots
might be involved (not to be double counted), as well as complex roots (not to be
counted). Let P (x) = 2x6 − 3x5 + 3x4 + x3 − 3x2 + 3x − 1. Now, a is a double root of
the polynomial P (x) if and only if P (a) = P 0 (a) = 0. Hence, we consider the system

P (a) = 2a6 − 3a5 + 3a3 + a3 − 3a2 + 3a − 1 = 0


P 0 (a) = 12a5 − 15a4 + 12a3 + 3a2 − 6a + 3 = 0
=⇒ 3a4 + 8a3 − 15a2 + 18a − 7 = 0
37a3 − 57a2 + 57a − 20 = 0
a2 − a + 1 = 0

3
We have used polynomial long division to deduce that any double root must be a root
of a2 − a + 1! With this information, we can see that P (x) = (x2 − x + 1)2 (2x2 + x − 1).
The real roots are easily computed via the quadratic formula, leading to an answer of
− 12 . In fact the repeated roots were complex.

10. Suppose f and g are differentiable functions such that

xg(f (x))f 0 (g(x))g 0 (x) = f (g(x))g 0 (f (x))f 0 (x)

for all real x. Moreover, f is nonnegative and g is positive. Furthermore,


Z a
e−2a
f (g(x))dx = 1 −
0 2

for all reals a. Given that g(f (0)) = 1, compute the value of g(f (4)).
Answer: e−16 or e116
Solution: Differentiating the given integral with respect to a gives f (g(a)) = e−2a .
Now
d [ln (f (g(x)))] f 0 (g(x))g 0 (x) g 0 (f (x))f 0 (x) d [ln (g(f (x)))]
x =x = =
dx f (g(x)) g(f (x)) dx

where the second equals sign follows from the given. Since ln (f (g(x))) = −2x, we have
2
−x2 +C = ln (g(f (x))), so g(f (x)) = Ke−x . It follows that K = 1 and g(f (4)) = e−16 .

4
10th Annual Harvard-MIT Mathematics Tournament
Saturday 24 February 2007

Individual Round: Calculus Test

1. [3] Compute:
x2
lim
x→0 1 − cos(x)

2. [3] Determine the real number a having the property that f (a) = a is a relative minimum of f (x) =
x4 − x3 − x2 + ax + 1.

3. [4] Let a be a positive real number. Find the value of a such that the definite integral
Z a2
dx

a x+ x

achieves its smallest possible value.


4. [4] Find the real number α such that the curve f (x) = ex is tangent to the curve g(x) = αx2 .
5. [5] The function f : R → R satisfies f (x2 )f 00 (x) = f 0 (x)f 0 (x2 ) for all real x. Given that f (1) = 1 and
f 000 (1) = 8, determine f 0 (1) + f 00 (1).
6. [5] The elliptic curve y 2 = x3 +1 is tangent to a circle centered at (4, 0) at the point (x0 , y0 ). Determine
the sum of all possible values of x0 .
7. [5] Compute

X 1
.
n=1
n · (n + 1) · (n + 1)!

 2006
X 1
8. [6] Suppose that ω is a primitive 2007th root of unity. Find 22007 − 1 .
j=1
2 − ωj
k
For this problem only, you may express your answer in the form m · n + p, where m, n, k, and p are
positive integers. Note that a number z is a primitive nth root of unity if z n = 1 and n is the smallest
number amongst k = 1, 2, . . . , n such that z k = 1.
9. [7] g is a twice differentiable function over the positive reals such that

g(x) + 2x3 g 0 (x) + x4 g 00 (x) = 0 for all positive reals x. (1)


lim xg(x) = 1 (2)
x→∞

Find the real number α > 1 such that g(α) = 1/2.


10. [8] Compute

e−x sin(x)
Z
dx
0 x

1
10th Annual Harvard-MIT Mathematics Tournament
Saturday 24 February 2007

Individual Round: Calculus Test

1. [3] Compute:
x2
lim
x→0 1 − cos(x)

Answer: 2. Since sin2 (x) = 1 − cos2 (x), we multiply the numerator and denominator by 1 + cos(x)
and use the fact that x/ sin(x) → 1, obtaining
2
x2 x2 (1 + cos(x))

x
lim = lim = lim ·2=2
x→0 1 − cos(x) x→0 1 − cos2 (x) x→0 sin(x)
x2 2x
Remarks. Another solution, using L’Hôpital’s rule, is possible: limx→0 1−cos(x) = limx→0 sin(x) = 2.

2. [3] Determine the real number a having the property that f (a) = a is a relative minimum of f (x) =
x4 − x3 − x2 + ax + 1.
Answer: 1. Being a relative minimum, we have 0 = f 0 (a) = 4a3 − 3a2 − 2a + a = a(4a + 1)(a − 1).
Then a = 0, 1, −1/4 are the only possibilities. However, it is easily seen that a = 1 is the only value
satisfying f (a) = a.
3. [4] Let a be a positive real number. Find the value of a such that the definite integral
Z a2
dx

a x+ x

achieves its smallest possible value.



Answer: 3 − 2 2. Let F (a) denote the given definite integral. Then
Z a2
d dx 1 1
F 0 (a) = √ = 2a · √ − √ .
ada x+ x a2 + a2 a+ a
√ √ √ √
Setting F√0 (a) = 0, we√find that 2a + 2√ a = a + 1 or ( a + 1)2 = 2. We find a = ± 2 − 1, and
because a > 0, a = ( 2 − 1)2 = 3 − 2 2.
4. [4] Find the real number α such that the curve f (x) = ex is tangent to the curve g(x) = αx2 .
Answer: e2 /4. Suppose tangency occurs at x = x0 . Then ex0 = αx20 and f 0 (x0 ) = 2αx0 . On the
other hand, f 0 (x) = f (x), so αx20 = 2αx0 . Clearly, α = 0 and x0 = 0 are impossible, so it must be that
x0 = 2. Then α = ex0 /(x20 ) = e2 /4.
5. [5] The function f : R → R satisfies f (x2 )f 00 (x) = f 0 (x)f 0 (x2 ) for all real x. Given that f (1) = 1 and
f 000 (1) = 8, determine f 0 (1) + f 00 (1).
Answer: 6. Let f 0 (1) = a and f 00 (1) = b. Then setting x = 1 in the given equation, b = a2 .
Differentiating the given yields

2xf 0 (x2 )f 00 (x) + f (x2 )f 000 (x) = f 00 (x)f 0 (x2 ) + 2xf 0 (x)f 00 (x2 ).

Plugging x = 1 into this equation gives 2ab + 8 = ab + 2ab, or ab = 8. Then because a and b are real,
we obtain the solution (a, b) = (2, 4).
Remarks. A priori, the function needn’t exist, but one possibility is f (x) = e2x−2 .

1
6. [5] The elliptic curve y 2 = x3 +1 is tangent to a circle centered at (4, 0) at the point (x0 , y0 ). Determine
the sum of all possible values of x0 .
1
Answer: 3. Note that y 2 ≥ 0, so x3 ≥ −1 and x ≥ −1. Let the circle be defined by (x − 4)2 + y 2 = c
dy
for some c ≥ 0. Now differentiate the equations with respect to x, obtaining 2y dx = 3x2 from the
dy dy
given and 2y dx = −2x + 8 from the circle. For tangency, the two expressions dx must be equal if they
are well-defined, and this is almost always the case. Thus, −2x0 + 8 = 3x20 so x0 = −2 or x0 = 4/3,
but only the latter corresponds to a point on y 2 = x3 + 1. Otherwise, y0 = 0, and this gives the trivial
solution x0 = −1.
7. [5] Compute

X 1
.
n=1
n · (n + 1) · (n + 1)!

Answer: 3 − e. We write
∞ ∞   ∞ ∞
X 1 X 1 1 1 X 1 X 1
= − = −
n=1
n · (n + 1) · (n + 1)! n=1
n n + 1 (n + 1)! n=1
n · (n + 1)! n=1
(n + 1) · (n + 1)!
∞ ∞ ∞
1 X 1 X 1 1 X 1 1
+ − = + −
2 n=2 n · (n + 1)! n=1 (n + 1) · (n + 1)! 2 n=1 (n + 1) · (n + 2)! (n + 1) · (n + 1)!
∞    
1 X 1 − (n + 2) 1 1 1 1 1 1
+ = − + + ··· = 3 − + + + · · · = 3 − e.
2 n=1 (n + 1) · (n + 2)! 2 3! 4! 0! 1! 2!

Alternatively, but with considerably less motivation, we can induce telescoping by adding and sub-
tracting e − 2 = 1/2! + 1/3! + · · · , obtaining
∞ ∞
X n(n + 1) + 1 X (n + 1)2 − n
2−e+ =2−e+
n=1
n · (n + 1) · (n + 1)! n=1
n · (n + 1) · (n + 1)!

X 1 1
2−e+ − = 3 − e.
n=1
n · n! (n + 1) · (n + 1)!

 2006
X 1
8. [6] Suppose that ω is a primitive 2007th root of unity. Find 22007 − 1 .
j=1
2 − ωj
k
For this problem only, you may express your answer in the form m · n + p, where m, n, k, and p are
positive integers. Note that a number z is a primitive nth root of unity if z n = 1 and n is the smallest
number amongst k = 1, 2, . . . , n such that z k = 1.
Answer: 2005 · 22006 + 1. Note that
P2006 Q i
1 1 j=1 i6=j (z − ω )
+ ··· + =
z−ω z − ω 2006 (z − ω) · · · (z − ω 2006 )
d
 
z 2006 + z 2005 + · · · + 1 2006z 2005 + 2005z 2004 + · · · + 1 z − 1
= dz 2006 2005
= ·
z +z + ··· + 1 z 2006 + z 2005 + · · · + 1 z−1
2006z 2006 − z 2005 − z 2004 − · · · − 1 z − 1 2006z 2007 − 2007z 2006 + 1
= · = .
z 2007 − 1 z−1 (z 2007 − 1)(z − 1)
2005·22006 +1
Plugging in z = 2 gives 22007 −1 ; whence the answer.
9. [7] g is a twice differentiable function over the positive reals such that
g(x) + 2x3 g 0 (x) + x4 g 00 (x) = 0 for all positive reals x. (1)
lim xg(x) = 1 (2)
x→∞

2
Find the real number α > 1 such that g(α) = 1/2.
Answer: 6
π. In the first equation, we can convert the expression 2x3 g 0 (x)+x4 g 00 (x) into the derivative
of a product, and in fact a second derivative, by writing y = 1/x. Specifically,
     
3 0 4 00 1 −3 0 1 −4 00 1
0 = g(x) + 2x g (x) + x g (x) = g + 2y g +y g
y y y
    
1 d −2 0 1
= g + −y g
y dy y
d2
    
1 1
= g + 2 g
y dy y
 
Thus g y1 = c1 cos(y) + c2 sin(y) or g(x) = c1 cos(1/x) + c2 sin(1/x). Now the second condition gives

sin(1/x)
1 = lim c1 x + c2 · = c2 + lim c1 x
x→∞ 1/x x→∞

It must be that c1 = 0, c2 = 1. Now since 0 < 1/α < 1, the value of α such that g(α) = sin(1/α) = 1/2
is given by 1/α = π/6 and so α = 6/π.
10. [8] Compute

e−x sin(x)
Z
dx
0 x

Answer: π4 . We can compute the integral by introducing a parameter and exchanging the order of
integration:
Z ∞   Z ∞ Z 1  Z 1 Z ∞ 
sin(x)
e−x dx = e−x cos(ax)da dx = e−x cos(ax)dx da
0 x 0 0 0 0
Z 1 Z ∞ Z 1 " ∞ #
e(−1+ai)x

(−1+ai)x
= Re e dx da = Re da
−1 + ai

0 0 0
x=0
Z 1   Z 1  
1 1 + ai
= Re da = Re da
0 1 − ai 0 1 + a2
Z 1 1
1 −1
π
= da = tan (a) =

1 + a2
4
0
a=0

3
11th Annual Harvard-MIT Mathematics Tournament
Saturday 23 February 2008

Individual Round: Calculus Test

1. [3] Let f (x) = 1 + x + x2 + · · · + x100 . Find f 0 (1).

2. [3] Let ` be the line through (0, 0) and tangent to the curve y = x3 + x + 16. Find the slope of `.
Z y
1
3. [4] Find all y > 1 satisfying x ln x dx = .
1 4
2
(ln(2 − x))
4. [4] Let a, b be constants such that lim = 1. Determine the pair (a, b).
x→1 x2 + ax + b

5. [4] Let f (x) = sin6 x4 + cos6 x4 for all real numbers x. Determine f (2008) (0) (i.e., f differentiated
 

2008 times and then evaluated at x = 0).


n  −1
X n
6. [5] Determine the value of lim .
n→∞ k
k=0
√ √ √ 
7. [5] Find p so that limx→∞ xp x + 1 + 3 x − 1 − 2 3 x is some non-zero real number.
3

Z ln 2
2e3x + e2x − 1
8. [7] Let T = 3x 2x − ex + 1
dx. Evaluate eT .
0 e + e
1
9. [7] Evaluate the limit lim n− 2 (1+ n ) 11 · 22 · · · · · nn
1 1 
n2
.
n→∞
Z 1
10. [8] Evaluate the integral ln x ln(1 − x) dx.
0

1
11th Annual Harvard-MIT Mathematics Tournament
Saturday 23 February 2008

Individual Round: Calculus Test

1. [3] Let f (x) = 1 + x + x2 + · · · + x100 . Find f 0 (1).


Answer: 5050 Note that f 0 (x) = 1 + 2x + 3x2 + · · · + 100x99 , so f 0 (1) = 1 + 2 + · · · + 100 = 100·101
2 =
5050.
2. [3] Let ` be the line through (0, 0) and tangent to the curve y = x3 + x + 16. Find the slope of `.
Answer: 13 Let the point of tangency be (t, t3 + t + 16), then the slope of ` is (t3 + t + 16)/t. On
the other hand, since dy/dx = 3x2 + 1, the slope of ` is 3t2 + 1. Therefore,

t3 + t + 16
= 3t2 + 1.
t
Simplifying, we get t3 = 8, so t = 2. It follows that the slope is 3(2)2 + 1 = 13.
Z y
1
3. [4] Find all y > 1 satisfying x ln x dx = .
1 4

Answer: e Applying integration by parts with u = ln x and v = 21 x2 , we get
Z y y 1 Z y
1 2 1 1 1
x ln x dx = x ln x − x dx = y 2 ln y − y 2 + .

1 2 1 2 1 2 4 4

So y 2 ln y = 21 y 2 . Since y > 1, we obtain ln y = 21 , and thus y = e.
2
(ln(2 − x))
4. [4] Let a, b be constants such that lim = 1. Determine the pair (a, b).
x→1 x2 + ax + b

Answer: (−2, 1) When x = 1, the numerator is 0, so the denominator must be zero as well, so
1 + a + b = 0. Using l’Hôpital’s rule, we must have
2
(ln(2 − x)) 2 ln(2 − x)
1 = lim = lim ,
x→1 x2 + ax + b x→1 (x − 2)(2x + a)

and by the same argument we find that 2 + a = 0. Thus, a = −2 and b = 1. This is indeed a solution,
as can be seen by finishing the computation.

5. [4] Let f (x) = sin6 x4 + cos6 x4 for all real numbers x. Determine f (2008) (0) (i.e., f differentiated
 

2008 times and then evaluated at x = 0).


3
Answer: 8 We have

sin6 x + cos6 x = (sin2 x + cos2 x)3 − 3 sin2 x cos2 x(sin2 x + cos2 x)


 
3 3 1 − cos 4x
= 1 − 3 sin2 x cos2 x = 1 − sin2 2x = 1 −
4 4 2
5 3
= + cos 4x.
8 8
5 3 3
It follows that f (x) = cos x. Thus f (2008) (x) =
8 + 8 8 cos x. Evaluating at x = 0 gives 83 .
n  −1
X n
6. [5] Determine the value of lim .
n→∞ k
k=0

1
n −1 n −1
 
Answer: 2 Let Sn denote the sum in the limit. For n ≥ 1, we have Sn ≥ 0 + n = 2. On
the other hand, for n ≥ 3, we have
 −1  −1  −1  −1 n−2
X n−1  −1
n n n n 2 n
Sn = + + + + ≤ 2 + + (n − 3)
0 1 n−1 n k n 2
k=2

which goes to 2 as n → ∞. Therefore, Sn → 2.


√ √ √ 
7. [5] Find p so that limx→∞ xp 3 x + 1 + 3 x − 1 − 2 3 x is some non-zero real number.
5
Answer: 3 Make the substitution t = x1 . Then the limit equals to
r r r !
−p 1 1 3 1 1 √ √
= lim t−p− 3 3 1 + t + 3 1 − t − 2 .
3 3

lim t +1+ −1−2
t→0 t t t t→0

√ √
We need the degree of the first nonzero term in the MacLaurin expansion of 3 1 + t + 3 1 − t − 2. We
have
√ 1 1 √ 1 1
3
1 + t = 1 + t − t2 + o(t2 ), 3
1 − t = 1 − t − t2 + o(t2 ).
3 9 3 9

3

3 2 2 2
It follows that 1 + t + 1 − t − 2 = − 9 t + o(t ). By consider the degree of the leading term, it
follows that −p − 13 = −2. So p = 35 .
Z ln 2
2e3x + e2x − 1
8. [7] Let T = dx. Evaluate eT .
0 e3x + e2x − ex + 1
11
Answer: 4 Divide the top and bottom by ex to obtain that

ln 2
2e2x + ex − e−x
Z
T = dx
0 e2x+ ex − 1 + e−x

Notice that 2e2x + ex − e−x is the derivative of e2x + ex − 1 + e−x , and so


iln 2    
h 1 11
T = ln |e2x + ex − 1 + e−x | = ln 4 + 2 − 1 + − ln 2 = ln
0 2 4
11
Therefore, eT = 4 .
1
9. [7] Evaluate the limit lim n− 2 (1+ n ) 11 · 22 · · · · · nn
1 1 
n2
.
n→∞

Answer: e−1/4 Taking the logarithm of the expression inside the limit, we find that it is
  n n  
1 1 1 X 1Xk k
− 1+ ln n + 2 k ln k = ln .
2 n n n n n
k=1 k=1

R1
We can recognize this as the as Riemann sum expansion for the integral 0 x ln x dx, and thus the
limit of the above sum as n → ∞ equals to the value of this integral. Evaluating this integral using
integration by parts, we find that
Z 1 1 Z 1 x
1 1
x ln x dx = x2 ln x − dx = − .

0 2 0 0 2 4

Therefore, the original limit is e−1/4 .

2
Z 1
10. [8] Evaluate the integral ln x ln(1 − x) dx.
0

π2 x2 x3
Answer: 2− 6 We have the MacLaurin expansion ln(1 − x) = −x − 2 − 3 − · · · . So

1 1 ∞ ∞
xn 1 1 n
Z Z X X Z
ln x ln(1 − x) dx = − ln x dx = − x ln x dx.
0 0 n=1
n n=1
n 0

Using integration by parts, we get


1 Z
1 n+1 1
xn
Z
x ln x 1
xn ln x dx = − dx = − .

0 n+1 0 n + 1 (n + 1)2
0

(We used the fact that limx→0 xn ln x = 0 for n > 0, which can be proven using l’Hôpital’s rule.)
Therefore, the original integral equals to
∞ ∞  
X 1 X 1 1 1
= − − .
n=1
n(n + 1)2 n=1
n n + 1 (n + 1)2

π2
P∞ 1
Telescoping the sum and using the well-known identity n=0 n2 = 6 , we see that the above sum is
2
equal to 2 − π6 .

3
12th Annual Harvard-MIT Mathematics Tournament
Saturday 21 February 2009

Individual Round: Calculus Test

1. [3] Let f be a differentiable real-valued function defined on the positive real numbers. The tangent
lines to the graph of f always meet the y-axis 1 unit lower than where they meet the function. If
f (1) = 0, what is f (2)?
2. [3] The differentiable function F : R → R satisfies F (0) = −1 and

d
F (x) = sin(sin(sin(sin(x)))) · cos(sin(sin(x))) · cos(sin(x)) · cos(x).
dx
Find F (x) as a function of x.
3. [4] Compute eA where A is defined as
4/3
2x2 + x + 1
Z
dx.
3/4 x3 + x2 + x + 1

4. [4] Let P be a fourth degree polynomial, with derivative P 0 , such that P (1) = P (3) = P (5) = P 0 (7) = 0.
Find the real number x 6= 1, 3, 5 such that P (x) = 0.
5. [4] Compute
π π π π π
    
sin 3 + 4h − 4 sin 3 + 3h + 6 sin 3 + 2h − 4 sin 3 + h + sin 3
lim .
h→0 h4

6. [5] Let p0 (x), p1 (x), p2 (x), . . . be polynomials such that p0 (x) = x and for all positive integers n,
d
dx pn (x) = pn−1 (x). Define the function p(x) : [0, ∞) → R by p(x) = pn (x) for all x ∈ [n, n + 1). Given
that p(x) is continuous on [0, ∞), compute

X
pn (2009).
n=0

7. [5] A line in the plane is called strange if it passes through (a, 0) and (0, 10 − a) for some a in the
interval [0, 10]. A point in the plane is called charming if it lies in the first quadrant and also lies below
some strange line. What is the area of the set of all charming points?
8. [7] Compute √
Z 3  2x2 +1 
2
x2x +1
+ ln x2x dx.
1

9. [7] Let R be the region in the plane bounded by the graphs of y = x and y = x2 . Compute the volume
of the region formed by revolving R around the line y = x.
10. [8] Let a and b be real numbers satisfying a > b > 0. Evaluate
Z 2π
1
dθ.
0 a + b cos(θ)

Express your answer in terms of a and b.


12th Annual Harvard-MIT Mathematics Tournament
Saturday 21 February 2009

Individual Round: Calculus Test Solutions

1. [3] Let f be a differentiable real-valued function defined on the positive real numbers. The tangent
lines to the graph of f always meet the y-axis 1 unit lower than where they meet the function. If
f (1) = 0, what is f (2)?
Answer: ln 2

Solution: The tangent line to f at x meets the y-axis at f (x) − 1 for any x, so the slope of the
tangent line is f 0 (x) = x1 , and so f (x) = ln(x) + C for some a. Since f (1) = 0, we have C = 0, and so
f (x) = ln(x). Thus f (2) = ln(2).

2. [3] The differentiable function F : R → R satisfies F (0) = −1 and


d
F (x) = sin(sin(sin(sin(x)))) · cos(sin(sin(x))) · cos(sin(x)) · cos(x).
dx
Find F (x) as a function of x.
Answer: − cos(sin(sin(sin(x))))

Solution: Substituting u = sin(sin(sin(x))), we find


Z
F (x) = sin(u)du = − cos(u) + C.

for some C. Since F (0) = 1 we find C = 0.

3. [4] Compute eA where A is defined as


4/3
2x2 + x + 1
Z
dx.
3/4 x3+ x2 + x + 1

16
Answer: 9

1
Solution: We can use partial fractions to decompose the integrand to x+1 + x2x+1 , and then integrate
the addends separately by substituting u = x + 1 for the former and u = x2 + 1 for latter, to obtain
4/3 √ 4/3
ln(x + 1) + 21 ln(x2 + 1) 3/4 = ln((x + 1) x2 + 1) 3/4 = ln 16 A
9 . Thus e = 16/9.

Alternate solution: Substituting u = 1/x, we find


3/4 4/3
2u + u2 + u3
Z Z
1 2/u + 1 + u
A= (− )du = du
4/3 1 + u + u2 + u3 u2 3/4 1 + u + u2 + u3

Adding this to the original integral, we find


Z 4/3 Z 4/3
2/u + 2 + 2u + 2u2 2
2A = 2 + u3
du = du
3/4 1 + u + u 3/4 u

Thus A = ln 16 A
9 and e =
16
9 .

1
4. [4] Let P be a fourth degree polynomial, with derivative P 0 , such that P (1) = P (3) = P (5) = P 0 (7) = 0.
Find the real number x 6= 1, 3, 5 such that P (x) = 0.
89
Answer: 11

P 0 (7)
Solution: Observe that 7 is not a root of P . If r1 , r2 , r3 , r4 are the roots of P , then P (7) =
P 1 P −1 −1
1
i 7−ri = 0. Thus r4 = 7 − i6=4 7−ri = 7 + 16 + 14 + 12 = 7 + 12/11 = 89/11.

5. [4] Compute
π π π π π
    
sin 3 + 4h − 4 sin 3 + 3h + 6 sin 3 + 2h − 4 sin 3 + h + sin 3
lim
h→0 h4

3
Answer: 2

f (x+h)−f (x)
Solution: The derivative of a function is defined as f 0 (x) = limh→0 h . Iterating this
formula four times yields

f (x + 4h) − 4f (x + 3h) + 6f (x + 2h) − 4f (x + h) + f (x)


f (4) (x) = lim .
h→0 h4

Substituting f = sin and x = π/3, the expression is equal to sin(4) (π/3) = sin(π/3) = 3
2 .

6. [5] Let p0 (x), p1 (x), p2 (x), . . . be polynomials such that p0 (x) = x and for all positive integers n,
d
dx pn (x) = pn−1 (x). Define the function p(x) : [0, ∞) → R x by p(x) = pn (x) for all x ∈ [n, n + 1].
Given that p(x) is continuous on [0, ∞), compute

X
pn (2009).
n=0

Answer: e2010 − e2009 − 1

Solution: By writing out the first few polynomials, one can guess and then show by induction that
1 1 n
pn (x) = (n+1)! (x + 1)n+1 − n! x . Thus the sum evaluates to e2010 − e2009 − 1 by the series expansion
x
of e .

7. [5] A line in the plane is called strange if it passes through (a, 0) and (0, 10 − a) for some a in the
interval [0, 10]. A point in the plane is called charming if it lies in the first quadrant and also lies below
some strange line. What is the area of the set of all charming points?
Answer: 50/3

Solution: The strange lines form an envelope (set of tangent lines) of a curve f (x), and we first find
the equation for f on [0, 10]. Assuming the derivative f 0 is continuous, the point of tangency of the
line ` through (a, 0) and (0, b) to f is the limit of the intersection points of this line with the lines `
passing through (a + , 0) and (0, b − ) as  → 0. If these limits exist, then the derivative is indeed
continuous and we can calculate the function from the points of tangency.
a(a−)
The intersection point of ` and ` can be calculated to have x-coordinate a+b , so the tangent point
2 2
b2 (10−a)2
of ` has x-coordinate lim→0 a(a−)
a+b = a
a+b = a
10 . Similarly, the y-coordinate is 10 = 10 . Thus,

2
√ √
solving for the y coordinate in terms of the x coordinate for a ∈ [0, 10], we find f (x) = 10−2 10 x+x,
and so the area of the set of charming points is
Z 10  √ √ 
10 − 2 10 x + x dx = 50/3.
0

8. [7] Compute √
Z 3  2x2 +1 
2
x2x +1
+ ln x2x dx.
1

Answer: 13

Solution: Using the fact that x = eln(x) , we evaluate the integral as follows:
Z  2x2 +1  Z
2 2 2
x2x +1 + ln x2x dx = x2x +1 + x2x +1 ln(x2 )dx
Z
2
= eln(x)(2x +1) (1 + ln(x2 ))dx
Z
2 2
= xex ln(x ) (1 + ln(x2 ))dx

Noticing that the derivative of x2 ln(x2 ) is 2x(1 + ln(x2 )), it follows that the integral evaluates to
1 x2 ln(x2 ) 1 2
e = x2x .
2 2

Evaluating this from 1 to 3 we obtain the answer.

9. [7] let R be the region in the plane bounded by the graphs of y = x and y = x2 . Compute the volume
of the region formed by revolving R around the line y = x.


Answer: 60

Solution: We integrate from 0 to 1 using the method of washers. Fix d between 0 and 1. Let the
line x = d intersect the graph of y = x2 at Q, and let the line x = d intersect the graph of y = x at
P . Then P = (d, d), and Q = (d, d2 ). Now drop a perpendicular from Q to the line y = x,√and let R
be the foot of this perpendicular. Because P√ QR is a 45 − 45 − 90√triangle, QR = (d − d2 )/ 2. So the
2
differential washer has a √
radius
√ of (d − d )/ 2 and a height of 2dx. So we integrate (from 0 to 1)
the expression [(x − x2 )/ 2]2 2dx, and the answer follows.

10. [8] Let a and b be real numbers satisfying a > b > 0. Evaluate
Z 2π
1
dθ.
0 a + b cos(θ)

Express your answer in terms of a and b.


Answer: √ 2π
a2 −b2

3
Solution: Using the geometric series formula, we can expand the integral as follows:
Z 2π Z 2π  2
1 1 b b
dθ = 1+
cos(θ) + cos2 (θ)dθ
0 a + b cos(θ) a 0 a a
∞ Z  n  iθ n
1 X 2π b e + e−iθ
= dθ
a n=0 0 a 2
∞  n 2n
2π X b2 n
= dθ
a n=0 a2 22n

1 2n

To evaluate this sum, recall that Cn = n+1 n is the nth Catalan number. The generating function
for the Catalan numbers is √

X
n 1 − 1 − 4x
Cn x = ,
n=0
2x
P 2n n
and taking the derivative of x times this generating function yields √ 1
n x = 1−4x
. Thus the

integral evaluates to √a2 −b2 , as desired.

4
13th Annual Harvard-MIT Mathematics Tournament
Saturday 20 February 2010
Calculus Subject Test
1. [3] Suppose that p(x) is a polynomial and that p(x) − p0 (x) = x2 + 2x + 1. Compute p(5).
2. [3] Let f be a function such that f (0) = 1, f 0 (0) = 2, and

f 00 (t) = 4f 0 (t) − 3f (t) + 1

for all t. Compute the 4th derivative of f , evaluated at 0.

3. [4] Let p be a monic cubic polynomial such that p(0) = 1 and such that all the zeros of p0 (x) are also
zeros of p(x). Find p. Note: monic means that the leading coefficient is 1.
Pn
k=1 | cos(k)|
4. [4] Compute lim .
n→∞ n
5. [4] Let the functions f (α, x) and g(α) be defined as

x α

d4 f

2
f (α, x) = g(α) =
x−1 dx4 x=2

Then g(α) is a polynomial in α. Find the leading coefficient of g(α).

6. [5] Let f (x) = x3 − x2 . For a given value of c, the graph of f (x), together with the graph of the line
c + x, split the plane up into regions. Suppose that c is such that exactly two of these regions have
finite area. Find the value of c that minimizes the sum of the areas of these two regions.
7. [6] Let a1 , a2 , and a3 be nonzero complex numbers with non-negative real and imaginary parts. Find
the minimum possible value of

|a1 + a2 + a3 |
p .
3
|a1 a2 a3 |
∞ ∞
X 1 X
8. [6] Let f (n) = . Calculate f (n).
k n · k! n=2
k=2

9. [7] Let x(t) be a solution to the differential equation

2
(x + x0 ) + x · x00 = cos t
q
with x(0) = x0 (0) = 2 π

5. Compute x 4 .
n
X 1
10. [8] Let f (n) = . Then there exists constants γ, c, and d such that
k
k=1
c d 1
f (n) = ln(n) + γ + + 2 + O( 3 ),
n n n
where the O( n13 ) means terms of order 1
n3 or lower. Compute the ordered pair (c, d).
13th Annual Harvard-MIT Mathematics Tournament
Saturday 20 February 2010
Calculus Subject Test
1. [3] Suppose that p(x) is a polynomial and that p(x) − p0 (x) = x2 + 2x + 1. Compute p(5).
Answer: 50 Observe that p(x) must be quadratic. Let p(x) = ax2 + bx + c. Comparing coefficients
gives a = 1, b − 2a = 2, and c − b = 1. So b = 4, c = 5, p(x) = x2 + 4x + 5 and p(5) = 25 + 20 + 5 = 50.
2. [3] Let f be a function such that f (0) = 1, f 0 (0) = 2, and

f 00 (t) = 4f 0 (t) − 3f (t) + 1

for all t. Compute the 4th derivative of f , evaluated at 0.


Answer: 54 Putting t = 0 gives f 00 (0) = 6. By differentiating both sides, we get f (3) (t) = 4f 00 (t) −
3f 0 (t) and f (3) (0) = 4·6−3·2 = 18. Similarly, f (4) (t) = 4f (3) (t)−3f 00 (t) and f (4) (0) = 4·18−3·6 = 54.
3. [4] Let p be a monic cubic polynomial such that p(0) = 1 and such that all the zeros of p0 (x) are also
zeros of p(x). Find p. Note: monic means that the leading coefficient is 1.
Answer: (x + 1)3 A root of a polynomial p will be a double root if and only if it is also a root of
p0 . Let a and b be the roots of p0 . Since a and b are also roots of p, they are double roots of p. But p
can have only three roots, so a = b and a becomes a double root of p0 . This makes p0 (x) = 3c(x − a)2
for some constant 3c, and thus p(x) = c(x − a)3 + d. Because a is a root of p and p is monic, d = 0
and c = 1. From p(0) = 1 we get p(x) = (x + 1)3 .
Pn
k=1 | cos(k)|
4. [4] Compute lim .
n→∞ n
2
Answer: π The main idea lies on the fact that positive integers are uniformly distributed modulo
π. (In the other words, if each integer n is written as qπ + r where q is an integer and 0 ≤ r < π, the
value of r will distribute uniformly in the interval [0, π].) Using this fact, the summation is equivalent
to the average value (usingR the Riemann summation) of the function | cos(k)| over the interval [0, π].
π
Therefore, the answer is π1 0 | cos(k)| = π2 .
5. [4] Let the functions f (α, x) and g(α) be defined as

x α

d4 f

2
f (α, x) = g(α) =
x−1 dx4 x=2

Then g(α) is a polynomial in α. Find the leading coefficient of g(α).


1

Answer: 16 Write the first equation as (x − 1)f = x2 . For now, treat α as a constant. From this
equation, repeatedly applying derivative with respect to x gives
 α   x α−1
(x − 1)f 0 + f =
2 2
 α   α − 1   x α−2
00 0
(x − 1)f + 2f =
2 2 2
 α   α − 1   α − 2   x α−3
(x − 1)f (3) + 3f 00 =
2 2 2 2
 α   α − 1   α − 2   α − 3   x α−4
(x − 1)f (4) + 4f (3) =
2 2 2 2 2

Substituting x = 2 to all equations gives g(α) = f (4) (α, 2) = α2 α−1 α−2


   α−3 
2 2 2 − 4f (3) (α, 2).
1
Because f (3) (α, 2) is a cubic polynomial in α, the leading coefficient of g(α) is 16 .

Calculus Subject Test


6. [5] Let f (x) = x3 − x2 . For a given value of c, the graph of f (x), together with the graph of the line
c + x, split the plane up into regions. Suppose that c is such that exactly two of these regions have
finite area. Find the value of c that minimizes the sum of the areas of these two regions.
Answer: − 11
27 Observe that f (x) can be written as (x − 31 )3 − 13 (x − 13 ) − 2
27 , which has 180◦
2
symmetry around the point ( 13 , − 27 ). Suppose the graph of f cuts the line y = c + x into two segments
of lengths a and b. When we move the line toward point P with a small distance ∆x (measured along
the line perpendicular to y = x + c), the sum of the enclosed areas will increase by |a − b|(∆x). As long
as the line x + c does not passes through P , we can find a new line x + c∗ that increases the sum of
the enclosed areas. Therefore, the sum of the areas reaches its maximum when the line passes through
2
P . For that line, we can find that c = y − x = − 27 − 13 = − 27
11
.

7. [6] Let a1 , a2 , and a3 be nonzero complex numbers with non-negative real and imaginary parts. Find
the minimum possible value of

|a1 + a2 + a3 |
p .
3
|a1 a2 a3 |
√ √
Answer: 3 3 2 Write a1 in its polar form reiθ where 0 ≤ θ ≤ π2 . Suppose a2 , a3 and r are fixed so
that the denominator is constant. Write a2 + a3 as seiφ . Since a2 and a3 have non-negative real and
imaginary parts, the angle φ lies between 0 and π2 . Consider the function

f (θ) = |a1 + a2 + a3 |2 = |reiθ + seiφ |2 = r2 + 2rs cos(θ − φ) + s2 .

Its second derivative is f 00 (θ) = −2rs(cos(θ − φ))). Since − π2 ≤ (θ − φ) ≤ π2 , we know that f 00 (θ) < 0
and f is concave. Therefore, to minimize f , the angle θ must be either 0 or π2 . Similarly, each of a1 , a2
and a3 must be either purely real or purely imaginary to minimize f and the original fraction.
By the AM-GM inequality, if a1 , a2 and a3 are all real or all imaginary, then the minimum value of
the fraction is 3. Now suppose only two of the ai ’s, say, a1 and a2 are real. Since the fraction is
homogenous, we may fix a1 + a2 - let the sum be 2. The term a1 a2 in the denominator acheives its
maximum only when a1 and a2 are equal, i.e. when a1 = a2 = 1. Then, if a3 = ki for some real
number k, then the expression equals √
k2 + 4
√3
.
k
√ √
3
Squaring and√ taking the derivative, we find that the minimum value of the fraction is 3 2, attained
when k = 2. With similar reasoning, the case where only one of the ai ’s is real yields the same
minimum value.
∞ ∞
X 1 X
8. [6] Let f (n) = . Calculate f (n).
k n · k! n=2
k=2

Answer: 3 − e
∞ ∞ X

X X 1
f (n) =
n=2
kn · k!
k=2 n=2
∞ ∞
X 1
X 1
=
k! n=2 k n
k=2

X 1 1
= ·
k! k(k − 1)
k=2

X 1 1
= · 2
(k − 1)! k (k − 1)
k=2

Calculus Subject Test


∞  
X 1 1 1 1
= − −
(k − 1)! k − 1 k 2 k
k=2
∞  
X 1 1 1
= − −
(k − 1)(k − 1)! k · k! k!
k=2
∞   X ∞
X 1 1 1
= − −
(k − 1)(k − 1)! k · k! k!
k=2 k=2
 
1 1 1
= − e− −
1 · 1! 0! 1!
=3−e

9. [7] Let x(t) be a solution to the differential equation

2
(x + x0 ) + x · x00 = cos t
q
with x(0) = x0 (0) = 2 π

5. Compute x 4 .

4
Answer: 450
5 Rewrite the equation as x2 + 2xx0 + (xx0 )0 = cos t. Let y = x2 , so y 0 = 2xx0 and
the equation becomes y + y 0 + 12 y 00 = cos t. The term cos t suggests that the particular solution should
be in the form A sin t + B cos t. By substitution and coefficient comparison, we get A = 54 and B = 25 .
Since the function y(t) = 45 sin t + 52 cos t already satisfies the initial conditions y(0) = x(0)2 = 25 and
y 0 (0) = 2x(0)x0 (0) = 45 , the function y also solves the initial value problem. Note  that since x is
positive at t = 0 and y = x2 never reaches zero before t reaches π4 , the value of x π4 must be positive.
q  q √ √
4
Therefore, x π4 = + y π4 = 65 · 22 = 450

5 .

n
X 1
10. [8] Let f (n) = . Then there exists constants γ, c, and d such that
k
k=1
c d 1
f (n) = ln(n) + γ + + 2 + O( 3 ),
n n n
where the O( n13 ) means terms of order 1
n3 or lower. Compute the ordered pair (c, d).
Answer: ( 21 , − 12
1
) From the given formula, we pull out the term k
n3 from O( n14 ), making f (n) =
c d k
log(n) + γ + n + n2 + n3+ O( n14 ). Therefore,
         
n+1 1 1 1 1 1 1 1
f (n+1)−f (n) = log −c − −d − −k − +O .
n n n+1 n2 (n + 1)2 n3 (n + 1)3 n4
1 1
For the left hand side, f (n + 1) − f (n) = n+1 . By substituting x = n, the formula above becomes
2
x 1 x+2 4 x + 3x + 3
= log (1 + x) − cx2 · − dx3 · − kx · + O(x4 ).
x+1 x+1 (x + 1)2 (x + 1)3
1 1
Because x is on the order of n , (x+1)3 is on the order of a constant. Therefore, all the terms in the
4 x2 +3x+3
expansion of kx · (x+1)3 are of order x4 or higher, so we can collapse it into O(x4 ). Using the Taylor
expansions, we get
 
1 1
x 1 − x + x2 + O x4 = x − x2 + x3 − cx2 (1 − x) − dx3 (2) + O x4 .
  
2 3
1 1
Coefficient comparison gives c = 2 and d = − 12 .

Calculus Subject Test


14th Annual Harvard-MIT Mathematics Tournament
Saturday 12 February 2011

1. Let a, b, and c be positive real numbers. Determine the largest total number of real roots that the
following three polynomials may have among them: ax2 + bx + c, bx2 + cx + a, and cx2 + ax + b.
2. Josh takes a walk on a rectangular grid of n rows and 3 columns, starting from the bottom left corner.
At each step, he can either move one square to the right or simultaneously move one square to the left
and one square up. In how many ways can he reach the center square of the topmost row?
3. Let f : R → R be a differentiable function such that f (0) = 0, f (1) = 1, and |f 0 (x)| ≤ 2 for all real
Z 1
numbers x. If a and b are real numbers such that the set of possible values of f (x) dx is the open
0
interval (a, b), determine b − a.
4. Let ABC be a triangle such that AB = 7, and let the angle bisector of ∠BAC intersect line BC at
D. If there exist points E and F on sides AC and BC, respectively, such that lines AD and EF
are parallel and divide triangle ABC into three parts of equal area, determine the number of possible
integral values for BC.
5. Nathaniel and Obediah play a game in which they take turns rolling a fair six-sided die and keep a
running tally of the sum of the results of all rolls made. A player wins if, after he rolls, the number
on the running tally is a multiple of 7. Play continues until either player wins, or else indefinitely. If
Nathaniel goes first, determine the probability that he ends up winning.
6. Let a ? b = ab + a + b for all integers a and b. Evaluate 1 ? (2 ? (3 ? (4 ? . . . (99 ? 100) . . .))).
7. Let f : [0, 1) → R be a function that satisfies the following condition: if

X an
x= n
= .a1 a2 a3 . . .
n=1
10

is the decimal expansion of x and there does not exist a positive integer k such that an = 9 for all
n ≥ k, then

X an
f (x) = 2n
.
n=1
10

Determine f 0 13 .


8. Find all integers x such that 2x2 + x − 6 is a positive integral power of a prime positive integer.
9. Let ABCDEF be a regular hexagon of area 1. Let M be the midpoint of DE. Let X be the
intersection of AC and BM , let Y be the intersection of BF and AM , and let Z be the intersection
of AC and BF . If [P ] denotes the area of polygon P for any polygon P in the plane, evaluate
[BXC] + [AY F ] + [ABZ] − [M XZY ].
10. For all real numbers x, let
1
f (x) = √
2011
.
1 − x2011
Evaluate (f (f (. . . (f (2011)) . . .)))2011 , where f is applied 2010 times.
Z ∞ 2011
ln x
11. Evaluate dx.
1 x
12. Let f (x) = x2 + 6x + c for all real numbers x, where c is some real number. For what values of c does
f (f (x)) have exactly 3 distinct real roots?
13. Sarah and Hagar play a game of darts. Let O0 be a circle of radius 1. On the nth turn, the player
whose turn it is throws a dart and hits a point pn randomly selected from the points of On−1 . The
player then draws the largest circle that is centered at pn and contained in On−1 , and calls this circle
On . The player then colors every point that is inside On−1 but not inside On her color. Sarah goes
first, and the two players alternate turns. Play continues indefinitely. If Sarah’s color is red, and
Hagar’s color is blue, what is the expected value of the area of the set of points colored red?
14. How many polynomials P with integer coefficients and degree at most 5 satisfy 0 ≤ P (x) < 120 for all
x ∈ {0, 1, 2, 3, 4, 5}?
15. Let f : [0, 1] → [0, 1] be a continuous function such that f (f (x)) = 1 for all x ∈ [0, 1]. Determine the
Z 1
set of possible values of f (x) dx.
0

16. Let f (x) = x2 − r2 x + r3 for all real numbers x, where r2 and r3 are some real numbers. Define a
sequence {gn } for all nonnegative integers n by g0 = 0 and gn+1 = f (gn ). Assume that {gn } satisfies
the following three conditions: (i) g2i < g2i+1 and g2i+1 > g2i+2 for all 0 ≤ i ≤ 2011; (ii) there exists
a positive integer j such that gi+1 > gi for all i > j, and (iii) {gn } is unbounded. If A is the greatest
number such that A ≤ |r2 | for any function f satisfying these properties, find A.
17. Let f : (0, 1) → (0, 1) be a differentiable function with a continuous derivative such that for every
positive integer n and odd positive integer a < 2n , there exists n
 an odd positive integer b < 2 such
a b 0 1

that f 2n = 2n . Determine the set of possible values of f 2 .
2π 2π
18. Let z = cos + i sin , and let
2011 2011
2008 · 2009 2009 · 2010
P (x) = x2008 + 3x2007 + 6x2006 + . . . x+
2 2
for all complex numbers x. Evaluate P (z)P (z 2 )P (z 3 ) . . . P (z 2010 ).
19. Let
1
F (x) = ,
(2 − x − x5 )2011

X
and note that F may be expanded as a power series so that F (x) = an xn . Find an ordered pair of
n=0
an
positive real numbers (c, d) such that lim = c.
n→∞ nd
p
20. Let {an } and
p {b n } be sequences defined recursively by a0 = 2; b0 = 2, and an+1 = an 1 + a2n + b2n −bn ;
2 2
bn+1 = bn 1 + an + bn + an . Find the ternary (base 3) representation of a4 and b4 .
14th Annual Harvard-MIT Mathematics Tournament
Saturday 12 February 2011
Algebra & Calculus Individual Test
1. Let a, b, and c be positive real numbers. Determine the largest total number of real roots that the
following three polynomials may have among them: ax2 + bx + c, bx2 + cx + a, and cx2 + ax + b.
Answer: 4 If all the polynomials had real roots, their discriminants would all be nonnegative: a2 ≥
4bc, b2 ≥ 4ca, and c2 ≥ 4ab. Multiplying these inequalities gives (abc)2 ≥ 64(abc)2 , a contradiction.
Hence one of the quadratics has no real roots. The maximum of 4 real roots is attainable: for example,
the values (a, b, c) = (1, 5, 6) give −2, −3 as roots to x2 + 5x + 6 and −1, − 51 as roots to 5x2 + 6x + 1.
2. Josh takes a walk on a rectangular grid of n rows and 3 columns, starting from the bottom left corner.
At each step, he can either move one square to the right or simultaneously move one square to the left
and one square up. In how many ways can he reach the center square of the topmost row?
Answer: 2n−1 Note that Josh must pass through the center square of each row. There are 2 ways
to get from the center square of row k to the center square of row k + 1. So there are 2n−1 ways to get
to the center square of row n.
3. Let f : R → R be a differentiable function such that f (0) = 0, f (1) = 1, and |f 0 (x)| ≤ 2 for all real
Z 1
numbers x. If a and b are real numbers such that the set of possible values of f (x) dx is the open
0
interval (a, b), determine b − a.
3
Answer: 4 Draw lines of slope ±2 passing through (0, 0) and (1, 1). These form a parallelogram with
vertices (0, 0), (.75, 1.5), (1, 1), (.25, −.5). By the mean value theorem, no point of (x, f (x)) lies outside
this parallelogram, but we can construct functions arbitrarily close to the top or the bottom of the
parallelogram while satisfying the condition of the problem. So (b − a) is the area of this parallelogram,
which is 34 .
4. Let ABC be a triangle such that AB = 7, and let the angle bisector of ∠BAC intersect line BC at
D. If there exist points E and F on sides AC and BC, respectively, such that lines AD and EF
are parallel and divide triangle ABC into three parts of equal area, determine the number of possible
integral values for BC.
Answer: 13

14
F
E
D

A 7 B

Algebra & Calculus Individual Test


Note that such E, F exist if and only if
[ADC]
= 2. (1)
[ADB]
([ ] denotes area.) Since AD is the angle bisector, and the ratio of areas of triangles with equal height
is the ratio of their bases,
AC DC [ADC]
= = .
AB DB [ADB]
Hence (1) is equivalent to AC = 2AB = 14. Then BC can be any length d such that the triangle
inequalities are satisfied:
d + 7 > 14
7 + 14 > d
Hence 7 < d < 21 and there are 13 possible integral values for BC.
5. Nathaniel and Obediah play a game in which they take turns rolling a fair six-sided die and keep a
running tally of the sum of the results of all rolls made. A player wins if, after he rolls, the number
on the running tally is a multiple of 7. Play continues until either player wins, or else indefinitely. If
Nathaniel goes first, determine the probability that he ends up winning.
5
Answer: 11 For 1 ≤ k ≤ 6, let xk be the probability that the current player, say A, will win when
the number on the tally at the beginning of his turn is k modulo 7. The probability that the total
is l modulo 7 after his roll is 61 for each l 6≡ k (mod 7); in particular, there is a 16 chance he wins
immediately. The chance that A will win if he leaves l on the board after his turn is 1 − xl . Hence for
1 ≤ k ≤ 6,
1 X 1
xk = (1 − xl ) + .
6 6
1≤l≤6, l6=k
P6 xk −s
Letting s = l=1 xl , this becomes xk = 6 + 1 or 5x6k = − 6s + 1. Hence x1 = · · · = x6 , and 6xk = s
for every k. Plugging this in gives 11x
6 = 1, or xk = 11 .
k 6

Since Nathaniel cannot win on his first turn, he leaves Obediah with a number not divisible by 7.
6 5
Hence Obediah’s chance of winning is 11 and Nathaniel’s chance of winning is 11 .
6. Let a ? b = ab + a + b for all integers a and b. Evaluate 1 ? (2 ? (3 ? (4 ? . . . (99 ? 100) . . .))).
Answer: 101! − 1 We will first show that ? is both commutative and associative.

• Commutativity: a ? b = ab + a + b = b ? a
• Associativity: a ? (b ? c) = a(bc + b + c) + a + bc + b + c = abc + ab + ac + bc + a + b + c and
(a ? b) ? c = (ab + a + b)c + ab + a + b + c = abc + ab + ac + bc + a + b + c. So a ? (b ? c) = (a ? b) ? c.

So we need only calculate ((. . . (1 ? 2) ? 3) ? 4) . . . ? 100). We will prove by induction that

((. . . (1 ? 2) ? 3) ? 4) . . . ? n) = (n + 1)! − 1.

• Base case (n = 2): (1 ? 2) = 2 + 1 + 2 = 5 = 3! − 1


• Inductive step:
Suppose that
(((. . . (1 ? 2) ? 3) ? 4) . . . ? n) = (n + 1)! − 1.
Then,

((((. . . (1 ? 2) ? 3) ? 4) . . . ? n) ? (n + 1)) = ((n + 1)! − 1) ? (n + 1)


= (n + 1)!(n + 1) − (n + 1) + (n + 1)! − 1 + (n + 1)
= (n + 2)! − 1

Algebra & Calculus Individual Test


Hence, ((. . . (1 ? 2) ? 3) ? 4) . . . ? n) = (n + 1)! − 1 for all n. For n = 100, this results to 101! − 1.
7. Let f : [0, 1) → R be a function that satisfies the following condition: if

X an
x= n
= .a1 a2 a3 . . .
n=1
10

is the decimal expansion of x and there does not exist a positive integer k such that an = 9 for all
n ≥ k, then

X an
f (x) = .
n=1
102n

Determine f 0 13 .

P∞
Answer: 0 Note that 13 = n=1 103n .
Clearly f is an increasing function. Also for any integer n ≥ 1, we see from decimal expansions that
f ( 31 ± 101n ) − f ( 13 ) = ± 1012n .
Consider h such that 10−n−1 ≤ |h| < 10−n . The two 1properties
1
of f outlined above show that |f ( 3 +
1
f ( +h)−f ( 3 )
h) − f ( 13 )| < 1012n . And from | h1 | ≤ 10n+1 , we get 3 h 1
< 10n−1 . Taking n → ∞ gives h → 0
and f 0 ( 13 ) = limn→∞ 1
10n−1 = 0.

8. Find all integers x such that 2x2 + x − 6 is a positive integral power of a prime positive integer.
Answer: −3, 2, 5 Let f (x) = 2x2 + x − 6 = (2x − 3)(x + 2). Suppose a positive integer a divides
both 2x − 3 and x + 2. Then a must also divide 2(x + 2) − (2x − 3) = 7. Hence, a can either be 1 or
7. As a result, 2x − 3 = 7n or −7n for some positive integer n, or either x + 2 or 2x − 3 is ±1. We
consider the following cases:

• (2x − 3) = 1. Then x = 2, which yields f (x) = 4, a prime power.


• (2x − 3) = −1. Then x = 1, which yields f (x) = −3, not a prime power.
• (x + 2) = 1). Then x = −1, which yields f (x) = −5 not a prime power.
• (x + 2) = −1. Then x = −3, which yields f (x) = 9, a prime power.
• (2x − 3) = 7. Then x = 5, which yields f (x) = 49, a prime power.
• (2x − 3) = −7. Then x = −2, which yields f (x) = 0, not a prime power.
(2x − 3) + 7
• (2x − 3) = ±7n , for n ≥ 2. Then, since x + 2 = , we have that x + 2 is divisible by 7
2
but not by 49. Hence x + 2 = ±7, yielding x = 5, −9. The former has already been considered,
while the latter yields f (x) = 147.

So x can be either -3, 2 or 5.


(Note: In the official solutions packet we did not list the answer -3. This oversight was quickly noticed
on the day of the test, and only the answer −3, 2, 5 was marked as correct.
9. Let ABCDEF be a regular hexagon of area 1. Let M be the midpoint of DE. Let X be the
intersection of AC and BM , let Y be the intersection of BF and AM , and let Z be the intersection
of AC and BF . If [P ] denotes the area of polygon P for any polygon P in the plane, evaluate
[BXC] + [AY F ] + [ABZ] − [M XZY ].
Answer: 0

Algebra & Calculus Individual Test


A B

Y X

F O C

E M D

Let O be the center of the hexagon. The desired area is [ABCDEF ] − [ACDM ] − [BF EM ]. Note
that [ADM ] = [ADE]/2 = [ODE] = [ABC], where the last equation holds because sin 60◦ = sin 120◦ .
Thus, [ACDM ] = [ACD] + [ADM ] = [ACD] + [ABC] = [ABCD], but the area of ABCD is half the
area of the hexagon. Similarly, the area of [BF EM ] is half the area of the hexagon, so the answer is
zero.
10. For all real numbers x, let
1
f (x) = √
2011
.
1 − x2011
Evaluate (f (f (. . . (f (2011)) . . .)))2011 , where f is applied 2010 times.

2011
1 − x2011
Answer: 20112011 Direct calculation shows that f (f (x)) = and f (f (f (x))) = x.
−x
Hence (f (f (. . . (f (x)) . . .))) = x, where f is applied 2010 times. So (f (f (. . . (f (2011)) . . .)))2011 =
20112011 .
Z ∞ 2011
ln x
11. Evaluate dx.
1 x
d n lnn−1 x
Answer: 2011!
20102012 By the chain rule, (ln x)n = .
dx x
We calculate the definite integral using integration by parts:

∞ x=∞ Z ∞
(ln x)n (ln x)n n(ln x)n−1
Z 
2011
dx = 2010
− 2011
dx
x=1 x −2010x x=1 x=1 −2010x

(ln x)n
But ln(1) = 0, and limx→∞ x2010 = 0 for all n > 0. So

∞ ∞
(ln x)n n(ln x)n−1
Z Z
dx = dx
x=1 x2011 x=1 2010x2011

It follows that ∞ ∞
(ln x)n 1
Z Z
n! n!
dx = dx =
x=1 x2011 2010n x=1 x2011 2010n+1
2011!
So the answer is 20102012 .

Algebra & Calculus Individual Test


12. Let f (x) = x2 + 6x + c for all real numbers x, where c is some real number. For what values of c does
f (f (x)) have exactly 3 distinct real roots?

11− 13
Answer: 2 Suppose f has only one distinct root r1 . Then, if x1 is a root of f (f (x)), it must
be the case that f (x1 ) = r1 . As a result, f (f (x)) would have at most two roots, thus not satisfying
the problem condition. Hence f has two distinct roots. Let them be r1 6= r2 .
Since f (f (x)) has just three distinct roots, either f (x) = r1 or f (x) = r2 has one distinct root. Assume
without loss of generality that r1 has one distinct root. Then f (x) = x2 + 6x + c = r1 has one root, so
that x2 + 6x + c − r1 is a square polynomial. Therefore, c − r1 = 9, so that r1 = c − 9. So c − 9 is a
root of f . So (c − 9)2 + 6(c − 9) + c = 0, yielding c2 − 11c + 27 = 0, or (c − 11 2 13
2 ) = 2 . This results to

c = 11±2 13 .
√ √ √ √ √
If c = 11−2 13 , f (x) = x2 + 6x + 11−2 13 = (x + 7+2 13 )(x + 5−2 13 ). We know f (x) = −7−2 13 has a
√ √
double root, -3. Now −5+2 13 > −7−2 13 so the second root is above the vertex of the parabola, and is
hit twice.
√ √ √ √ √
If c = 11+2 13 , f (x) = x2 + 6x + 11+2 13 = (x + 7−2 13 )(x + 5+2 13 ). We know f (x) = −7+2 13 has a
double root,

-3, and √this is the value√of f at the vertex of the parabola, so it is its minimum value.
Since −5−2 13 < −7+2 13 , f (x) = −5−2 13 has no solutions. So in this case, f has only one real root.

11− 13
So the answer is c = 2 .
Note: In the solutions packet we had both roots listed as the correct answer. We noticed this oversight
on the day of the test and awarded points only for the correct answer.
13. Sarah and Hagar play a game of darts. Let O0 be a circle of radius 1. On the nth turn, the player
whose turn it is throws a dart and hits a point pn randomly selected from the points of On−1 . The
player then draws the largest circle that is centered at pn and contained in On−1 , and calls this circle
On . The player then colors every point that is inside On−1 but not inside On her color. Sarah goes
first, and the two players alternate turns. Play continues indefinitely. If Sarah’s color is red, and
Hagar’s color is blue, what is the expected value of the area of the set of points colored red?

Answer: 7 Let f (r) be the average area colored red on a dartboard of radius r if Sarah plays first.
Then f (r) is proportional to r2 . Let f (r) = (πx)r2 for some constant x. We want to find f (1) = πx.
In the first throw, if Sarah’s dart hits a point with distance r from the center of O0 , the radius of O1
will be 1−r. The expected value of the area colored red will be (π −π(1−r)2 )+(π(1−r)2 −f (1−r)) =
π − f (1 − r). The value of f (1) is the average value of π − f (1 − r) over all points in O0 . Using polar
coordinates, we get


R R1
(π − f (1 − r))rdrdθ
0 0
f (1) = 2π
R R1
rdrdθ
0 0

R1
(π − πx(1 − r)2 )rdr
0
πx =
R1
rdr
0

Z 1
πx
= πr − πxr(1 − r)2 dr
2 0
πx π 1 2 1
= − πx( − + )
2 2 2 3 4
πx π πx
= −
2 2 12

Algebra & Calculus Individual Test



πx =
7

14. How many polynomials P with integer coefficients and degree at most 5 satisfy 0 ≤ P (x) < 120 for all
x ∈ {0, 1, 2, 3, 4, 5}?
Answer: 86400000 For each nonnegative integer i, let xi = x(x − 1) · · · (x − i + 1). (Define x0 = 1.)
Lemma: Each polynomial with integer coefficients f can be uniquely written in the form

f (x) = an xn + . . . + a1 x1 + a0 x0 , an 6= 0.

Proof: Induct on the degree. The base case (degree 0) is clear. If f has degree m with leading coefficient
c, then by matching leading coefficients we must have m = n and an = c. By the induction hypothesis,
f (x) − cxn can be uniquely written as an−1 xn−1 (x) + . . . + a1 x1 + a0 x0 .
There are 120 possible choices for a0 , namely any integer in [0, 120). Once a0 , . . . , ai−1 have been
chosen so 0 ≤ P (0), . . . , P (i − 1) < 120, for some 0 ≤ i ≤ 5, then we have

P (i) = ai i! + ai−1 ii−1 + · · · + a0

so by choosing ai we can make P (i) any number congruent to ai−1 ii−1 + · · · + a0 modulo i!. Thus there
are 120
i! choices for ai . Note the choice of ai does not affect the value of P (0), . . . , P (i − 1). Thus all
polynomials we obtain in this way are valid. The answer is
5
Y 120
= 86400000.
i=0
i!

Note: Their is also a solution involving finite differences that is basically equivalent to this solution.
One proves that for i = 0, 1, 2, 3, 4, 5 there are 5!
i! ways to pick the ith finite difference at the point 0.

15. Let f : [0, 1] → [0, 1] be a continuous function such that f (f (x)) = 1 for all x ∈ [0, 1]. Determine the
Z 1
set of possible values of f (x) dx.
0
R1
Answer: ( 34 , 1] Since the maximum value of f is 1, 0
f (x)dx ≤ 1.
By our condition f (f (x)) = 1, f is 1 at any point within the range of f . Clearly, 1 is in the range of
f , so f (1) = 1. Now f (x) is continuous on a closed interval so it attains a minimum value c. Since c
is in the range of f , f (c) = 1.
R1
If c = 1, f (x) = 1 for all x and 0 f (x)dx = 1.
Now assume c < 1. By the intermediate value theorem, since f is continuous it attains all values
between c and 1. So for all x ≥ c, f (x) = 1. Therefore,
Z 1 Z c
f (x)dx = f (x)dx + (1 − c).
0 0
Rc
Since f (x) ≥ c, 0 f (x)dx > c2 , and the equality is strict because f is continuous and thus cannot be
c for all x < c and 1 at c. So

1
1 3 3
Z
f (x)dx > c2 + (1 − c) = (c − )2 + ≥ .
0 2 4 4
3
R1
Therefore 4 < 0
f (x)dx ≤ 1, and it is easy to show that every value in this interval can be reached.

Algebra & Calculus Individual Test


16. Let f (x) = x2 − r2 x + r3 for all real numbers x, where r2 and r3 are some real numbers. Define a
sequence {gn } for all nonnegative integers n by g0 = 0 and gn+1 = f (gn ). Assume that {gn } satisfies
the following three conditions: (i) g2i < g2i+1 and g2i+1 > g2i+2 for all 0 ≤ i ≤ 2011; (ii) there exists
a positive integer j such that gi+1 > gi for all i > j, and (iii) {gn } is unbounded. If A is the greatest
number such that A ≤ |r2 | for any function f satisfying these properties, find A.
Answer: 2 Consider the function f (x) − x. By the constraints of the problem, f (x) − x must be
negative for some x, namely, for x = g2i+1 , 0 ≤ i ≤ 2011. Since f (x) − x is positive for x of large
absolute value, the graph of f (x) − x crosses the x-axis twice and f (x) − x has two real roots, say
a < b. Factoring gives f (x) − x = (x − a)(x − b), or f (x) = (x − a)(x − b) + x.
Now, for x < a, f (x) > x > a, while for x > b, f (x) > x > b. Let c 6= b be the number such that
f (c) = f (b) = b. Note that b is not the vertex as f (a) = a < b, so by the symmetry of quadratics, c
r2 r2 +1
exists and b+c2 = 2 as the vertex of the parabola. By the same token, 2 =
b+a
2 is the vertex of
f (x) − x. Hence c = a − 1. If f (x) > b then x < c or x > b. Consider the smallest j such that gj > b.
Then by the above observation, gj−1 < c. (If gi ≥ b then f (gi ) ≥ gi ≥ b so by induction, gi+1 ≥ gi for
all i ≥ j. Hence j > 1; in fact j ≥ 4025.) Since gj−1 = f (gj−2 ), the minimum value of f is less than c.
The minimum value is the value of f evaluated at its vertex, b+a−1 2 , so
 
b+a−1
f <c
2
  
b+a−1 b+a−1 b+a−1
−a −b + <a−1
2 2 2
1 − (b − a)2 b−a+1
+ <0
4 2
3 (b − a)2 b−a
< −
4 4 2
4 < (b − a − 1)2 .

Then either b − a − 1 < −2 or b − a − 1 > 2, but b > a, so the latter must hold and (b − a)2 > 9.
Now, the discriminant of f (x) − x equals (b − a)2 (the square of the difference of the two roots) and
(r2 + 1)2 − 4r3 (from the coefficients), so (r2 + 1)2 > 9 + 4r3 . But r3 = g1 > g0 = 0 so |r2 | > 2.
We claim that we can make |r2 | arbitrarily close to 2, so that the answer is 2. First √ define Gi , i ≥ 0
as follows. Let N ≥ 2012 be an integer. For ε > 0 let h(x) = x2 − 2 − ε, gε (x) = − x + 2 + ε and
G2N +1 = 2 + ε, and define Gi recursively by Gi = gε (Gi+1 ), Gi+1 = h(Gi ). (These two equations are
consistent.) Note the following.
√ √
(i) G2i < G2i+1 and G2i+1 > G2i+2 for 0 ≤ i ≤ N −1. First note G2N = − 4 + 2ε > − 4 + 2ε + ε2 =
−2 − ε. Let l be the negative solution to h(x) = x. Note that −2 − ε < G2N < l < 0 since
h(G2N ) > 0 > G2N . Now gε (x) is defined as long as x ≥ −2 − ε, and it sends (−2 − ε, l) into (l, 0)
and (l, 0) into (−2 − ε, l). It follows that the Gi , 0 ≤ i ≤ 2N are well-defined; moreover, G2i < l
and G2i+1 > l for 0 ≤ i ≤ N − 1 by backwards induction on i, so the desired inequalities follow.
(ii) Gi is increasing for i ≥ 2N + 1. Indeed, if x ≥ 2 + ε, then x2 − x = x(x − 1) > 2 + ε so h(x) > x.
Hence 2 + ε = G2N +1 < G2N +2 < · · · .
(iii) Gi is unbounded. This follows since h(x) − x = x(x − 2) − 2 − ε is increasing for x > 2 + ε, so Gi
increases faster and faster for i ≥ 2N + 1.
Now define f (x) = h(x + G0 ) − G0 = x2 + 2G0 x + G20 − G0 − 2 − ε. Note Gi+1 = h(Gi ) while
gi+1 = f (gi ) = h(gi + G0 ) − G0 , so by induction gi = Gi − G0 . Since {Gi }∞
i=0 satisfies (i), (ii), and
(iii), so does gi .
We claim that we can make G0 arbitrarily close to −1 by choosing N large enough and ε small enough;
this will make r2 = −2G0 arbitrarily close to 2. Choosing N large corresponds to taking G0 to be a
larger iterate of 2 + ε under gε (x). By continuity of this function with respect to x and ε, it suffices to

Algebra & Calculus Individual Test


take ε = 0 and show that (letting g = g0 )
g (n) (2) = g(· · · g (2) · · · ) → −1 as n → ∞.
| {z }
n
π
But note that for 0 ≤ θ ≤ 2,


   
θ π θ
g(−2 cos θ) = − 2 − 2 cos θ = −2 sin = 2 cos − .
2 2 2
Hence by induction, g (n) (−2 cos θ) = −2 cos π2 − π4 + · · · + (−1)n θ − 2πn . Hence g (n) (2) = g (n−1) (−2 cos 0)


converges to −2 cos( π2 − π4 + · · · ) = −2 cos( π3 ) = −1, as needed.


17. Let f : (0, 1) → (0, 1) be a differentiable function with a continuous derivative such that for every
positive integer n and odd positive integer a < 2n , there exists n
 an odd positive integer b < 2 such
a b 0 1

that f 2n = 2n . Determine the set of possible values of f 2 .
Answer: {−1, 1} The key step is to notice that for such a function f , f 0 (x) 6= 0 for any x.
Assume, for sake of contradiction that there exists 0 < y < 1 such that f 0 (y) = 0. Since f 0 is
a continuous function, there is some small interval (c, d) containing y such that |f 0 (x)| ≤ 21 for all
x ∈ (c, d). Now there exists some n, a such that 2an , a+1 2n are both in the interval (c, d). From the
f ( a+1
n ) − f ( a
n ) b 0
b
definition, 2
a+1 a
2
= 2n ( n − n ) = b0 − b where b, b0 are integers; one is odd, and one is
2n − 2n
2 2
0
even. So b − b is an odd integer. Since f is differentiable, by the mean value theorem there exists a
point where f 0 = b0 − b. But this point is in the interval (c, d), and |b0 − b| > 12 . This contradicts the
assumption that |f 0 (x)| ≤ 12 for all x ∈ (c, d).
Since f 0 (x) 6= 0, and f 0 is a continuous function, f 0 is either always positive or always negative. So f is
either increasing or decreasing. f ( 21 ) = 21 always. If f is increasing, it follows that f ( 14 ) = 14 , f ( 34 ) = 34 ,
and we can show by induction that indeed f ( 2an ) = 2an for all integers a, n. Since numbers of this form
are dense in the interval (0, 1), and f is a continuous function, f (x) = x for all x.
It can be similarly shown that if f is decreasing f (x) = 1 − x for all x. So the only possible values of
f 0 ( 21 ) are −1, 1.
Query: if the condition that the derivative is continuous were omitted, would the same result still
hold?
2π 2π
18. Let z = cos + i sin , and let
2011 2011
2008 · 2009 2009 · 2010
P (x) = x2008 + 3x2007 + 6x2006 + . . . x+
2 2
for all complex numbers x. Evaluate P (z)P (z 2 )P (z 3 ) . . . P (z 2010 ).
Answer: 20112009 · (10052011 − 10042011 ) Multiply P (x) by x − 1 to get

2009 · 2010
P (x)(x − 1) = x2009 + 2x2008 + . . . + 2009x − ,
2
or,

P (x)(x − 1) + 2010 · 1005 = x2009 + 2x2008 + . . . + 2009x + 2010.


Multiplying by x − 1 once again:

2010 · 2011
(x − 1)(P (x)(x − 1) + ) = x2010 + x2009 + . . . + x − 2010,
2
= (x2010 + x2009 + . . . + x + 1) − 2011.

Algebra & Calculus Individual Test


Hence,
(x2010 + x2009 + . . . + x + 1) − 2011
− 2011 · 1005
P (x) = x−1
x−1
Note that x2010 +x2009 +. . .+x+1 has z, z 2 , . . . , z 2010 as roots, so they vanish at those points. Plugging
those 2010 powers of z into the last equation, and multiplying them together, we obtain
1004
2010 (−2011) · 1005 · (x − )
1005 .
Y
i
P (z ) =
i=1
(x − 1)2

Note that (x − z)(x − z 2 ) . . . (x − z 2010 ) = x2010 + x2009 + . . . + 1. Using this, the product turns out to
be 20112009 · (10052011 − 10042011 ).
19. Let
1
F (x) = ,
(2 − x − x5 )2011

X
and note that F may be expanded as a power series so that F (x) = an xn . Find an ordered pair of
n=0
an
positive real numbers (c, d) such that lim = c.
n→∞ nd

Answer: ( 6201112010! , 2010) First notice that all the roots of 2 − x − x5 that are not 1 lie strictly
outside the unit circle. As such, we may write 2 − x − x5 as 2(1 − x)(1 − r1 x)(1 − r2 x)(1 − r3 x)(1 − r4 x)
1 b0 b1 b4 (1−x)
where |ri | < 1, and let (2−x−x5 ) = (1−x) + (1−r x) +. . .+ (1−r x) . We calculate b0 as limx→1 (2−x−x5 ) =
1 4
(−1)
limx→1 (−1−5x4 ) = 61 .
Now raise the equation above to the 2011th power.
 2011
1 1/6 b1 b4
= + + . . . +
(2 − x − x5 )2011 (1 − x) (1 − r1 x) (1 − r4 x)

Expand the right hand side using multinomial expansion and then apply partial fractions. The result
will be a sum of the terms (1 − x)−k and (1 − ri x)−k , where k ≤ 2011.
Since |ri | < 1, the power series of (1 − ri x)−k will have exponentially decaying coefficients, so we only
need to consider the (1 − x)−k terms. The coefficient of xn in the power series of (1 − x)−k is n+k−1

k−1 ,
which is a (k − 1)th degree polynomial in variable n. So when we sum up all coefficients, only the
power series of (1 − x)−2011 will have impact on the leading term n2010 .
The coefficient of the (1 − x)−2011 term in the multinomial expansion is ( 16 )2011 . The coefficient of
the xn term in the power series of (1 − x)−2011 is n+2010 1

2010 = 2010! n2010 + . . .. Therefore, (c, d) =
1
( 62011 2010! , 2010).
p
20. Let {an } and p {b n } be sequences defined recursively by a0 = 2; b0 = 2, and a n+1 = an 1 + a2n + b2n −bn ;
2 2
bn+1 = bn 1 + an + bn + an . Find the ternary (base 3) representation of a4 and b4 .
Answer: 1000001100111222 and 2211100110000012
p n
Note first that 1 + a2n + b2n = 32 . The proof is by induction; the base case follows trivially p from what
is given. For the inductive step, note that 1+a2n+1 +b2n+1 = 1+a2n (1+a2n +b2n )+b2n −2an bn 1 + a2n + b2n +
p
b2n (1 + a2n + b2n ) + a2n + 2an bn 1 + a2n + b2n = 1q
+ (a2n + b2n )(1 + a2n + b2n ) + a2n + b2n = (1 + a2n + b2n )2 .
n n+1
Invoking the inductive hypothesis, we see that 1 + a2n+1 + b2n+1 = (32 )2 = 32 , as desired.
The quickest way to finish from here is to consider a sequence of complex numbers {zn } defined by
n
zn = an + bn i for all nonnegative integers n. It should be clear that z0 = 2 + 2i and zn+1 = zn (32 + i).
0 1 2 3
Therefore, z4 = (2 + 2i)(32 + i)(32 + i)(32 + i)(32 + i). This product is difficult to evaluate in

Algebra & Calculus Individual Test


the decimal number system, but in ternary the calculation is a cinch! To speed things up, we will
use balanced ternary 1 , in which the three digits allowed are −1, 0, and 1 rather than 0, 1, and 2. Let
0 1 2 3
x + yi = (32 + i)(32 + i)(32 + i)(32 + i), and consider the balanced ternary representation of x and
y. For all 0 ≤ j ≤ 15, let xj denote the digit in the 3j place of x, let yj denote the digit in the 3j place
of y, and let b(j) denote the number of ones in the binary representation of j. It should be clear that
xj = −1 if b(j) ≡ 2 (mod 4), xj = 0 if b(j) ≡ 1 (mod 2), and xj = 1 if b(j) ≡ 0 (mod 4). Similarly,
yj = −1 if b(j) ≡ 1 (mod 4), yj = 0 if b(j) ≡ 0 (mod 2), and yj = 1 if b(j) ≡ 3 (mod 4). Converting
to ordinary ternary representation, we see that x = 2212112211220013 and y = 1100222022121203 . It
remains to note that a4 = 2x − 2y and b4 = 2x + 2y and perform the requisite arithmetic to arrive at
the answer above.

1 http://en.wikipedia.org/wiki/Balanced_ternary

Algebra & Calculus Individual Test


1st Annual Harvard-MIT November Tournament
Saturday 8 November 2008

Individual Round

1. [2] Find the minimum of x2 − 2x over all real numbers x.

2. [3] What is the units digit of 72009 ?

3. [3] How many diagonals does a regular undecagon (11-sided polygon) have?

4. [4] How many numbers between 1 and 1, 000, 000 are perfect squares but not perfect cubes?

5. [5] Joe has a triangle with area 3. What’s the smallest perimeter it could have?

6. [5] We say “s grows to r” if there exists some integer n > 0 such that sn = r. Call a real number r
“sparse” if there are only finitely many real numbers s that grow to r. Find all real numbers that are
sparse.

7. [6] Find all ordered pairs (x, y) such that

(x − 2y)2 + (y − 1)2 = 0.

8. [7] How many integers between 2 and 100 inclusive cannot be written as m · n, where m and n have
no common factors and neither m nor n is equal to 1? Note that there are 25 primes less than 100.

9. [7] Find the product of all real x for which

23x+1 − 17 · 22x + 2x+3 = 0.

10. [8] Find the largest positive integer n such that n3 + 4n2 − 15n − 18 is the cube of an integer.
1st Annual Harvard-MIT November Tournament
Saturday 8 November 2008

Individual Round

1. [2] Find the minimum of x2 − 2x over all real numbers x.


Answer: -1 Write x2 − 2x = x2 − 2x + 1 − 1 = (x − 1)2 − 1. Since (x − 1)2 ≥ 0, it is clear that the
minimum is −1.
Alternate method: The graph of y = x2 − 2x is a parabola that opens up. Therefore, the minimum
−(−2)
occurs at its vertex, which is at −b
2a = 2 = 1. But 12 − 2 · 1 = −1, so the minimum is −1.

2. [3] What is the units digit of 72009 ?


Answer: 7 Note that the units digits of 71 , 72 , 73 , 74 , 75 , 76 , . . . follows the pattern 7, 9, 3, 1, 7, 9, 3, 1, . . ..
The 2009th term in this sequence should be 7.
Alternate method: Note that the units digit of 74 is equal to 1, so the units digit of (74 )502 is also
1. But (74 )502 = 72008 , so the units digit of 72008 is 1, and therefore the units digit of 72009 is 7.

3. [3] How many diagonals does a regular undecagon (11-sided polygon) have?
Answer: 44 There are 8 diagonals coming from the first vertex, 8 more from the next, 7 from the
next, 6 from the next, 5 from the next, etc., and 1 from the last, for 8 + 8 + 7 + 6 + 5 + 4 + 3 + 2 + 1 = 44
total.
Third method: Each vertex has 8 diagonals touching it. There are 11 vertices. Since each diagonal
touches two vertices, this counts every diagonal twice, so there are 8·11
2 = 44 diagonals.

4. [4] How many numbers between 1 and 1, 000, 000 are perfect squares but not perfect cubes?
Answer: 990 1000000 = 10002 = 106 . A number is both a perfect square and a perfect cube if and
only if it is exactly a perfect sixth power. So, the answer is the number of perfect squares, minus the
number of perfect sixth powers, which is 1000 − 10 = 990.

5. [5] Joe has a triangle with area 3. What’s the smallest perimeter it could have?
Answer: 6 The minimum occurs for an equilateral triangle. The area of an equilateral triangle
√ √ q√
with side-length s is 43 s2 , so if the area is 3 then s = 3 √43 = 2. Multiplying by 3 to get the
perimeter yields the answer.

6. [5] We say “s grows to r” if there exists some integer n > 0 such that sn = r. Call a real number r
“sparse” if there are only finitely many real numbers s that grow to r. Find all real numbers that are
sparse.
√ √ √
Answer: -1,0,1 For any number x, other than these 3, x, 3 x, 5 x, 7 x, . . . provide infinitely many
possible values of s, so these are the only possible sparse numbers. On the other hand, −1 is the only
possible value of s for r = −1, 0 is the only value for r = 0, and −1 and 1 are the only values for r = 1.
Therefore, −1, 0, and 1 are all sparse.

7. [6] Find all ordered pairs (x, y) such that

(x − 2y)2 + (y − 1)2 = 0.

Answer: (2,1) The square of a real number is always at least 0, so to have equality we must have
(x − 2y) = 0 and (y − 1)2 = 0. Then y = 1 and x = 2y = 2.
2

1
8. [7] How many integers between 2 and 100 inclusive cannot be written as m · n, where m and n have
no common factors and neither m nor n is equal to 1? Note that there are 25 primes less than 100.
Answer: 35 A number cannot be written in the given form if and only if it is a power of a prime.
We can see this by considering the prime factorization. Suppose that k = pe11 pe22 · · · penn , with p1 , . . . , pn
primes. Then we can write m = pe11 and n = pe22 · · · penn . So, we want to find the powers of primes that
are less than or equal to 100. There are 25 primes, as given in the problem statement. The squares
of primes are 22 , 32 , 52 , 72 . The cubes of primes are 23 , 33 . The fourth powers of primes are 24 , 34 .
The fifth powers of primes are 25 , The sixth powers of primes are 26 . There are no seventh or higher
powers of primes between 2 and 100. This adds 10 non-primes to the list, so that in total there are
10 + 25 = 35 such integers.

9. [7] Find the product of all real x for which

23x+1 − 17 · 22x + 2x+3 = 0.

Answer: -3 We can re-write the equation as 2x (2·(2x )2 −17·(2x )+8) = 0, or 2·(2x )2 −17·(2x )+8 = 0.
Make the substitution

y = 2x . Then we have 2y 2 − 17y + 8 = 0, which has solutions (by the quadratic
17± 289−64
formula) y = 4 = 17±15
4 = 8, 21 , so 2x = 8, 12 and x = 3, −1. The product of these numbers is
−3.

10. [8] Find the largest positive integer n such that n3 + 4n2 − 15n − 18 is the cube of an integer.
Answer: 19 Note that the next cube after n3 is (n + 1)3 = n3 + 3n2 + 3n + 1. After that, it is
(n+2) = n +6n2 +12n+8. n3 +6n3 +12n+8 is definitely bigger than n3 +4n2 −15n−18, so the largest
3 3

cube that n3 + 4n2 − 15n − 18 could be is (n + 1)3 . On the other hand, for n ≥ 4, n3 + 4n2 − 15n − 18 is
larger than (n−2)3 = n3 −6n2 +12n−8 (as 4n2 −15n−18 > −6n2 +12n−8 ⇐⇒ 10n2 −27n−10 > 0,
which is true for n ≥ 4).
So, we will check for all solutions to n3 + 4n2 − 15n − 18 = (n − 1)3 , n3 , (n + 1)3 . The first case yields

n3 + 4n2 − 15n − 18 = n3 − 3n2 + 3n − 1 ⇐⇒ 7n2 − 18n − 17 = 0

which has no integer solutions. The second case yields

n3 + 4n2 − 15n − 18 = n3 ⇐⇒ 4n2 − 15n − 18 = 0

which also has no integer solutions. The final case yields

n3 + 4n2 − 15n − 18 = n3 + 3n2 + 3n + 1 ⇐⇒ n2 − 18n − 19 = 0

which has integer solutions n = −1, 19. So, the largest possible n is 19.

Remark: The easiest way to see that the first two polynomials have no integer solutions is using the
Rational Root Theorem, which states that the rational solutions of a polynomial axn + . . . + b are all

of the form ± ab ′ , where b′ divides b and a′ divides a.

2
2nd Annual Harvard-MIT November Tournament
Saturday 7 November 2009
General Test
1. [2] Evaluate the sum:

112 − 12 + 122 − 22 + 132 − 32 + . . . + 202 − 102 .


1 1
2. [3] Given that a + b + c = 5 and that 1 ≤ a, b, c ≤ 2, what is the minimum possible value of a+b + b+c ?
3. [3] What is the period of the function f (x) = cos(cos(x))?

4. [4] How many subsets A of {1, 2, 3, 4, 5, 6, 7, 8, 9, 10} have the property that no two elements of A sum
to 11?
5. [5] A polyhedron has faces that are all either triangles or squares. No two square faces share an edge,
and no two triangular faces share an edge. What is the ratio of the number of triangular faces to the
number of square faces?

6. [5] Find the maximum value of x + y, given that x2 + y 2 − 3y − 1 = 0.


7. [6] There are 15 stones placed in a line. In how many ways can you mark 5 of these stones so that
there are an odd number of stones between any two of the stones you marked?
8. [7] Let 4ABC be an equilateral triangle with height 13, and let O be its center. Point X is chosen
at random from all points inside 4ABC. Given that the circle of radius 1 centered at X lies entirely
inside 4ABC, what is the probability that this circle contains O?
9. [7] A set of points is convex if the points are the vertices of a convex polygon (that is, a non-self-
intersecting polygon with all angles less than or equal to 180◦ ). Let S be the set of points (x, y) such
that x and y are integers and 1 ≤ x, y ≤ 26. Find the number of ways to choose a convex subset of S
that contains exactly 98 points.
10. [8] Compute

∞  3n  3n !
Y 1 1
1− + .
n=0
2 4
2nd Annual Harvard-MIT November Tournament
Saturday 7 November 2009
General Test
1. [2] Evaluate the sum:

112 − 12 + 122 − 22 + 132 − 32 + . . . + 202 − 102 .


P10 P10
Answer: 2100 This sum can be written as a=1 (a + 10)2 − a2 = a=1 10(2a + 10) = 10 ∗ 10 ∗
11 + 10 ∗ 10 ∗ 10 = 2100.
1 1
2. [3] Given that a + b + c = 5 and that 1 ≤ a, b, c ≤ 2, what is the minimum possible value of a+b + b+c ?

Answer: 47 If a > 1 and b < 2, we can decrease the sum by decreasing a and increasing b. You can
follow a similar procedure if c > 1 and b < 2. Therefore, the sum is minimized when b = 2. We can
a+c+4 7
then cross-multiply the two fractions and see that we are trying to minimize (a+2)(c+2) = (a+2)(c+2) .
The product of two numbers with a fixed sum is maximized when those two numbers are equal, so
7 3 4
(a+2)(c+2) is minimized for a = c = 2 , which gives us an answer of 7 .

3. [3] What is the period of the function f (x) = cos(cos(x))?


Answer: π Since f (x) never equals cos(1) for x ∈ (0, π) but f (0) = cos(1), the period is at least π.
However, cos(x + π) = − cos(x), so cos(cos(x + π)) = cos(cos(x)).
4. [4] How many subsets A of {1, 2, 3, 4, 5, 6, 7, 8, 9, 10} have the property that no two elements of A sum
to 11?
Answer: 243 For each element listed, there is exactly one other element such that the two elements
sum to 11. Thus, we can list all the 10 numbers above as 5 pairs of numbers, such that each pair sums
to 11. The problem then can be solved as follows: in any given subset with no two elements summing
to 11, at most one element from each pair can be present. Thus, there are 3 ways in which each pair
can contribute to a given subset (no element, the first element in the pair, or the second element in
the pair). Since there are 5 pairs, the total number of ways to construct a subset with no two elements
summing to 11 is 35 = 243.
5. [5] A polyhedron has faces that are all either triangles or squares. No two square faces share an edge,
and no two triangular faces share an edge. What is the ratio of the number of triangular faces to the
number of square faces?
Answer: 43 Let s be the number of square faces and t be the number of triangular faces. Every
edge is adjacent to exactly one square face and one triangular face. Therefore, the number of edges is
equal to 4s, and it is also equal to 3t. Thus 4s = 3t and st = 43
6. [5] Find the maximum value of x + y, given that x2 + y 2 − 3y − 1 = 0.

26+3
Answer: 2 We can rewrite x2 + y 2 − 3y − 1 = 0 as x2 + (y − 23 )2 = 13
4 . We then see that the set

of solutions to x2 − y 2 − 3y − 1 = 0 is the circle of radius 213 and center (0, 32 ). This can be written as
√ √ √ √ √
x = 213 cos(θ) and y = 213 sin(θ)+ 23 . Thus, x+y = 23 + 213 (cos(θ)+sin(θ)) = 32 + 213 2 sin(θ +45◦ ),

which is maximized for θ = 45◦ and gives 26+3 2 . (We could also solve this geometrically by noting
that if x + y attains a maximum value of s then the line x + y = s is tangent to the circle.)
7. [6] There are 15 stones placed in a line. In how many ways can you mark 5 of these stones so that
there are an odd number of stones between any two of the stones you marked?
Answer: 77 Number the stones 1 through 15 in order. We note that the condition is equivalent
to stipulating that the stones have either all odd numbers or all even numbers. There are 85 ways to


choose 5 odd-numbered stones, and 75 ways to choose all even-numbered stones, so the total number

8 7 n
  
of ways to pick the stones is 5 + 5 = 77. ( k is the number of ways to choose k out of n items. It
n!
equals k!(n−k)! ).

8. [7] Let 4ABC be an equilateral triangle with height 13, and let O be its center. Point X is chosen
at random from all points inside 4ABC. Given that the circle of radius 1 centered at X lies entirely
inside 4ABC, what is the probability that this circle contains O?


Answer: 100 The set of points X such that the circle of radius 1 centered at X lies entirely inside
4ABC is itself a triangle, A0 B 0 C 0 , such that AB is parallel to A0 B 0 , BC is parallel to B 0 C 0 , and CA is
parallel to C 0 A0 , and furthermore AB and A0 B 0 , BC and B 0 C 0 , and CA and C 0 A0 are all 1 unit apart.
We can use this to calculate that A0 B 0 C 0 is an equilateral triangle with height 10, and hence has area
100
√ . On the other hand, the set of points X such that the circle of radius 1 centered at X contains O
3
is a circle of radius 1, centered at O, and hence has area π. The probability that the circle centered √
at
π 3π
X contains O given that it also lies in ABC is then the ratio of the two areas, that is, 100 √
= 100 .
3

9. [7] A set of points is convex if the points are the vertices of a convex polygon (that is, a non-self-
intersecting polygon with all angles less than or equal to 180◦ ). Let S be the set of points (x, y) such
that x and y are integers and 1 ≤ x, y ≤ 26. Find the number of ways to choose a convex subset of S
that contains exactly 98 points.
Answer: 4958 For this problem, let n = 26. A convex set may be divided into four subsets: a set of
points with maximal y coordinate, a set of points with minimal y coordinate, the points to the left of
one of these subsets, and the points to the right of one of these subsets (the left, top, right, and bottom
of the corresponding convex polygon). Each of these four parts contains at most n points. (All points
in the top or bottom have distinct x coordinates while all points in the left or right have distinct y
coordinates.) Moreover, there are four corners each of which is contained in two of these regions. This
implies that at most 4n − 4 distinct points are in any convex set. To find a set of size 4n − 6 we can
remove 2 additional points. Either exactly one of the top, bottom, left, or right contains exactly n − 2
points or some two of them each contain exactly n − 1 points.
Any of the 100

98 = 4950 sets of 98 points with either x or y coordinate either 1 or 26 have this property.
Suppose instead that some of the points have x coordinate and y coordinate both different from 1 and
from 26. In this case we can check that it is impossible for one side to have n−2 points. If two opposite
sides (top/bottom or left/right) have n − 1 points, then we obtain all the points on the boundary of an
n − 1 by n rectangle (of which there are four). If two adjacent sides (any of the other pairs) have n − 1
points, then we obtain the points on the boundary of an n by n square with the points (1, 1), (1, 2),
(2, 1) missing and the point (2, 2) added (or one of its rotations). There are an additional 4 such sets,
for a total of 4958.
10. [8] Compute

∞  3n  3n !
Y 1 1
1− + .
n=0
2 4

3n+1
1+( 21 )
Answer: 23 We can rewrite each term as 3n . In the infinite product, each term of the form
1+( 12 )
n
3
1 + 12

with n > 0 appears once in the numerator and once in the denominator. The only remaining
1
term is 1 + 21 in the first denominator.
3rd Annual Harvard-MIT November Tournament
Sunday 7 November 2010
General Test
1. [2] Jacob flips five coins, exactly three of which land heads. What is the probability that the first two
are both heads?
2. [3] How many sequences a1 , a2 , . . . , a8 of zeroes and ones have a1 a2 + a2 a3 + · · · + a7 a8 = 5?

3. [3] Triangle ABC has AB = 5, BC = 7, and CA = 8. New lines not containing but parallel to AB,
BC, and CA are drawn tangent to the incircle of ABC. What is the area of the hexagon formed by
the sides of the original triangle and the newly drawn lines?
4. [4] An ant starts at the point (1, 0). Each minute, it walks from its current position to one of the four
adjacent lattice points until it reaches a point (x, y) with |x| + |y| ≥ 2. What is the probability that
the ant ends at the point (1, 1)?
5. [5] A polynomial P is of the form ±x6 ± x5 ± x4 ± x3 ± x2 ± x ± 1. Given that P (2) = 27, what is
P (3)?
6. [5] What is the sum of the positive solutions to 2x2 − xbxc = 5, where bxc is the largest integer less
than or equal to x?

7. [6] What is the remainder when (1 + x)2010 is divided by 1 + x + x2 ?


8. [7] Two circles with radius one are drawn in the coordinate plane, one with center (0, 1) and the other
with center (2, y), for some real number y between 0 and 1. A third circle is drawn so as to be tangent
to both of the other two circles as well as the x axis. What is the smallest possible radius for this third
circle?
9. [7] What is the sum of all numbers between 0 and 511 inclusive that have an even number of 1s when
written in binary?
10. [8] You are given two diameters AB and CD of circle Ω with radius 1. A circle is drawn in one of the
smaller sectors formed such that it is tangent to AB at E, tangent to CD at F , and tangent to Ω at
P . Lines P E and P F intersect Ω again at X and Y . What is the length of XY , given that AC = 32 ?
3rd Annual Harvard-MIT November Tournament
Sunday 7 November 2010
General Test
1. [2] Jacob flips five coins, exactly three of which land heads. What is the probability that the first two
are both heads?
3
Answer: 10 We can associate with each sequence of coin flips a unique word where H represents
heads, and T represents tails. For example, the word HHTTH would correspond to the coin flip
sequence where the first two flips were heads, the next two were tails, and the last was heads. We are
given that exactly three of the five coin flips came up heads, so our word must be some rearrangement
of HHHTT. To calculate the total number of possibilities, any rearrangement corresponds to a choice
of three spots to place the H flips, so there are 53 = 10 possibilities. If the first two flips are both


heads, then we can only rearrange the last three  HTT flips, which corresponds to choosing one spot
for the remaining H. This can be done in 31 = 3 ways. Finally, the probability is the quotient of
3
these two, so we get the answer of 10 . Alternatively, since the total number of possiblities is small, we
can write out all rearrangements: HHHTT, HHTHT, HHTTH, HTHHT, HTHTH, HTTHH, THHHT,
THHTH, THTHH, TTHHH. Of these ten, only in the first three do we flip heads the first two times,
3
so we get the same answer of 10 .
2. [3] How many sequences a1 , a2 , . . . , a8 of zeroes and ones have a1 a2 + a2 a3 + · · · + a7 a8 = 5?
Answer: 9 First, note that we have seven terms in the left hand side, and each term can be either
0 or 1, so we must have five terms equal to 1 and two terms equal to 0. Thus, for n ∈ {1, 2, ..., 8}, at
least one of the an must be equal to 0. If we can find i, j ∈ {2, 3, ..., 7} such that ai = aj = 0 and
i < j, then the terms ai−1 ai , ai ai+1 , and aj aj+1 will all be equal to 0. We did not count any term
twice because i − 1 < i < j, so we would have three terms equal to 0, which cannot happen because
we can have only two. Thus, we can find at most one n ∈ {2, 3, ..., 7} such that an = 0. We will do
casework on which n in this range have an = 0.
If n ∈ {3, 4, 5, 6}, then we know that the terms an−1 an = an an+1 = 0, so all other terms must be 1,
so a1 a2 = a2 a3 = ... = an−2 an−1 = 1 and an+1 an+2 = ... = a7 a8 = 1. Because every ai appears in one
of these equations for i 6= n, then we must have ai = 1 for all i 6= n, so we have 1 possibility for each
choice of n and thus 4 possibilities total for this case.
If n = 2, then again we have a1 a2 = a2 a3 = 0, so we must have a3 a4 = a4 a5 = ... = a7 a8 = 1, so
a3 = a4 = ... = a8 = 1. However, this time a1 is not fixed, and we see that regardless of our choice of
a1 the sum will still be equal to 5. Thus, since there are 2 choices for a1 , then there are 2 possibilities
total for this case. The case where n = 7 is analogous, with a8 having 2 possibilities, so we have
another 2 possibilities.
Finally, if an = 1 for n ∈ {2, 3, ..., 7}, then we will have a2 a3 = a3 a4 = ... = a6 a7 = 1. We already
have five terms equal to 1, so the remaining two terms a1 a2 and a7 a8 must be 0. Since a2 = 1, then
we must have a1 = 0, and since a7 = 1 then a8 = 0. Thus, there is only 1 possibility for this case.
Summing, we have 4 + 2 + 2 + 1 = 9 total sequences.
3. [3] Triangle ABC has AB = 5, BC = 7, and CA = 8. New lines not containing but parallel to AB,
BC, and CA are drawn tangent to the incircle of ABC. What is the area of the hexagon formed by
the sides of the original triangle and the newly drawn lines?

Answer: 31 5 3

General Test
A

D E

B H G C
 
52 +82 −72
From the law of cosines we compute ]A = cos−1 2(5)(8) = 60◦ . Using brackets to denote the area
of a region, we find that

[ABC] = 12 AB · AC · sin 60◦ = 10 3.

The radius of the incircle can be computed by the formula

2[ABC]
√ √
r= AB+BC+CA = 20 3
20 = 3.
√ √ √
Now the height from A to BC is 2[ABC]
BC = 207 3 . Then the height from A to DE is 207 3 − 2r = 6 7 3 .
 √ 2
6 3/7 9
Then [ADE] = 20 √
3/7
[ABC] = 100 [ABC]. Here, we use the fact that 4ABC and 4ADE are
similar.
√ √
Similarly, we compute that the height from B to CA is 2[ABC]
CA = 208 3 = 5 2 3 . Then the height from
√ √ √ 2
B to HJ is 5 2 3 − 2r = 23 . Then [BHJ] = 5√3/23/2
1
[ABC] = 25 [ABC].
√ √
Finally, we compute that the height from C to AB is 2[ABC]
5 = 205 3 = 4 3. Then the height from C
√ √  √ 2
to F G is 4 3 − 2r = 2 3. Then [CF G] = 42√33 [ABC] = 14 [ABC].
Finally we can compute the area of hexagon DEF GHJ. We have

[DEF GHJ] = [ABC] − [ADE] − [BHJ] − [CF G] = [ABC] 1 − 9 1 1 31


 
− − = [ABC] =
√  31 √ 100 25 4 50
10 3 3150 = 5 3.

4. [4] An ant starts at the point (1, 0). Each minute, it walks from its current position to one of the four
adjacent lattice points until it reaches a point (x, y) with |x| + |y| ≥ 2. What is the probability that
the ant ends at the point (1, 1)?
7 1 1
Answer: 24 From the starting point of (1, 0), there is a 4 chance we will go directly to (1, 1), a 2
chance we will end at (2, 0) or (1, −1), and a 14 chance we will go to (0, 0). Thus, if p is the probability
that we will reach (1, 1) from (0, 0), then the desired probability is equal to 14 + 14 p, so we need only
calculate p. Note that we can replace the condition |x| + |y| ≥ 2 by |x| + |y| = 2, since in each iteration
the quantity |x| + |y| can increase by at most 1. Thus, we only have to consider the eight points
(2, 0), (1, 1), (0, 2), (−1, 1), (−2, 0), (−1, −1), (0, −2), (1, −1). Let p1 , p2 , ..., p8 be the probability of

General Test
reaching each of these points from (0, 0), respectively. By symmetry, we see that p1 = p3 = p5 = p7
and p2 = p4 = p6 = p8 . We also know that there are two paths from (0, 0) to (1, 1) and one path from
(0, 0) to (2, 0), thus p2 = 2p1 . Because the sum of all probabilities is 1, we have p1 + p2 + ... + p8 = 1.
Combining these equations, we see that 4p1 +4p2 = 12p1 = 1, so p1 = 12 1
and p2 = 16 . Since p = p2 = 16 ,
1 1 1 7
then the final answer is 4 + 4 · 6 = 24

5. [5] A polynomial P is of the form ±x6 ± x5 ± x4 ± x3 ± x2 ± x ± 1. Given that P (2) = 27, what is
P (3)?
Answer: 439 We use the following lemma:
Lemma. The sign of ±2n ± 2n−1 ± · · · ± 2 ± 1 is the same as the sign of the 2n term.
Proof. Without loss of generality, let 2n be positive. (We can flip all signs.) Notice that 2n ± 2n−1 ±
2n−2 ± · · · 2 ± 1 ≥ 2n − 2n−1 − 2n−2 − · · · − 2 − 1 = 1, which is positive.
We can use this lemma to uniquely determine the signs of P . Since our desired sum, 27, is positive, the
coefficient of x6 must be positive. Subtracting 64, we now have that ±25 ± 24 ± . . . ± 2 ± 1 = −37, so the
sign of 25 must be negative. Continuing in this manner, we find that P (x) = x6 −x5 −x4 +x3 +x2 −x+1,
so P (3) = 36 − 35 − 34 + 33 + 32 − 3 + 1 = 439.
6. [5] What is the sum of the positive solutions to 2x2 − xbxc = 5, where bxc is the largest integer less
than or equal to x?
√ √
Answer: 3+ 41+2 11
4 We first note that bxc ≤ x, so 2x2 − xbxc ≥ 2x2 − x2 = x2 . Since this function

is increasing on the positive reals, all solutions must be at most 5. This gives us 3 possible values of
bxc: 0, 1, and 2.
q
If bxc = 0, then our equation becomes 2x2 = 5, which has positive solution x = 52 . This number is
greater than 1, so its floor is not 0; thus, there are no solutions in this case.
If bxc = 1, then√our equation becomes 2x2 − x = 5. Using the quadratic formula, we find the positive

solution x = 1+4 41 . Since 3 < 41 < 7, this number is between 1 and 2, so it satisfies the equation.

If bxc = 2, then our equation becomes 2x2 − 2x = 5. We find the positive solution x = 1+ 11
. Since
√ 2
3 < 11 < 5, this number is between 2 and 3, so it satisfies the equation.
√ √ √ √
1+ 41 1+ 11 3+ 41+2 11
We then find that the sum of positive solutions is 4 + 2 = 4 .
7. [6] What is the remainder when (1 + x)2010 is divided by 1 + x + x2 ?
Answer: 1 We use polynomial congruence mod 1 + x + x2 to find the desired remainder. Since
x2 + x + 1|x3 − 1, we have that x3 ≡ 1 (mod 1 + x + x2 ). Now:

(1 + x)2010 ≡ (−x2 )2010 (mod 1 + x + x2 )


≡ x4020 (mod 1 + x + x2 )
≡ (x3 )1340 (mod 1 + x + x2 )
≡ 11340 (mod 1 + x + x2 )
≡1 (mod 1 + x + x2 )

Thus, the answer is 1.


8. [7] Two circles with radius one are drawn in the coordinate plane, one with center (0, 1) and the other
with center (2, y), for some real number y between 0 and 1. A third circle is drawn so as to be tangent
to both of the other two circles as well as the x axis. What is the smallest possible radius for this third
circle?

Answer: 3 − 2 2 Suppose that the smaller circle has radius r. Call the three circles (in order from
left to right) O1 , O2 , and O3 . The distance between the centers of O1 and O2 is 1 + r, and the distance
in their y-coordinates is 1 − r. Therefore, by the Pythagorean theorem, the difference in x-coordinates

General Test
√ √
is (1 + r)2 − (1 − r)2 = 2 r, which means that O2 has a center at (2 r, r). But O2 is also tangent
p

to O3 , which means that the difference in x-coordinate from the right-most point√of O2 to the center
of O3 √is at most 1. Therefore, the center of O3 has an x-coordinate of at most 2 r + r√+ 1, meaning

that 2 r + r√+ 1 ≤ 2. We can use the quadratic formula to see that this implies that r ≤ 2 − 1,
so r ≤ 3√− 2 2. We can achieve equality by placing the center of O3 at (2, r) (which in this case is
(2, 3 − 2 2)).

9. [7] What is the sum of all numbers between 0 and 511 inclusive that have an even number of 1s when
written in binary?
Answer: 65408 Call a digit in the binary representation of a number a bit. We claim that for any
given i between 0 and 8, there are 128 numbers with an even number of 1s that have a 1 in the bit
representing 2i . To prove this, we simply make that bit a 1, then consider all possible configurations
of the other bits, excluding the last bit (or the second-last bit if our given bit is already the last bit).
The last bit will then be restricted to satisfy the parity condition on the number of 1s. As there are
128 possible configurations of all the bits but two, we find 128 possible numbers, proving our claim.
Therefore, each bit is present as a 1 in 128 numbers in the sum, so the bit representing 2i contributes
128 · 2i to our sum. Summing over all 0 ≤ i ≤ 8, we find the answer to be 128(1 + 2 + . . . + 128) =
128 · 511 = 65408.
10. [8] You are given two diameters AB and CD of circle Ω with radius 1. A circle is drawn in one of the
smaller sectors formed such that it is tangent to AB at E, tangent to CD at F , and tangent to Ω at
P . Lines P E and P F intersect Ω again at X and Y . What is the length of XY , given that AC = 23 ?

Answer: 4 3 2 Let O denote the center of circle Ω. We first prove that OX ⊥ AB and OY ⊥ CD.
Consider the homothety about P which maps the smaller circle to Ω. This homothety takes E to X
and also takes AB to the line tangent to circle Ω parallel to AB. Therefore, X is the midpoint of the
arc AB, and so OX ⊥ AB. Similarly, OY ⊥ CD.
q
Let θ = ∠AOC. By the Law of Sines, we have AC = 2 sin θ2 . Thus, sin θ2 = 13 , and cos θ2 = 1 − ( 13 )2 =

2 2
3 . Therefore,

∠XOY
XY = 2 sin
 2
θ


= 2 sin 90 −
2
θ
= 2 cos
√ 2
4 2
= .
3

General Test
4th Annual Harvard-MIT November Tournament
Saturday 12 November 2011
General Test
1. [3] Find all ordered pairs of real numbers (x, y) such that x2 y = 3 and x + xy = 4.
2. [3] Let ABC be a triangle, and let D, E, and F be the midpoints of sides BC, CA, and AB, respectively.
Let the angle bisectors of ∠F DE and ∠F BD meet at P . Given that ∠BAC = 37◦ and ∠CBA = 85◦ ,
determine the degree measure of ∠BP D.

3. [4] Alberto, Bernardo, and Carlos are collectively listening to three different songs. Each is simultane-
ously listening to exactly two songs, and each song is being listened to by exactly two people. In how
many ways can this occur?

4. [4] Determine the remainder when


1·2 2·3 2011·2012
2 2 +2 2 + ··· + 2 2

is divided by 7.

5. [5] Find all real values of x for which

1 1 1
√ √ +√ √ = .
x+ x−2 x+2+ x 4

6. [5] Five people of heights 65, 66, 67, 68, and 69 inches stand facing forwards in a line. How many
orders are there for them to line up, if no person can stand immediately before or after someone who
is exactly 1 inch taller or exactly 1 inch shorter than himself?

7. [5] Determine the number of angles θ between 0 and 2π, other than integer multiples of π/2, such that
the quantities sin θ, cos θ, and tan θ form a geometric sequence in some order.

8. [6] Find the number of integers x such that the following three conditions all hold:

• x is a multiple of 5
• 121 < x < 1331
• When x is written as an integer in base 11 with no leading 0s (i.e. no 0s at the very left), its
rightmost digit is strictly greater than its leftmost digit.

9. [7] Let P and Q be points on line l with P Q = 12. Two circles, ω and Ω, are both tangent to l at P
and are externally tangent to each other. A line through Q intersects ω at A and B, with A closer to
Q than B, such that AB = 10. Similarly, another line through Q intersects Ω at C and D, with C
closer to Q than D, such that CD = 7. Find the ratio AD/BC.

10. [8] Let r1 , r2 , . . . , r7 be the distinct complex roots of the polynomial P (x) = x7 − 7. Let
Y
K= (ri + rj ),
1≤i<j≤7

that is, the product of all numbers of the form ri +rj , where i and j are integers for which 1 ≤ i < j ≤ 7.
Determine the value of K 2 .
4th Annual Harvard-MIT November Tournament
Saturday 12 November 2011
General Test
1. [3] Find all ordered pairs of real numbers (x, y) such that x2 y = 3 and x + xy = 4.
Answer: (1, 3), (3, 13 ) Multiplying the second equation by x gives

x2 + x2 y = 4x,

and substituting our known value of x2 y gives the quadratic

x2 − 4x + 3 = 0,

so x = 1 or x = 3. Hence, we obtain the solutions (x, y) = (1, 3), (3, 1/3).


2. [3] Let ABC be a triangle, and let D, E, and F be the midpoints of sides BC, CA, and AB, respectively.
Let the angle bisectors of ∠F DE and ∠F BD meet at P . Given that ∠BAC = 37◦ and ∠CBA = 85◦ ,
determine the degree measure of ∠BP D.
Answer: 61◦ Because D, E, F are midpoints, we have ABC ∼ DEF . Furthermore, we know that
F D k AC and DE k AB, so we have

∠BDF = ∠BCA = 180 − 37 − 85 = 58◦ .

Also, ∠F DE = ∠BAC = 37◦ . Hence, we have


85◦ 37◦
µ ¶
◦ ◦
∠BP D = 180 − ∠P BD − ∠P DB = 180 − − + 58◦ = 61◦ .
2 2

37

E
F

85 58

B D C

3. [4] Alberto, Bernardo, and Carlos are collectively listening to three different songs. Each is simultane-
ously listening to exactly two songs, and each song is being listened to by exactly two people. In how
many ways can this occur?
µ ¶
3
Answer: 6 We have = 3 choices for the songs that Alberto is listening to. Then, Bernardo and
2
Carlos must both be listening to the third song. Thus, there are 2 choices for the song that Bernardo
shares with Alberto. From here, we see that the songs that everyone is listening to are forced. Thus,
there are a total of 3 × 2 = 6 ways for the three to be listening to songs.

General Test
4. [4] Determine the remainder when
1·2 2·3 2011·2012
2 2 +2 2 + ··· + 2 2

is divided by 7.
Answer: 1 We have that 23 ≡ 1 (mod 7). Hence, it suffices to consider the exponents modulo
3. We note that the exponents are the triangular number and upon division by 3 give the pattern of
remainders 1, 0, 0, 1, 0, 0, . . ., so what we want is
1·2 2011·2012
2 2 + ··· + 2 2 ≡ 21 + 20 + 20 + 21 + . . . + 20 + 21 (mod 7)
2010 1
≡ (2 + 20 + 20 ) + 21
3
≡ (670)(4) + 2
≡ 1.

5. [5] Find all real values of x for which

1 1 1
√ √ +√ √ = .
x+ x−2 x+2+ x 4

257
Answer: 16 We note that

1 1 1
=√ √ +√ √
4 x+ x−2 x+2+ x
√ √ √ √
x− x−2 x+2− x
= √ √ √ √ + √ √ √ √
( x + x − 2)( x − x − 2) ( x + 2 + x)( x + 2 − x)
√ √ √ √
x− x−2 x+2− x
= +
2 2
1 √ √
= ( x + 2 − x − 2),
2
so that √ √
2 x + 2 − 2 x − 2 = 1.
Squaring, we get that
p p
8x − 8 (x + 2)(x − 2) = 1 ⇒ 8x − 1 = 8 (x + 2)(x − 2).

Squaring again gives


64x2 − 16x + 1 = 64x2 − 256,
257
so we get that x = 16 .

6. [5] Five people of heights 65, 66, 67, 68, and 69 inches stand facing forwards in a line. How many
orders are there for them to line up, if no person can stand immediately before or after someone who
is exactly 1 inch taller or exactly 1 inch shorter than himself?
Answer: 14 Let the people be A, B, C, D, E so that their heights are in that order, with A the
tallest and E the shortest. We will do casework based on the position of C.

• Case 1: C is in the middle. Then, B must be on one of the two ends, for two choices. This leaves
only one choice for D–the other end. Then, we know the positions of A and E since A cannot
neighbor B and E cannot neighbor D. So we have 2 options for this case.

General Test
• Case 2: C is in the second or fourth spot. Then, we have two choices for the position of C.
Without loss of generality, let C be in the second spot. Then, the first and third spots must be A
and E, giving us two options. This fixes the positions of B and D, so we have a total of 2 × 2 = 4
options for this case.
• Case 3: C is in the first or last spot. Then, we have two choices for the position of C. Without
loss of generality, let it be in the first spot. Either A or E is in the second spot, giving us two
choices. Without loss of generality, let it be A. Then, if D is in the third spot, the positions of B
and E are fixed. If E is in third spot, the positions of B and D are fixed, so we have a total of
2 × 2 × (1 + 1) = 8 options for this case.
Hence, we have a total of 2 + 4 + 8 = 14 possibilities.
7. [5] Determine the number of angles θ between 0 and 2π, other than integer multiples of π/2, such that
the quantities sin θ, cos θ, and tan θ form a geometric sequence in some order.
Answer: 4 If sin θ, cos θ, and tan θ are in a geometric progression, then the product of two must
equal the square of the third. Using this criterion, we have 3 cases.
• Case 1: sin θ · tan θ = cos2 θ. This implies that (sin2 θ) = (cos3 θ). Writing sin2 θ as 1 − cos2 θ
and letting cos θ = x, we have that x3 + x2 − 1 = 0. We wish to find the number of solutions of
this where |x| ≤ 1. Clearly −1 is not a root. If −1 < x ≤ 0, we have that x2 + x3 ≤ x2 < 1 so
x3 + x2 − 1 < 0 and there are no roots. If 0 < x ≤ 1, then x3 + x2 − 1 is a strictly increasing
function. Since it has value −1 at x = 0 and value 1 and x = 1, there is exactly one root between
0 and 1, non-inclusive. There are 2 values of θ such that cos θ equals this root, and thus, two
solutions in this case.
• Case 2: sin θ · cos θ = tan2 θ. This implies that cos3 θ = sin θ. To find the number of solutions in
this case, we can analyze the graphs of the functions in different ranges. Note that from θ = 0
to θ = π2 , cos3 θ decreases strictly from 1 to 0 while sin θ increases strictly from 0 to 1. Hence,
there is one solution in this range. By a similar argument, a solution exists between θ = π and
π
θ = 3π 3π
2 . In the intervals [ 2 , π] and [ 2 , 2π], we have that one function is negative and the other
is positive, so there are no solutions. Thus, there are two solutions in this case.
• Case 3: cos θ · tan θ = sin2 θ. This implies that sin θ = sin2 θ, so sin θ = 0, 1. Clearly the only
solutions of these have θ as an integer multiple of π2 . Thus, there are no pertinent solutions int
his case.
We can see that the solutions for the first two cases are mutually exclusive. Hence, there are 4 solutions
in total.
8. [6] Find the number of integers x such that the following three conditions all hold:
• x is a multiple of 5
• 121 < x < 1331
• When x is written as an integer in base 11 with no leading 0s (i.e. no 0s at the very left), its
rightmost digit is strictly greater than its leftmost digit.
Answer: 99 We will work in base 11, so let x = def 11 such that d > 0. Then, based on the first
two conditions, we aim to find multiples of 5 between 10011 and 100011 . We note that

def 11 ≡ 112 · d + 11 · e + f ≡ d + e + f (mod 5).

Hence, x a multiple of 5 if and only if the sum of its digits is a multiple of 5. Thus, we wish to find
triples (d, e, f ) with elements in 0, 1, 2, · · · , 9, 10 such that d + e + f ≡ 0 (mod 5) and 0 < d < f .
Note that if we choose d and f such that d < f , there is exactly one value of e modulo 5 that would
make d + e + f ≡ 0 (mod 5). Once the this value of e is fixed, then there are two possibilities for e
unless e ≡ 0 (mod 5), in which case there are three possibilities. Thus, our answer is twice the number
of ways to choose d and f such that 0 < d < f plus the number of ways to choose d and f such that

General Test
d + f ≡ 0 (mod 5) and 0 < d < f (to account ¡ ¢for the extra choice for the value of e). Note that the
number of ways to choose 0 < d < f is just 10 2 since any any choice of two digits yields exactly one
way to order them. The number of ways to choose d + f ≡ 0 (mod 5) and 0 < d < f can be found by
listing: (d, f ) = (1, 4), (1, 9), (2, 3), (2, 8), (3, 7), (4, 6), (5, 10), (6, 9), (7, 8), for 9 such pairings.
µ ¶
10
Hence, the total is 2 + 9 = 99 possibilities for x.
2
9. [7] Let P and Q be points on line l with P Q = 12. Two circles, ω and Ω, are both tangent to l at P
and are externally tangent to each other. A line through Q intersects ω at A and B, with A closer to
Q than B, such that AB = 10. Similarly, another line through Q intersects Ω at C and D, with C
closer to Q than D, such that CD = 7. Find the ratio AD/BC.
8
Answer: 9 We first apply the Power of a Point theorem repeatedly. Note that QA · QB = QP 2 =
QC · QD. Substituting in our known values, we obtain QA(QA + 10) = 122 = QC(QC + 7). Solving
these quadratics, we get that QA = 8 and QC = 9.
AQ CQ
We can see that DQ = BQ and that ∠AQD = ∠CQB, so QAD ∼ QCB . (Alternatively, going
back to the equality QA · QB = QC · QD, we realize that this is just a Power of a Point theorem on
the quadrilateral ABDC, and so this quadrilateral is cyclic. This implies that ∠ADQ = ∠ADC =
AQ
∠ABC = ∠QBC.) Thus, AD 8
BC = QC = 9 .

7

C

P 12 Q

ω 8
A

10

10. [8] Let r1 , r2 , . . . , r7 be the distinct complex roots of the polynomial P (x) = x7 − 7. Let
Y
K= (ri + rj ),
1≤i<j≤7

that is, the product of all numbers of the form ri +rj , where i and j are integers for which 1 ≤ i < j ≤ 7.
Determine the value of K 2 .
Answer: 117649 We first note that x7 − 7 = (x − r1 )(x − r2 ) · · · (x − r7 ), which implies, replacing
x by −x and taking the negative of the equation, that (x + r1 )(x + r2 ) · · · (x + r7 ) = x7 + 7. Also note
that the product of the ri is just the constant term, so r1 r2 · · · r7 = 7.

General Test
Now, we have that
7
Y
27 · 7 · K 2 = ( 2ri )K 2
i=1
7
Y Y
= 2ri (ri + rj )2
i=1 1≤i<j≤7
Y Y Y
= (ri + rj ) (ri + rj ) (ri + rj )
1≤i=j≤7 1≤i<j≤7 1≤j<i≤7
Y
= (ri + rj )
1≤i,j≤7
7 Y
Y 7
= (ri + rj ).
i=1 j=1

7
Y
However, note that for any fixed i, (ri + rj ) is just the result of substuting x = ri into (x + r1 )(x +
j=1
r2 ) · · · (x + r7 ). Hence,
7
Y
(ri + rj ) = ri7 + 7 = (ri7 − 7) + 14 = 14.
j=1

Therefore, taking the product over all i gives 147 , which yields K 2 = 76 = 117649.

General Test
GENERAL TEST
This test consists of 10 short-answer problems to be solved individually in 50 minutes. Problems are unequally
weighted with point values as shown in brackets on the answer form. The maximum possible score is 50 points.
There is no point penalty for guessing, though in the case of a tie it is slightly more advantageous not to answer
than to answer incorrectly.

No translators, books, notes, slide rules, calculators, abaci, or other computational aids are permitted. Similarly,
graph paper, rulers, protractors, compasses, and other drawing aids are not permitted.

Answers should be simplified as much as is reasonably possible and must be exact unless otherwise specified. Ra-
tional numbers should be written in lowest terms, although denominators of irrationals need not be rationalized.
An nth root should be simplified so that the radicand is not divisible by the nth power of any prime.

Correct mathematical notation must be used. No partial credit will be given unless otherwise specified.

If you believe the test contains an error, please submit your protest in writing to the Science Center Lobby during
lunchtime.

Enjoy!
HMMT November 2012
Saturday 10 November 2012
General Test
1. [3] What is the sum of all of the distinct prime factors of 253 − 272 ?

2. [3] Let Q(x) = x2 + 2x + 3, and suppose that P (x) is a polynomial such that

P (Q(x)) = x6 + 6x5 + 18x4 + 32x3 + 35x2 + 22x + 8.

Compute P (2).

3. [3] ABCD is a rectangle with AB = 20 and BC = 3. A circle with radius 5, centered at the midpoint
of DC, meets the rectangle at four points: W , X, Y , and Z. Find the area of quadrilateral W XY Z.

4. [4] If you roll four fair 6-sided dice, what is the probability that at least three of them will show the
same value?

5. [4] How many ways are there to arrange three indistinguishable rooks on a 6 × 6 board such that no
two rooks are attacking each other? (Two rooks are attacking each other if and only if they are in the
same row or the same column.)

6. [5] ABCD is a parallelogram satisfying AB = 7, BC = 2, and ∠DAB = 120◦ . Parallelogram


ECF A is contained in ABCD and is similar to it. Find the ratio of the area of ECF A to the area of
ABCD.

7. [6] Find the number of ordered 2012-tuples of integers (x1 , x2 , . . . , x2012 ), with each integer between
0 and 2011 inclusive, such that the sum x1 + 2x2 + 3x3 + · · · + 2012x2012 is divisible by 2012.
8. [7] Let n be the 200th smallest positive real solution to the equation x − π2 = tan x. Find the greatest
integer that does not exceed n2 .

9. [7] Consider triangle ABC where BC = 7, CA = 8, and AB = 9. D and E are the midpoints of BC
and CA, respectively, and AD and BE meet at G. The reflection of G across D is G′ , and G′ E meets
CG at P . Find the length P G.

10. [8] Let α and β be reals. Find the least possible value of

(2 cos α + 5 sin β − 8)2 + (2 sin α + 5 cos β − 15)2 .


HMMT November 2012
Saturday 10 November 2012
General Test

Name Team ID#

School Team

1. [3]
2. [3]
3. [3]
4. [4]
5. [4]
6. [5]
7. [6]
8. [7]
9. [7]
10. [8]

Score:
HMMT November 2012
Saturday 10 November 2012
General Test
1. [3] What is the sum of all of the distinct prime factors of 253 − 272 ?
Answer: 28 We note that 253 − 272 = 56 − 36 = (53 − 33 )(53 + 33 ) = (5 − 3)(52 + 5 · 3 + 32 )(5 +
3)(52 − 5 · 3 + 32 ) = 2 · 72 · 23 · 19, so the sum of the distinct prime factors is 2 + 7 + 19 = 28.

2. [3] Let Q(x) = x2 + 2x + 3, and suppose that P (x) is a polynomial such that

P (Q(x)) = x6 + 6x5 + 18x4 + 32x3 + 35x2 + 22x + 8.

Compute P (2).
Answer: 2 Note that Q(−1) = 2. Therefore, P (2) = P (Q(−1)) = 1−6+18−32+35−22+8 = 2.

3. [3] ABCD is a rectangle with AB = 20 and BC = 3. A circle with radius 5, centered at the midpoint
of DC, meets the rectangle at four points: W , X, Y , and Z. Find the area of quadrilateral W XY Z.
Answer: 27 Suppose that X and Y are located on AB with X closer to A than B. Let O be the
center of the circle, and let P be the midpoint of AB. We have OP ⊥ AB so OP X and OP Y are
right triangles with right angles at P . Because OX = OY = 5 and OP = 3, we have XP = P Y = 4
by the Pythagorean theorem. Now, W XY Z is a¡trapezoid with W Z = W O + OZ = 5 + 5 = 10 ,
XY = XP + P Y = 8, and height 3, so its area is 10+8
¢
2 × 3 = 27.
4. [4] If you roll four fair 6-sided dice, what is the probability that at least three of them will show the
same value?
7
Answer: 72 We have two cases: either three of the dice show one value and the last shows a
different value, or all four dice show¡the
¢ same value. In the first case, there are six choices for the value
of the dice which are the same and 43 choice for which dice show that value. Then there are 5 choices
for the last die. In total, there are 6 43 5 = 120 possibilities. For the second case, there are 6 values
¡¢

that the last die can show. Consequently, the overall probability is, 120+6
64 = 126 7
64 = 72 .

5. [4] How many ways are there to arrange three indistinguishable rooks on a 6 × 6 board such that no
two rooks are attacking each other? (Two rooks are attacking each other if and only if they are in the
same row or the same column.)
Answer: 2400 There are 6 × 6 = 36 possible places to place the first rook. Since it cannot be in
the same row or column as the first, the second rook has 5 × 5 = 25 possible places, and similarly, the
third rook has 4 × 4 = 16 possible places. However, the rooks are indistinguishable, so there are 3! = 6
ways to reorder them. Therefore, the number of arrangements is 36×25×16
6 = 2400.
6. [5] ABCD is a parallelogram satisfying AB = 7, BC = 2, and ∠DAB = 120◦ . Parallelogram ECF A
is contained in ABCD and is similar to it. Find the ratio of the area of ECF A to the area of ABCD.
39
Answer: 67 First, note that BD is the long diagonal of ABCD, and AC is the long diagonal of
ECF A. Because the ratio of the areas of similar figures is equal to the square of the ratio of their side
AC 2
lengths, we know that the ratio of the area of ECF A to the area of ABCD is equal to the ratio BD 2.

Using law of cosines on triangle ABD, we have BD2 = AD2 + AB 2 − 2(AD)(AB) cos(120◦ ) = 22 +
72 − 2(2)(7)(− 12 ) = 67.
Using law of cosines on triangle ABC, we have AC 2 = AB 2 + BC 2 − 2(AB)(BC) cos(60◦ ) = 72 + 22 −
2(7)(2)( 12 ) = 39.
AC 2 39
Finally, BD 2 = 67 .

General Test
7. [6] Find the number of ordered 2012-tuples of integers (x1 , x2 , . . . , x2012 ), with each integer between
0 and 2011 inclusive, such that the sum x1 + 2x2 + 3x3 + · · · + 2012x2012 is divisible by 2012.
Answer: 20122011 We claim that for any choice of x2 , x3 , ..., x2012 , there is exactly one possible
value of x1 satisfying the condition. We have x1 + 2x2 + ... + 2012x2012 ≡ 0 (mod 2012) or x1 ≡
−(2x2 + ... + 2012x2012 ) (mod 2012). Indeed, we see that the right hand side is always an integer
between 0 and 2011, so x1 must equal this number.
Now, there are 2012 choices for each of the 2011 variables x2 , ..., x2012 , and each of the 20122011 possible
combinations gives exactly one valid solution, so the total number of 2012-tuples is 20122011 .

8. [7] Let n be the 200th smallest positive real solution to the equation x − π2 = tan x. Find the greatest
integer that does not exceed n2 .
π
Answer: 314 Drawing the graphs of the functions ³ y = x− 2 and y´= tan x, we may observe that the
graphs intersect exactly once in each of the intervals (2k−1)π
2 , (2k+1)π
2 for each k = 1, 2, · · · . Hence, the
200th intersection has x in the range ( 399π 401π π
2 , 2 ). At this intersection, y = x − 2 is large, and thus, the
401π 401π
intersection will be slightly less than 2 . We have that ⌊ 4 ⌋ = ⌊100π + 4 ⌋ = ⌊314.16+ π4 ⌋ = 314.
π

9. [7] Consider triangle ABC where BC = 7, CA = 8, and AB = 9. D and E are the midpoints of BC
and CA, respectively, and AD and BE meet at G. The reflection of G across D is G′ , and G′ E meets
CG at P . Find the length P G.

145 1
Answer: 9 Observe that since G′ is a reflection and GD = 2 AG, we have AG = GG′ and
1
therefore, P is the centroid of triangle ACG′ . Thus, extending CG to hit AB at F , P G = 3 CG =
q √
2 2 2(82 +72 )−92 145
9 CF = 9 4 = 9 by the formula for the length of a median.

10. [8] Let α and β be reals. Find the least possible value of

(2 cos α + 5 sin β − 8)2 + (2 sin α + 5 cos β − 15)2 .

Answer: 100 Let the vector − →v = (2 cos α, 2 sin α) and −



w = (5 sin β, 5 cos β). The locus of ends of

→ −

vectors expressible in the form v + w are the points which are five units away from a point on the
circle of radius two about the origin. The expression that we desire to minimize is the square of the
distance from this point to X = (8, 15). Thus, the closest distance from such a point to X is when
the point is 7 units away from the origin along the segment from the origin to X. Thus, since X is 17
units away from the origin, the minimum is 102 = 100.

General Test
HMMT November 2013
Saturday 9 November 2013
General Test
1. [2] What is the smallest non-square positive integer that is the product of four prime numbers (not
necessarily distinct)?
2. [3] Plot points A, B, C at coordinates (0, 0), (0, 1), and (1, 1) in the plane, respectively. Let S denote
the union of the two line segments AB and BC. Let X1 be the area swept out when Bobby rotates
S counterclockwise 45 degrees about point A. Let X2 be the area swept out when Calvin rotates S
clockwise 45 degrees about point A. Find X1 +X 2
2
.

3. [4] A 24-hour digital clock shows times h : m : s, where h, m, and s are integers with 0 ≤ h ≤ 23,
0 ≤ m ≤ 59, and 0 ≤ s ≤ 59. How many times h : m : s satisfy h + m = s?
4. [4] A 50-card deck consists of 4 cards labeled “i” for i = 1, 2, . . . , 12 and 2 cards labeled “13”. If Bob
randomly chooses 2 cards from the deck without replacement, what is the probability that his 2 cards
have the same label?

5. [5] Let ABC be an isosceles triangle with AB = AC. Let D and E be the midpoints of segments
−−→
AB and AC, respectively. Suppose that there exists a point F on ray DE outside of ABC such that
AB
triangle BF A is similar to triangle ABC. Compute BC .
6. [5] Find the number of positive integer divisors of 12! that leave a remainder of 1 when divided by 3.
7. [6] Find the largest real number λ such that a2 + b2 + c2 + d2 ≥ ab + λbc + cd for all real numbers
a, b, c, d.

8. [6] How many of the first 1000 positive integers can be written as the sum of finitely many distinct
numbers from the sequence 30 , 31 , 32 , . . .?
√ √
9. [7] Let ABC be a triangle and D a point on BC such that AB = 2, AC = 3, ∠BAD = 30◦ , and
∠CAD = 45◦ . Find AD.
10. [8] How many functions f : {1, 2, . . . , 2013} → {1, 2, . . . , 2013} satisfy f (j) < f (i)+j −i for all integers
i, j such that 1 ≤ i < j ≤ 2013?
HMMT November 2013
Saturday 9 November 2013
General Test
1. [2] What is the smallest non-square positive integer that is the product of four prime numbers (not
necessarily distinct)?
Answer: 24 The smallest two integers that are the product of four primes are 24 = 16 and
3
2 · 3 = 24. Since 16 is a perfect square and 24 is not, the answer is 24.

2. [3] Plot points A, B, C at coordinates (0, 0), (0, 1), and (1, 1) in the plane, respectively. Let S denote
the union of the two line segments AB and BC. Let X1 be the area swept out when Bobby rotates
S counterclockwise 45 degrees about point A. Let X2 be the area swept out when Calvin rotates S
clockwise 45 degrees about point A. Find X1 +X 2
2
.
π
Answer: 4 It’s easy to see X1 = X2 . Simple cutting and pasting shows that X1 equals the area
√ √
of 1
8 of a circle with radius AC = 2, so X1 +X 2
2
= X1 = 81 π( 2)2 = π4 .
3. [4] A 24-hour digital clock shows times h : m : s, where h, m, and s are integers with 0 ≤ h ≤ 23,
0 ≤ m ≤ 59, and 0 ≤ s ≤ 59. How many times h : m : s satisfy h + m = s?
Answer: 1164 We are solving h + m = s in 0 ≤ s ≤ 59, 0 ≤ m ≤ 59, and 0 ≤ h ≤ 23. If s ≥ 24,
each h corresponds to exactly 1 solution, so we get 24(59 − 23) = 24(36) in this case. If s ≤ 23, we want
the number of nonnegative integer solutions to h + m ≤ 23, which by lattice point counting (or balls
and urns) is 23+2

2 = (23 + 2)(23 + 1)/2 = 25 · 12. Thus our total is 12(72 + 25) = 12(100 − 3) = 1164.
4. [4] A 50-card deck consists of 4 cards labeled “i” for i = 1, 2, . . . , 12 and 2 cards labeled “13”. If Bob
randomly chooses 2 cards from the deck without replacement, what is the probability that his 2 cards
have the same label?
73
Answer: All pairs of distinct cards (where we distinguish cards even with the same label) are
1225
equally likely. There are 22 + 12 42 = 73 pairs of cards with the same label and 50 49
  
2 = 100 · 4 = 1225
73
pairs of cards overall, so the desired probability is 1225 .
5. [5] Let ABC be an isosceles triangle with AB = AC. Let D and E be the midpoints of segments
−−→
AB and AC, respectively. Suppose that there exists a point F on ray DE outside of ABC such that
AB
triangle BF A is similar to triangle ABC. Compute BC .

Answer: 2 Let α = ∠ABC = ∠ACB, AB = 2x, and BC = 2y, so AD = DB = AE = EC = x
and DE = y. Since 4BF A ∼ 4ABC and BA = AC, we in fact have 4BF A ∼ = 4ABC, so
BF = BA = 2x, F A = 2y, and ∠DAF = α. But DE k BC yields √ ∠ADF = ∠ABC = α as well,
whence 4F AD ∼ 4ABC gives 2y FA AB 2x AB
x = AD = BC = 2y =⇒ BC = y =
x
2.

6. [5] Find the number of positive integer divisors of 12! that leave a remainder of 1 when divided by 3.
Answer: 66 First we factor 12! = 210 35 52 71 111 , and note that 2, 5, 11 ≡ −1 (mod 3) while 7 ≡ 1
(mod 3). The desired divisors are precisely 2a 5b 7c 11d with 0 ≤ a ≤ 10, 0 ≤ b ≤ 2, 0 ≤ c ≤ 1, 0 ≤ d ≤ 1,
and a + b + d even. But then for any choice of a, b, exactly one d ∈ {0, 1} makes a + b + d even, so we
have exactly one 1 (mod 3)-divisor for every triple (a, b, c) satisfying the inequality constraints. This
gives a total of (10 + 1)(2 + 1)(1 + 1) = 66.
7. [6] Find the largest real number λ such that a2 + b2 + c2 + d2 ≥ ab + λbc + cd for all real numbers
a, b, c, d.
3
Answer: 2 Let f (a, b, c, d) = (a2 + b2 + c2 + d2 ) − (ab + λbc + cd). For fixed (b, c, d), f is minimized
at a = 2b , and for fixed (a, b, c), f is minimized at d = 2c , so simply we want the largest λ such that
f ( 2b , b, c, 2c ) = 43 (b2 + c2 ) − λbc is always nonnegative. By AM-GM, this holds if and only if λ ≤ 2 34 = 32 .
8. [6] How many of the first 1000 positive integers can be written as the sum of finitely many distinct
numbers from the sequence 30 , 31 , 32 , . . .?
Answer: 105 We want to find which integers have only 0’s and 1’s in their base 3 representation.
Note that 100010 = 11010013 . We can construct a bijection from all such numbers to the binary strings,
by mapping x3 ↔ x2 . Since 11010012 = 10510 , we conclude that the answer is 105.
√ √
9. [7] Let ABC be a triangle and D a point on BC such that AB = 2, AC = 3, ∠BAD = 30◦ , and
∠CAD = 45◦ . Find AD.
√ √
6 √3
Answer: 2 OR 2
Note that [BAD] + [CAD] = [ABC]. If α1 = ∠BAD, α2 = ∠CAD, then
sin(α1 +α2 ) sin α1 sin α2
we deduce AD = AC + AB upon division by AB · AC · AD. Now

sin(30◦ + 45◦ )
AD = sin√30◦ ◦ .

3
+ sin√45
2
√ √
6 sin√30◦ sin√45◦
But sin(30◦ + 45◦ ) = sin 30◦ cos 45◦ + sin 45◦ cos 30◦ = sin 30◦ √12 + sin 45◦ 2
3
= 2 ( 3
+ 2
), so

6
our answer is 2 .

10. [8] How many functions f : {1, 2, . . . , 2013} → {1, 2, . . . , 2013} satisfy f (j) < f (i)+j −i for all integers
i, j such that 1 ≤ i < j ≤ 2013?
4025

Answer: 2013 Note that the given condition is equivalent to f (j) − j < f (i) − i for all
1 ≤ i < j ≤ 2013. Let g(i) = f (i) − i, so that the condition becomes g(j) < g(i) for i < j and
1 − i ≤ g(i) ≤ 2013 − i. However, since g is decreasing, we see by induction that g(i + 1) is in
the desired range so long as g(i) is in the desired range. Hence, it sufficesto choose 2013 values for
g(1), . . . , g(2013) in decreasing order from [−2012, 2012], for a total of 4025
2013 possible functions.

Potrebbero piacerti anche